Download as pdf or txt
Download as pdf or txt
You are on page 1of 202

A Practical Guide to

Point of Care
Ultrasound (POCUS)
Arunangshu Chakraborty
Balakrishnan Ashokka
Editors

123
A Practical Guide to Point of Care
Ultrasound (POCUS)
Arunangshu Chakraborty 
Balakrishnan Ashokka
Editors

A Practical Guide
to Point of Care
Ultrasound (POCUS)
Editors
Arunangshu Chakraborty Balakrishnan Ashokka
Department of Anaesthesia Department of Anaesthesia
Critical Care and Pain, Tata National University Hospital
Medical Center Singapore, Singapore
Newtown, Kolkata, West Bengal, India

ISBN 978-981-16-7686-4    ISBN 978-981-16-7687-1 (eBook)


https://doi.org/10.1007/978-981-16-7687-1

© The Editor(s) (if applicable) and The Author(s), under exclusive license to Springer Nature
Singapore Pte Ltd. 2022
This work is subject to copyright. All rights are solely and exclusively licensed by the Publisher,
whether the whole or part of the material is concerned, specifically the rights of translation,
reprinting, reuse of illustrations, recitation, broadcasting, reproduction on microfilms or in any
other physical way, and transmission or information storage and retrieval, electronic adaptation,
computer software, or by similar or dissimilar methodology now known or hereafter developed.
The use of general descriptive names, registered names, trademarks, service marks, etc. in this
publication does not imply, even in the absence of a specific statement, that such names are
exempt from the relevant protective laws and regulations and therefore free for general use.
The publisher, the authors, and the editors are safe to assume that the advice and information in
this book are believed to be true and accurate at the date of publication. Neither the publisher nor
the authors or the editors give a warranty, expressed or implied, with respect to the material
contained herein or for any errors or omissions that may have been made. The publisher remains
neutral with regard to jurisdictional claims in published maps and institutional affiliations.

This Springer imprint is published by the registered company Springer Nature Singapore Pte Ltd.
The registered company address is: 152 Beach Road, #21-01/04 Gateway East, Singapore
189721, Singapore
I dedicate this work to my better half Ananya, for her
unrelenting support and our children, Anwesh and Agniv for
being my inspiration.
Arunangshu Chakraborty
I dedicate this academic journey to ‘Tanvi Ashokka’, my tween
twinkle, for helping me ‘Focus’ on that matters beyond the
‘POCUS’.
Balakrishnan Ashokka
Foreword

Although the concept of ultrasound is over 200 years old, the history of using
ultrasound as a medical diagnostic tool began only about 70 years ago. Soon,
the principle was also found to be an excellent tool for viewing the heart, and
echocardiography was born. Strictly being in the radiologists’ domain for the
first 50 years, it caught the attention of the intensivists and the emergency
physicians. They found ultrasound to be an excellent tool for inexpensive,
non-invasive and bedside imaging. They started using it for even hitherto
uncharted areas like lung ultrasonography. These physicians were not trained
radiologists, but they needed this valuable imaging tool throughout the day.
So, a term point-of-care ultrasonography was born. It was the use of the ultra-
sound machine to answer some specific questions and assist in some specific
procedures, especially needed in critical care. We are already seeing the
future when an ultrasound transducer attached to our smartphone will replace
the stethoscope for the emergency and critical care physicians.
With this background my former colleague Dr. Arunangshu Chakraborty
along with Dr Ashokka Balakrishnan has written this excellent book on point-­
of-­care ultrasound. The book is meant for the critical care physician, the
emergency physician and the anaesthesiologist. These physicians will find it
to be comprehensive, yet handy. The book addresses all aspects of critical
care ultrasonography and transthoracic echocardiography, including its use in
the diagnostics and the common bedside procedures. The authors also pro-
vide excellent illustrations to explain the salient points. Each chapter ends
with a set of self-assessment questions. It also comes with some high-quality
videos as an additional feature. This book will teach the beginner and stay
with the regular user of point-of-care ultrasonography as a quick reference. I
foresee a wide acceptance of this book among the critical care physicians.

Department of Critical Care Susruta Bandyopadhyay


AMRI Hospitals
Kolkata, West Bengal, India

vii
Preface

Point-of-care ultrasound (POCUS) is defined as a goal-directed, bedside


ultrasound examination performed by a healthcare provider to answer a spe-
cific diagnostic question or to guide the performance of an invasive proce-
dure. The field of POCUS is vast and the scope is infinite, so much so that it
is difficult to cover all of them in a single volume. What we wanted was to
present a clear and concise description of the most common day-to-day uses
of point-of-care ultrasound (POCUS) in a pocket-sized manual for the resi-
dents and trainees of the multifaceted disciplines of anaesthesia, intensive
care and emergency medicine. This book is more focused on the practical
applications than theoretical discussions. Although ultrasound-guided
regional anaesthesia (USRA) has been included in some of the POCUS text-
books, we have not included it in this book as we believe that USRA is a
larger topic and deserves its own textbook. There are many indeed, including
our primer Blockmate—A Practical Guide to Ultrasound Guided Regional
Anaesthesia by Springer Nature.
Currently, there is a shortage of textbooks in the field, and few cover all the
necessary areas that are needed for the learners in the aforementioned disci-
plines. Each chapter in this book has been written by authors who are active
practitioners of POCUS in their respective fields, and therefore tackles the prac-
tical issues comprehensively and they also host training sessions and work-
shops teaching these principles. The chapters contain multiple choice questions
for the learners to hone their knowledge and prepare for the examinations.
A special feature of this book is the wonderfully curated video library, cre-
ated by Dr. Amit Dikshit and Dr. Sudhakar Subramani. Readers of A Practical
Guide to Point of Care Ultrasound will have access to these videos through
the book. These step-by-step approaches demonstrated in the videos will
enable the reader, who may be even a novice to the field, to be able to start
practising POCUS on his own.
POCUS is going to be one of the most sought-after areas in medicine in
the coming decades. This book will be useful for the learners in anaesthesia,
critical care, emergency medicine and trauma as well as the clinical special-
ists who did not have POCUS in their curriculum but want to pick up founda-
tions in knowledge, contemporaneous technical skills, to keep abreast with
the modern-day nuances of medical practice.

Kolkata, West Bengal, India Arunangshu Chakraborty


Singapore, Singapore  Balakrishnan Ashokka

ix
Contents

1 Basics
 of Point-of-Care Ultrasound������������������������������������������������   1
Arunangshu Chakraborty and Balakrishnan Ashokka
2 Ultrasound-Guided Vascular Cannulation������������������������������������  19
Arunangshu Chakraborty, Rakhi Khemka,
Sudhakar Subramani, and Li Jia Fan
3 Point
 of Care Ultrasound of the Airway����������������������������������������  55
Deborah Khoo
4 Point-of-Care
 Ultrasound of the Lungs ����������������������������������������  81
Archit Sharma and Sudhakar Subramani
5 Point-of-Care
 Ultrasound of the Heart: Transthoracic
Echocardiogram������������������������������������������������������������������������������ 103
Rohit Vijay Agrawal, Sudipta Mukherjee,
Chang Chuan Melvin Lee, Arunangshu Chakraborty,
and Manojit Lodha
6 Vascular Ultrasound������������������������������������������������������������������������ 121
Sudhakar Subramani, Satoshi Hanada,
and Arunangshu Chakraborty
7 Focused
 Assessment with Sonography in Trauma
(FAST) Exam������������������������������������������������������������������������������������ 149
Vishakha Prasad Erasu and Priyadarshini Marathe
8 Miscellaneous
 POCUS: Gastric Ultrasound, Urinary
Bladder Ultrasound, Ocular Ultrasound,
Obstetric POCUS ���������������������������������������������������������������������������� 171
Chetan Mehra and Amit Dikshit
9 POCUS:
 What does the Future Hold? ������������������������������������������ 189
Rachel Hui Xuan Chia and Balakrishnan Ashokka
Answer Keys ������������������������������������������������������������������������������������������ 199

xi
Basics of Point-of-Care Ultrasound
1
Arunangshu Chakraborty
and Balakrishnan Ashokka

1.1 Basics of Ultrasound: Physics introduction in the early 1960s [1]. It has
and Physiology evolved rapidly through scientific discoveries
and advancement in computing. When ultra-
Sound waves are waves of compression and rar- sound was used for the first time in regional
efaction in a medium such as air. For propagation anesthesia in the 1990s, the ultrasound output
of sound, the most important factors are its fre- was a chart of dots. Now it provides a real-time
quency, wavelength, and the qualities of the image which is easily relatable to the anatomy.
medium it travels through. Ultrasound is safer compared to ionizing radia-
Only a part of the sound waves present in tion and it is portable. The side effects of clini-
nature is audible to human ears, which is known cal ultrasound are negligible. The use of
as the hearing range. Human hearing range is ultrasound by anesthesiologists for the purpose
between 20 and20,000  Hz, although individual of interventions such as vascular cannulations
capabilities may vary. Any sound which has a fre- and regional anesthesia has made those tech-
quency lower than 20 Hz is not audible to most niques safer and more reliable compared to
humans and known as infrasound. On the other landmark-based techniques [2, 3].
hand, sound of frequency greater than 20,000 Hz
is also inaudible to human ears and known as
ultrasound. In the animal kingdom, animals such 1.1.1 Mechanism of Action
as elephants can generate and hear the infrasound
that allows them to communicate over a long dis- Ultrasound is created by the piezoelectric effect
tance, whereas bats and dolphins can generate and (PE) converting electrical energy into mechanical
receive ultrasound which endows them survival vibrations (Fig. 1.1). The word piezo is derived
edge in navigation and spatial orientation. from the Greek word “piezein,” meaning “to
Ultrasound has steadily gained importance press.” It was discovered by Pierre Curie in
and popularity in medical imaging since its 1880 in quartz crystals.
When a varying voltage is applied, the PE
A. Chakraborty (*) material starts to vibrate, with the frequency of
Department of Anaesthesia, Critical Care and Pain, the voltage determining the frequency of the
Tata Medical Center, Newtown, Kolkata, sound waves produced. When placed in contact
West Bengal, India
with skin via an “acoustic coupling” jelly, the
B. Ashokka “transducer” (commonly called “probe”) trans-
Centre for Medical Education (CenMED), National
University Health System, Singapore, Singapore
mits and receives the ultrasound beam.

© The Author(s), under exclusive license to Springer Nature Singapore Pte Ltd. 2022 1
A. Chakraborty, B. Ashokka (eds.), A Practical Guide to Point of Care Ultrasound (POCUS),
https://doi.org/10.1007/978-981-16-7687-1_1
2 A. Chakraborty and B. Ashokka

Piezoelectric crystal Electric current

Application of an
Ultrasound electric current to the crystal causes it
machine to vibrate and thus generate ultrasound
waves

Reflected sound waves hit the crystals,


Ultrasound causing it to vibrate and generate
machine electric current that is analyzed by the
ultrasound machine.

Fig. 1.1  Ultrasound created by piezoelectric effect converting electrical energy into mechanical vibrations

The pulse wave that is generated at the trans-


ducer is transmitted into the body of the subject, Key Concepts [4–6]
reflected off the tissue interface, and returned to 1. Acoustic velocity (c) is the speed at
the transducer again. These returning ultrasound which sound waves travel through a
waves cause the PE elements to vibrate within the medium. It is directly proportional to
transducer which causes a voltage to be gener- the density and stiffness of the
ated. Thus, the same crystals are used to send and medium
receive ultrasound waves. An image is created •  The velocity is fastest in solids and
out of the returning signal. slowest in air.
With advancements in digital signal process- The average speed of propagation of
ing and software tools, the ultrasound image has ultrasound in body tissue is about
evolved from a grayscale chart to a 3D image 1540 m/s.
over the last four decades.
1  Basics of Point-of-Care Ultrasound 3

• Lateral Resolution: It is the ability to distin-


2. Acoustic Impedance is the product of guish between structures lying perpendicular
the sound velocity and tissue density. to beam axis, i.e., structures lying side by side.
The difference in acoustic impedance It is affected by the beam width. For example,
between two tissues influences the when the ultrasound scan is done for an axil-
amplitude of the returning echo. lary block, the ultrasound beam has to clearly
3. Resolution is the ability to distinguish distinguish between the axillary artery, axil-
between two structures that are posi- lary veins, the median, radial, and ulnar nerves
tioned close to each other. as well as the muscle layer and fasciae
Resolution depends on the fre- (Fig. 1.3).
quency of ultrasound. Wavelength of
the ultrasound beam is inversely pro- Temporal Resolution: The word temporal is
portional to the frequency. Smaller the derived from the Latin root “tempus” which
wavelength, the better is the resolution. means time. Temporal resolution is the ability
Thus, higher frequency gives better to precisely locate moving structures at given
resolution. time instants. This has an important role in car-
Resolution can be classified as spa- diological imaging. It depends on the process-
tial and temporal. ing speed and refresh rate of the ultrasound
machine.

Spatial Resolution: It is the ability of ultra- 1.1.1.1 Interactions of Ultrasound


sound to distinguish between two objects lying with Tissue
side by side. It can be of two types, axial and The ultrasound wave is subjected to a number of
lateral. interactions as it travels through a medium. They
are (Fig. 1.4)
• Axial Resolution: This is the ability to sepa-
rately discern two structures lying along the • Reflection.
ultrasound beam axis as separate and distinct. • Transmission.
Affected by the frequency of the beam • Attenuation.
(Fig.  1.2). For example, when an abdominal • Scattering.
wall is imaged, the ultrasound beam traverses
the skin, subcutaneous tissue, and the abdomi-
nal wall muscles with their fasciae, perito- 1.1.2 Reflection
neum, and abdominal contents depending on
the depth setting. A good resolution allows us Reflection is the phenomenon in which a part of
to distinguish between each of these structures the energy is sent back to the medium from which
separately. the energy originates (Fig. 1.5).

Fig. 1.2  Axial resolution


4 A. Chakraborty and B. Ashokka

Fig. 1.3  Lateral resolution

Attenuation Absorption Reflection Scattering Refraction

Skin

Fig. 1.4  Interaction of ultrasound with tissue

Like all electromagnetic waves, sound waves media impedances which results in creation of
also exhibit the phenomenon of reflection. The strong echoes.
amount of reflection from a surface depends on Ultrasound is almost totally reflected at the
the angle of incidence and the difference in interface between tissue/liquid and air, producing
impedances between two media. the brightest echo. E.g., the echo produced by
There is an absence of echo/reflection if there pleura in a normal lung.
is no difference in media impedances. However, Refraction is the change of direction of sound
in an interface between lung or bone and soft tis- while crossing the interface between two media.
sues, there is a significant difference between the The radiological significance of this phenomenon
1  Basics of Point-of-Care Ultrasound 5

Z = Density x Velocity

Transducer

Fig. 1.5 Reflection

Transducer

Fig. 1.6  Transmission of waves

is the creation of artifacts such as those seen the sound energy. Lost energy is absorbed by the
under larger vessels on USG. medium producing heat. The loss of energy, and
thus attenuation is directly related to the fre-
quency of the ultrasound beam.
1.1.3 Transmission Thus, greater the frequency, more the atten-
uation, and lesser is the penetration of the ultra-
Not all waves are reflected when passing through sound wave.
dissimilar media, some are transmitted Attenuation coefficient is a measure of atten-
(Fig. 1.6). The transmitted waves generally pro- uation caused by each tissue as a function of the
duce weaker echo therefore with increasing ultrasound wave frequency. The practical aspect
depth the amplitude and resolution of ultrasound of this, for example, is that tissues such as bones
weaken. have a high attenuation coefficient which greatly
limits the transmission of the ultrasound beam.
Also, this means penetration shall decrease with
1.1.4 Attenuation increasing frequency.
Scattering: This is the redirection of sound
The amplitude of the sound waves decreases with waves in different directions caused due to inter-
increasing depth of penetration in the body. This action with a rough surface or small reflector
is known as attenuation. It happens due to loss of (Fig. 1.7).
6 A. Chakraborty and B. Ashokka

Fig. 1.7  Scattering of sound waves

Echogenic Hyperechoic Anechoic

Fig. 1.8  Basis of echogenicity

1.1.5 Echogenicity produces minimal or no echo is called anechoic,


e.g., liquid filled cavity such as blood vessels and
The ultrasound waves reflected by tissues return pleural effusion.
to the transducer to produce image. This phe-
nomenon is similar to echoes that we hear in an 1.1.5.1 Modes of Imaging
empty hall. The property of tissues to generate Although medical ultrasound began with
echo is called echogenicity (Figs. 1.8 and 1.9). A-mode, gradually more and more complex
The tissue which produces a similar echo to its modes have been added. The mode most com-
surrounding tissue is called isoechoic, the tissue monly used in regional anesthesia practice is
that causes lesser echo hypoechoic, e.g., muscles; B-mode, which is also known as 2D ultrasound.
the tissue that causes more echo is called hyper- Table  1.1 summarizes different modes of ultra-
echoic, e.g., fascia, bones, pleura; the tissue that sound in medical imaging.
1  Basics of Point-of-Care Ultrasound 7

Hypoechoic Isoechoic Hyperechoic


Anechoic
Less echogenic than Same echogenicity More echogenic than
Absence of Echoes
surrounding tissue as surrounding tissue surrounding tissue

Fig. 1.9  Basis of echogenicity—anechoic, hypoechoic, isoechoic, and hyperechoic tissues

Table 1.1  Modes of imaging 1.1.5.2 Transducers


A-Mode This is the most basic mode and works by The ultrasound transducer, commonly called
displaying the reflected sound pulses on a “probe” is the part held by the operator’s hand that
time axis from a single line scan. This
comes in contact with the patient. It is a vital part
mode is not of much use in clinical
anesthesia of the machine as it contains the PE crystals that
B-Mode This is a 2D version of the A-mode and emit and receive ultrasound. Transducers come in
the most popularly used mode in various sizes and shapes. The shape of a probe
anesthesia. On passing through a slice of governs the field of vision while the frequency of
tissue, the reflected ultrasound beam is
displayed which depicts the anatomical
sound waves emitted governs the image resolu-
cross section tion and depth of penetration (Fig. 1.10).
M-Mode This mode can detect the movement of The linear probe emits a linear array of ultra-
reflecting media along a single line scan. sound and produces a rectangular image. It is
Often used along with the B-mode, this
usually of high frequency and low penetration.
mode is popularly used in cardiological
imaging to visualize heart valve The higher frequency gives the linear probe bet-
movement ter image resolution and is favored for superficial
Doppler This mode is based on the doppler effect interventions where high accuracy is required.
Mode which is described by a change in the The curvilinear probe emits a curved array
frequency of sound due to the relative
motion between the source and receiver of
of ultrasound beams and produces a curved
sound. In this mode, the reflected sound image. It has a lower frequency but a wider area
waves, when ultrasonic waves are beamed of imaging and greater depth. Due to low fre-
along an artery or vein, have a doppler quency, the image resolution is grainy and infe-
change in frequency due to the motion of
blood
rior compared to a linear probe. However, for
Colour This provides a color coded image of deeper blocks, imaging and interventions it is the
Doppler doppler shifts. The direction of blood flow transducer of choice.
depends on the direction of motion
towards or away from the transducer. By 1.1.5.3 Time Gain Compensation
convention, red and blue colors are
selected for the identification of direction This is an operator-controlled amplification tech-
and velocity of blood nique to make up for the sound attenuation as
Power Nearly five times more sensitive than ultrasound waves travel through tissue. It must be
Doppler color doppler and is used to scan smaller manually adjusted for each tissue type to be
vessels more accurately. This mode
however does not provide any information
scanned and manipulated for best image
on the speed and direction of flow optimization.
8 A. Chakraborty and B. Ashokka

Probe Probe

Beam
Beam

Fig. 1.10 (a) Linear and (b) Curvilinear transducers and their ultrasound beam pattern

The TGC control layout differs from one • Depth adjustment.


machine to another. The presence of slider knobs • Gain.
is a popular design; each knob in the slider set con- • Focus.
trols the gain for a specific depth, which gives a • Compound imaging use (Fig. 1.12).
well-balanced image (Fig. 1.11). Accordingly, the
sliders are called near field TGC and far field TGC. Frequency: Higher frequency is usually
selected for superficial interventions that require
1.1.5.4 Practical Aspects greater resolution. With decreasing frequency,
The ultimate objective of learning the basics of tissues at greater depth can be imaged, at the cost
ultrasound imaging is to be able to obtain an opti- of resolution (Fig. 1.13).
mal image. For image optimization, the follow- Depth: Depth of the image has to be adjusted
ing points should be kept in mind [7–9]: to the depth of the object to be blocked or the depth
of the intervention endpoint. The depth selected
• Frequency of the transducer. should be at least a few centimeters more than the
1  Basics of Point-of-Care Ultrasound 9

depth of the target. That way, if the needle over-


shoots the target it can be seen. Also, the nearby
anatomical structures would remain visible.
Decreasing the depth increases magnification
and vice versa. For superficial blocks, by decreas-
ing the depth setting, greater details can be
appreciated.
Gain: The gain function is used to increase
overall screen brightness. An optimum gain
should be used to obtain the best possible con-
trast between the muscles and the connective tis-
sue (fascia) for a nerve block because usually the
nerves produce echo that is similar to the connec-
tive tissue. While TGC controls are used to mod-
ify the gain at different depths, overall gain can
be adjusted using the gain button (Fig. 1.14).
Fig. 1.11  The TGC layout in a standard USG machine

Practical guide to choosing optimal


transducer frequency:

Optimal setup Frequency 10-13 MHz:


Interscalene, supraclavicular, axillary,
forearm, wrist, femoral, ankle and TAP
blocks

Frequency 6-10 MHz:


Infraclavicular, popliteal, subgluteal
sciatic nerve blocks

Activate Frequency 2-5 MHz:


compound Gluteal sciatic nerve, lumbar plexus
imaging and celiac ganglion blocks

5 important
functions
Depth to optimize Frequency
the image

Focusing Doppler
Gain

Fig. 1.12  Algorithm for optimum imaging with ultrasound


10 A. Chakraborty and B. Ashokka

Fig. 1.13  Frequency and transducers

Fig. 1.14  Gain adjustment: Ultrasound scan of the axillary area with gain adjustment: (a) low gain, (b) high gain, (c)
optimal gain
1  Basics of Point-of-Care Ultrasound 11

Fig. 1.15  US console


showing frequently used
functions

Focus: Focus of the ultrasound image is the Pressure: By putting mild pressure with the
narrowest point of the ultrasound beam. It is the probe, subcutaneous fat can be displaced and a
point where the image resolution is best. better image can be obtained.
Modern ultrasound machines have electronic Alignment: Proper alignment of the probe in
focus adjustment capacity. It is best to place the the recommended plane is required to obtain a
focus just at the level of or slightly below the clinically useful image. E.g., The US probe is
object to be viewed for optimum image held transversely for a TAP block, but sagittally
quality. for a SAP block.
The ultrasound machine console (Fig.  1.15) Rotation: The probe may need to be slightly
contains the buttons and sliders for modifying all rotated to align it perfectly with the underlying
the above factors such as frequency, depth, and tissue and the block needle. E.g., For the supra-
focus. One must be well versed with the console clavicular brachial plexus block, the probe needs
to obtain the best image and to be able to store to be slightly externally rotated.
and retrieve images. Tilt: Slight cephalad, caudal, lateral, medial,
or oblique tilt may be necessary for the optimal
Compound Imaging image. Because of the direction of the nerve
It is the technology that combines multiple fibers, some nerves seem different at different
coplanar images (spatial compounding) with angles and a little tilt is required to obtain the per-
images obtained from multiple frequency pendicular cross section and hence the best
spectra (frequency compounding) to form a image. This property of tissues is called anisot-
single image. This decreases artifacts and ropy. E.g., Caudal tilt is required for imaging the
speckles and improves resolution [10] supraclavicular brachial plexus.
(Fig. 1.16).
Needling Techniques
Maneuvering the US Probe: PART The Ultrasound beam travels in a straight line,
Even after optimizing the image setting, an opti- like a thin sheet, perpendicular to the probe. How
mal image may not be obtained. For that, the the intervening needle is visualized depends on
ultrasound probe needs to be held and maneu- how much of its length is in alignment with the
vered (Fig. 1.17). ultrasound beam [11, 12].
12 A. Chakraborty and B. Ashokka

Fig. 1.16 Compound image combines multiple coplanar images with images obtained from multiple frequency spectra
to form a single image

P • Pressure

A • Alignment

R • Rotation

T • Tilt

Fig. 1.17 PART maneuver to obtain optimal image

In-Plane
The needle is placed parallel to the transducer.
The needle shaft and tip are both visible
(Figs. 1.18 and 1.19).
Example: axillary block.
This approach is safe and easy to learn but is
difficult to practice in deeper blocks. The quality Fig. 1.18 In-plane needling technique
1  Basics of Point-of-Care Ultrasound 13

Fig. 1.19  In-plane needling technique. Note that the entire length of the needle and its tip is imaged in real time

Needle tip

IJV
CCA

Fig. 1.20  Out of plane cannulation of internal jugular ultrasound image on the right shows the sonoanatomy and
vein (IJV): Left panel shows the probe holding and needle the tip of the needle being seen as a bright hyperechoic
entering out of plane at a right angle to the probe. The spot. CCA common carotid artery lies medial to the IJV

of needle visualization depends on the angle of tant to observe tissue displacement by the
entry of the needle. The flatter the angle (acute advancing needle tip as often the needle tip may
angle), the better is the image. To reach deeper not be seen.
objects, a more perpendicular angle is required, For superficial procedures such as internal
which makes needle visualization in this tech- jugular vein cannulation, it is a good strategy to
nique difficult. estimate the depth of the target and calculate the
entry point of the needle, angle of inclination,
1.1.5.5 Out of Plane and angle of the ultrasound probe accordingly.
The needle is placed perpendicular to the trans- For deeper structures, hydrodissection with
ducer probe. The tip of the needle may be diffi- 1–2 ml saline can be used to confirm the location
cult to locate accurately in this approach and use of the needle tip.
of echogenic tip needles is advised. It is impor- Example: Vascular cannulations (Fig. 1.20).
14 A. Chakraborty and B. Ashokka

1.1.5.6 Bioeffect and Safety facts such as A lines and B lines are pathogno-
Ultrasound application produces biologic effects monic and useful for the detection of clinical
by thermal and nonthermal mechanisms. Heat is conditions.
generated through absorption of ultrasound by These artifacts are described in detail in text-
tissues and is directly proportional to ultrasound books of point-of-care ultrasound.
intensity, frequency, and duration.
Two standard indices are displayed by an
ultrasound machine (Fig. 1.21). 1.2 Point-of-Care Ultrasound:
Thermal index (TI): It is the transducer Concept and Limitations
acoustic power divided by the estimated power
needed to increase tissue temperature by 1°. Like the five elements of nature, the human body
Mechanical index (MI): It is the peak rar- is also composed of solid tissue (earth), liquid tis-
efactional pressure divided by the square root of sue (water), and gas-filled spaces (air). While
the center frequency of the pulse bandwidth. The imaging with ultrasound, we refer to tissue in
relative likelihood of thermal and mechanical terms of “echogenicity” or “echo signature.”
hazard is indicated by TI and MI, respectively. TI Water offers least resistance to the passage of
or MI >1.0 is dangerous. ultrasound thereby providing the least echo-
Different tissue and examination settings are genicity which is called anechoic. Of all parts in
saved and categorized in modern ultrasound the body, water containing tissues such as blood,
machines to aid in imaging as well as reducing pleural fluid, and amniotic fluid have the least
these bioeffects. For example, the ophthalmic echogenicity, the echogenicity being directly pro-
mode is a low power setting that allows examina- portional to the density of the fluid and it increases
tion of the eyes without causing any tissue injury if there are suspended particles. Thereby, an exu-
to the sensitive ophthalmic tissues such as the dative pleural fluid will have more echogenicity
retina. than a serous fluid. The sludge in the gallbladder
Artifacts: Artifacts are false images produced can be distinguished from normal bile because of
due to highly reflective tissue interfaces. There the suspended particles, which reflect ultrasound
are various types of artifacts, such as comet tail and produce a higher echogenicity.
artifacts and mirror image artifacts. Solid tissues reflect ultrasound based on their
Some of the artifacts are misleading, water content. Thereby, muscles are less echo-
whereas some are useful. Lung ultrasound arti- genic than the adipose tissue and the fibrous sep-
tae that separate them. The nerves being solid,
with lesser water content than muscle can be
MECHANISM identified too, but their echogenicity changes
from their origin to the end. Nerves at their origin
as nerve roots tend to be less echogenic and
appear as hollow circles or “bubbles” in the short
axis (SAX) view, but as the roots unite to form
divisions and the divisions unite to form nerves,
Thermal Non-thermal they gather more and more fatty layers of peri-
neural tissue and the nerves appear as “honey-
comb” in the SAX view.
Bones are the most dense tissue in the body
and they do not allow ultrasound to pass.
Mechanical Cavitation Therefore, the bones appear hyperechoic due to
the reflection of ultrasound by the shiny bony
Fig. 1.21  Bioeffect and safety surface and we see an “acoustic shadow” beneath
1  Basics of Point-of-Care Ultrasound 15

them, like the shadow cast by a solid object under ultrasound almost completely, producing the
the sun. highest hyperechoic image that appears shiny
Air also does not allow the passage of ultra- and glistening.
sound, but the tissue-air interface reflects the

View of ribs and pleura: Note the acoustic shadow cast by Honeycomb appearance of peripheral nerves, note the
the ribs, which are hyperechoic anechoic nature of the blood vessels and the injected
local anesthetic (LA) in contrast to the nerve and the
surrounding tissue

The purpose of Point-­ of-­


care ultrasound
(POCUS) is not to train the anesthetist, the inten- –– Gain settings.
sivist, or the emergency physician as a radiologist. –– Depth settings.
POCUS is defined as a goal-­ directed, bedside –– Tissue echo characteristics.
ultrasound examination performed by a health –– Ultrasound artifacts.
care provider to answer a specific diagnostic • For optimal imaging, it is important to
question or to guide the performance of an inva- adjust the pressure, alignment, rotation,
sive procedure. With the specific question in and tilt (PART) of the probe.
mind, the examination becomes brief and it does • The best needle image is obtained when
not have to bear the level of precision expected the needle forms an acute angle with the
from a radiologist. With the training on POCUS, ultrasound probe as more US waves
the physician should be able to identify the differ- reflect off the needle. The steeper the
ent tissues and carry out the specific objectives. angle, the lesser the needle visibility.
The best image is produced by a needle
that is parallel to the ultrasound probe,
i.e., perpendicular to the ultrasound
Key Concepts beam.
• Knowledge of ultrasound physics is • For deeper tissues, it is useful to enter
needed for image optimization. The key the needle 2–3 cm away from the edge
understandings are of the ultrasound probe so that a reason-
–– Appropriate transducer selection. ably acute angle can be formed.
16 A. Chakraborty and B. Ashokka

Self Assessment Questions C. Frequency and depth of penetration are


directly proportional
1. In-plane approach uses D. Image quality does not change with
A. Long axis view increasing depth of the tissue
B. Short axis view 9. Which of the following statements is NOT
C. Both views can be used correct?
D. None of the above A. Axial Resolution is the ability to sepa-
2. Echogenicity of blood is known as rately discern two structures lying along
A. Hypoechoic the ultrasound beam axis as separate and
B. Hyperechoic distinct
C. Anechoic B. Lateral Resolution is the ability to distin-
D. Both A and C guish between structures lying perpen-
3. Which ultrasound transducer provides better dicular to the ultrasound beam axis
resolution? C. Temporal resolution is the ability to pre-
A. High-frequency linear array cisely locate moving structures at given
B. Low-frequency curved array time instants
C. Phased array doppler D. Resolution is inversely proportional to
D. All of the above the frequency of ultrasound
4. Which of the following tissues appear bright- 10. Artifacts are created due to
est in an ultrasound image? A. Reflection
A. Tip of the bones such as spinous process B. Refraction
of vertebrae C. Transmission
B. Bony surface such as pelvic bones D. Attenuation
C. Pleura with an inflated lung underneath
D. Peripheral nerves
5. Gain is increased to References
A. Increase contrast of the image
B. Increase resolution of the image 1. Marhofer P, Chan VW.  Ultrasound-guided regional
C. Freeze the image anesthesia: current concepts and future trends. Anesth
Analg. 2007;104:1265–9.
D. Increase overall brightness of the image 2. Neal JM, Brull R, Chan VW, Grant SA, Horn JL, Liu
6. Time gain control (TGC) allows to SS, et al. The ASRA evidence-based medicine assess-
A. Increase or reduce the speed of ultra- ment of ultrasound-guided regional anesthesia and
sound scanning pain medicine. Executive summary. Reg Anesth Pain
Med. 2010;35:S1–9.
B. Increase or reduce the brightness of a 3. Cory PC.  Concerns regarding ultrasound-­
particular depth of the scanning area guided regional anesthesia. Anesthesiology.
C. Increase contrast of the image 2009;111:1167–8.
D. Increase gain of the overall image 4. Brull R, Macfarlane AJ, Tse CC.  Practical knobol-
ogy for ultrasound-guided regional anesthesia. Reg
7. Focus of the ultrasound beam should be set Anesth Pain Med. 2010;35:S68–73.
at 5. Merritt CR. Physics of ultrasound. In: Rumack CM,
A. At the middle of the image Wilson SR, Charboneau JA, editors. Diagnostic
B. At the bottom of the image ultrasound. 3rd ed. St. Louis, MO: Elsevier Mosby;
2005.
C. At or just below the target tissue 6. Sites BD, Brull R, Chan VW, Spence BC, Gallagher J,
D. Anywhere, it does not matter Beach ML, et al. Artifacts and pitfall errors associated
8. Which of the following is true? with ultrasound-guided regional anesthesia. Part II: a
A. Lower frequency transducers produce pictorial approach to understanding and avoidance.
Reg Anesth Pain Med. 2007;32:419–33.
better resolution 7. Bigeleisen PE, editor. Ultrasound-guided regional
B. Higher frequency transducers have lesser anesthesia and pain medicine. London: Lippincott
depth of penetration Williams and Wilkins; 2010.
1  Basics of Point-of-Care Ultrasound 17

8. Pollard BA, Chan VW.  Introductory curriculum for 11. Maecken T, Zenz M, Grau T.  Ultrasound character-
ultrasound-guided regional anesthesia. Toronto, ON: istics of needles for regional anesthesia. Reg Anesth
University of Toronto Press Inc.; 2009. Pain Med. 2007;32:440–7.
9. Tsui BC. Atlas of ultrasound and nerve stimulation-­ 12. Pollard BA.  New model for learning ultrasound-­
guided regional anesthesia. New  York: Springer; guided needle to target localization. Reg Anesth Pain
2007. Med. 2008;33:360–2.
10. Brian DS, Macfarlane AJ, Sites VR, Chan VW, Brull
R, et al. Clinical sonopathology for the regional anes-
thesiologist. Reg Anesth Pain Med. 2010;35:272–89.
Ultrasound-Guided Vascular
Cannulation 2
Arunangshu Chakraborty, Rakhi Khemka,
Sudhakar Subramani, and Li Jia Fan

2.1 Introduction (Category A2-B evidence). It is considered the


Gold standard now to cannulate the internal jugu-
Central venous cannulation was one of the earliest lar vein with ultrasound guidance.
clinical applications of point-of-care ultrasound.
It has been widely accepted [1] that using ultra-
sound for central venous cannulation reduces the 2.2 Principle of Vascular
number of attempts to cannulate and increases Cannulation
safety by reducing complications such as com-
mon carotid artery puncture, trauma to pleura, and 2.2.1 Identification of the Blood
the brachial plexus. Meta-analyses of RCTs com- Vessel
paring real-time ultrasound-guided venipuncture
of the internal jugular with an anatomical land- The blood vessels are clearly distinguishable
mark approach report higher first insertion attempt from the surrounding tissue due to their hyper-
success rates [1–14], higher overall success rates echoic wall and hypoechoic content (blood). The
[1–25], lower rates of arterial puncture [2–15, 18, posterior wall of the blood vessel and the tissue
20], and fewer insertion attempts (Category A1-B directly underneath often display a typical
evidence) [1–25]. RCTs also indicate reduced enhancement artifact. The smaller blood vessels
access time or times to cannulation with ultra- such as the radial artery when thrombosed may
sound compared with a landmark approach sometimes look similar to a tendon or a nerve
under ultrasound. It is useful to scan both up and
A. Chakraborty (*) downstream to identify the blood vessel by iden-
Department of Anaesthesia, Critical Care and Pain, tification of the continuity of the image pattern.
Tata Medical Center, Newtown, Kolkata,
West Bengal, India The hallmark of a vein is its collapsibility
under pressure of the ultrasound transducer
R. Khemka
Tata Medical Center, Kolkata, West Bengal, India (UST). While it is useful in identifying the veins,
it must be kept in mind that-
S. Subramani
Department of Anesthesia, University of Iowa,
Iowa City, IA, USA A sufficient pressure will obliterate even the
L. J. Fan arteries.
Division of Critical Care, Department of Paediatrics, While performing the cannulation the pressure
Khoo Teck Puat—National University Children’s from the UST must be carefully kept to the
Medical Institute, National University Hospital, minimum to facilitate the procedure.
Singapore, Singapore

© The Author(s), under exclusive license to Springer Nature Singapore Pte Ltd. 2022 19
A. Chakraborty, B. Ashokka (eds.), A Practical Guide to Point of Care Ultrasound (POCUS),
https://doi.org/10.1007/978-981-16-7687-1_2
20 A. Chakraborty et al.

a b c

Fig. 2.1  Blood vessels: (a) Radial artery, (b) Cubital vein, (c) Internal jugular vein and common carotid artery

One should also remember some practical but a


commonly overlooked aspects such as position of
the blood pressure (BP) cuff and its state of infla-
tion at the time of an arterial access. Often after probe
induction of anesthesia when the BP is low, or in distance 45˚

general in a hypotensive patient, arterial cannula-


tion is a challenge. In such patients, it is impor-

depth
tant to remember to lighten the pressure of the
UST while cannulating the artery (Fig. 2.1). distance = depth

b
2.2.2 Depth, Distance, and Angle

Unlike nerve blocks and other ultrasound-guided


procedures, vascular cannulations are mostly per-
probe
formed using an out-of-plane technique. The distance -60˚ 45˚
choice of whether to use an in-plane or an out-of-­
plane technique depends on the diameter of the
depth

vessel and its depth. While a larger and superficial


vessel can be cannulated in an in-plane technique,
distance = ½ of depth
a smaller or deeper target generally necessitates
the use of an out-of-plane technique.
For an out-of-plane approach to a blood vessel it c
is to be remembered that the needle tip is visible
only where it crosses the plane of the ultrasound
beam, making the window for error quite small. It is
probe
useful to mentally construct a right-­angled triangle,
45˚ -30˚
with the plane of the ultrasound beam forming the distance
perpendicular, distance from the midline of the
probe to the point of entry to the base and the needle
depth

trajectory of the hypotenuse while holding the ultra-


sound transducer (UST) vertically and in a trans- distance = 2x depth

verse orientation to the target vessel. For deeper and


bigger targets the ideal angle of entry should be
between 45 and 60° (Fig. 2.2). The distance of the Fig. 2.2  Relationship between the depth of the target,
distance of the point of entry from the transducer foot-
point of entry from the UST must be measured print, and the angle of the needle: (a) when the depth and
accordingly to derive the angle of entry. For deeper the distance are equal, the angle is exactly 45°. (b) The
and larger vessels such as the femoral vein, a steep angle increases when the distance reduces and (c) the
needle angle is usually required to retain the direc- angle decreases as the distance increases
2  Ultrasound-Guided Vascular Cannulation 21

tion. Even superficial but larger targets can be can- • Decide the type and bore of cannula/catheter.
nulated with a needle angle of 45° or more. For • Review relevant past medical history.
superficial and smaller targets a shallower approach
is usually required for cannulation as a steeper angle 2.2.4.2 Positioning/Room Set Up/
may lead the needle to go through the posterior wall Ergonomics
easily causing difficulty in cannulation, hematoma,
and requiring multiple attempts. Superficial vessels Patient
such as radial artery cannulation usually require an • Comfortably lying or sitting.
angle of 30° or less. The distance of the point of • Supine position with Trendelenburg for cen-
entry of the needle needs to be calculated accord- tral venous cannulation of upper extremity.
ingly. Moving the probe up and down and tilting the • Arm well supported; arm board (if available)
probe would help visualizing the needle. with the limb abducted and externally rotated.
• Adequate exposure to clean field.

2.2.3 Selecting the Transducer Operator


and Mode • Ergonomically positioned: Sitting or
standing.
While imaging for vascular cannulations, it is • Align ultrasound monitor, patient and patient’s
important to rationalize the appropriate trans- access point within the operator’s line of sight.
ducer and mode. • Equipment close, reachable, and in order to be
used.
2.2.3.1 Transducer
A high-frequency (10–20 mHz) linear transducer is Equipment
usually selected for cannulation of vessels up to the • Ultrasound machine.
depth of 5 cm. Cannulation of deeper vessels such –– Linear transducer (7.5–10  MHz):
as the femoral vessels in an obese individual may Superficial structures.
require the curvilinear UST which has a lower fre- –– Curvilinear transducer (2–5 MHz): Deeper
quency (7–10 mHz) but a greater depth of penetra- structures.
tion. A transducer with a smaller footprint such as • Probe cover.
the hockey stick transducer is useful for cannula- • Sterile gel.
tion in children or adults with small body parts. • Tourniquet for peripheral venous cannulation.
• Gloves.
2.2.3.2 Mode • Syringe of normal saline (flush).
2D mode is generally used for cannulation. A • Skin preparation—alcohol/antiseptic wipes.
color Doppler mode can be selected to identify • Cannula.
arteries from veins. Arteries have a pulsatile flow –– Check the required flow rate to determine
and the flow towards the transducer is usually col- the gauge required.
ored in red and flow away blue in most of the ultra- –– Standard vs longer cannula dependent on
sound machines. Color mode is particularly useful the depth of vein being accessed.
for distinguishing between deeper and closely • Adhesive transparent dressing (e.g.,
lying vessels such as the femoral artery and vein. Tegaderm).
• ± blood vials for pathology (as required).

2.2.4 Preparation
2.3 Central Venous Cannulation
2.2.4.1 Preprocedural Evaluation
• Review indications for cannulation. Although the placement of a central venous cath-
• Decide the technique—Seldinger/catheter- eter (CVC) is a routine procedure in intensive
over-needle. care medicine and in perioperative care, acute
22 A. Chakraborty et al.

severe complications such as inadvertent arterial corresponds with an orientation marker on one
puncture or cannulation, hematoma, hemothorax, side of the US scan sector shown on the US
or pneumothorax occur in a relevant proportion device screen helps to obtain the correct probe
of patients [26]. Traditionally, IJV central cannu- orientation during US examination [30]. It is
lation placement is performed using landmark important to note here that the IJV and the CCA
techniques based on the knowledge of anatomic often have considerable overlapping, starting
structures and palpation of arteries next to the from 20% to even 100% at this level. When the
veins. Unfortunately, these landmark techniques CCA is directly below the IJV, even with the
have their own limitations in detecting anatomic ultrasound guidance there is a considerable risk
variations at the CVC insertion site as well as of inadvertent CCA puncture. Therefore, it is
venous thrombosis that is especially common in prudent to scan up and down the IJV to find out
oncologic and critically ill patients can make the position where the overlapping is minimum
CVC placement impossible or dangerous for the and choose that point for the puncture. Turning
patient [27]. The use of ultrasound has been pro- the head too much to the other side sometimes
posed to increase the safety and quality of CVC increases the overlap. These factors should be
placement by reducing CVC-related complica- considered at the time of positioning. If the
tions [28]. The meta-analysis of over 5000 patient is awake, he/she can be advised to opti-
patients from 35 trials demonstrated a reduced mize the head turn to the optimal level.
total rate of complications by using US compared The distance of the point of entry of the needle
with conventional landmark techniques (4.0% vs from the footprint of the UST should be kept in
13.5%, respectively). In addition, the overall suc- mind while inserting the needle and the angle
cess rate was higher in the US (97.6% vs 87.6%). adjusted accordingly.
The benefits of US-guided CVC placement with Sometimes the needle would push the skin
regard to the total complication rate, overall suc- and subcutaneous tissue in a manner before actu-
cess rate, and number of attempts until success ally piercing that it appears that the needle has
were consistent across experienced and inexperi- gone past the vein. In such a situation the vein
enced operators [29]. appears collapsed under the pressure of the nee-
dle. As soon as the needle enters the vein the
pressure is relieved and the needle tip is seen as a
2.3.1 Internal Jugular Vein bright echogenic point inside the hypoechoic
Cannulation lumen of the IJV.
While positioning the UST, the midpoint
One of the earliest vascular cannulations that is of the UST, which often has a mark, should
performed using ultrasound guidance is the inter- be directly above the midpoint of the vein.
nal jugular vein (IJV) cannulation. The patient-­ Lateral and medial ends should be identified
awake or under anesthesia/sedation is placed before entering the skin. In case of doubt,
supine, head turned to the other side and table/ remember CCA lies medially and posterior
bed tilted to 10–15° Trendelenburg. The head (below) to the IJV.
down maneuver helps in engorging the vein and US can be used in different ways to facilitate
also prevents inadvertent venous air embolism CVC placement. Using US prior to catheter
(VAE). After antiseptic dressing and draping, the placement only to visualize the IJV to determine
ultrasound probe, covered in a sterile plastic patency and anatomical variants has been
sheet is placed on the neck at the level of the referred to as the US-assisted approach.
­cricoid cartilage and slided laterally to image the However, the US-guided approach, which is
IJV and the common carotid artery (CCA). Small, real-time visualization of needle placement and
linear array, high-frequency transducers advancement is generally recommended to pre-
(5–15 MHz) with scanning surface of 2–5 cm are vent or minimize catheter-related complications
best suited for IJV cannulation. An index mark [31]. US guidance during needle advancement
2  Ultrasound-Guided Vascular Cannulation 23

can be performed using: a short-axis (SAX) to learn for physicians not familiar with ultra-
probe orientation and an out-of-plane view of the sound [35]. The advantage of the SAX approach
needle; a long-axis (LAX) probe orientation and allows better visualization of the vein in relation
an in-plane view of the needle or occasionally an to the artery might more sufficiently help to
oblique orientation, a view that is halfway avoid accidental arterial puncture, but the needle
between the short-axis and the long-axis view is only visualized as an echogenic point which is
with the US probe placed at approximately 45° not necessarily the tip of the needle [36]. The
with respect to the target vessel [32, 33]. In addi- entire needle can be visualized in its complete
tion a new anteroposterior SAX but in-plane course with LAX in-plane approach and the
technique that combines the advantage of in- depth of the needle tip is also visualized simulta-
plane technique to track the needle tip and short- neously thereby reducing the risk of penetration
axis view of visualizing nearby anatomical of the posterior vessel wall [37] (Figs. 2.3, 2.4,
structures by placing the probe on the side of the 2.5, and 2.6).
neck, oriented anteroposteriorly, perpendicular
to the long axis of the neck have been described.
This view visualizes IJV and its relationship to 2.3.2 Subclavian Vein Cannulation
the carotid artery in the short axis [34].
With limited available data unable to predict 2.3.2.1 Anatomy of Subclavian Vessels
superiority of one technique over another one. The subclavian vein is the continuation of the
However, SAX out-of-plane approach is easier axillary vein as it courses beneath the clavicle. At

Fig. 2.3  US image of internal jugular vein in (a) short (out-of-plane view) and (b) long axis(in-plane view)
24 A. Chakraborty et al.

Fig. 2.4  Oblique approach for IJV cannulation

Fig. 2.5  Alternative in-plane technique for IJV cannulation

Fig. 2.6  Cannulation of the internal jugular vein (IJV): print to the point of entry of the needle is shorter than the
Left: Note the high-frequency linear UST being held in a depth of the target vessel. Right: the needle tip is seen as a
transverse orientation and the needle entering at an angle bright echogenic object inside the lumen of the IJV. Note
of about 60°. Note that the distance from the UST foot- the overlap of CCA and IJV to more than 30%
2  Ultrasound-Guided Vascular Cannulation 25

the lateral border of the first rib, it travels superi- such as aneurysm, stenosis, or thrombosis of the
orly then under the clavicle travels medially until vessels. Compared to US-guided cannulation of
it joins the internal jugular vein to form the IJV there are limited data on the utilization of US
innominate vein. The subclavian artery runs pos- for SV cannulation. Meta-analysis of over 2000
terior and superior to the subclavian vein. The patients from nine studies showed that the use of
brachial plexus courses superiorly and deep to US resulted in a reduced rate of accidental arte-
the medial portion of the subclavian artery. On rial puncture (0.8% vs 5.9%); and hematoma for-
the left side, the thoracic duct lies close to the mation (1.2% vs 6.6%). However, there was no
subclavian vessels. Most importantly, the lung statistically significant difference between the
and pleural cavity lie inferior and deeper to the use of US and the conventional landmark tech-
subclavian vein and are particularly vulnerable to nique with regard to the total complication rate,
accidental puncture more often on the left chest the overall success rate, the number of attempts
where the apex of the lung can extend just above until success, the time to successful cannulation,
the first rib. Familiarity with anatomical variants and the success rate with the first attempt [38,
of subclavian vessels are essential prior to any 39]. Currently there is no strong evidence of
interventional procedures or cannulation of both using the US for SV cannulation from different
vein and artery. societies (Fig. 2.7).

2.3.2.2 Ultrasound Assessment 2.3.2.3 Ultrasound-Guided Subclavian


of Subclavian Vessels Vein Cannulation Technique
Similar to IJV, 2D and color Doppler US are uti- While IJV cannulation with ultrasound is one of
lized to assess the morphology, patency, anatomi- the basic techniques of POCUS, the subclavian
cal variants, and certain pathological conditions vein (SCV) cannulation is a more advanced tech-

a b

Fig. 2.7  US image of subclavian vessels. (a) Short axis image with probe in parasagittal orientation, (b) Long axis
image, with the probe in coronal/transverse orientation
26 A. Chakraborty et al.

nique. The clavicle, which lies directly above the After instilling local anesthetic (LA) along the
SCV, precludes ultrasound imaging. SCV being a path of the needle entry, the central venous can-
deeper vein and the subclavian artery lying in nulation needle, fixed with an empty 2  cc (or
close proximity makes the cannulation even more 5 cc) syringe is inserted in-plane from lateral to
challenging. Subclavian vein (SV) can be visual- medial. (Once the needle tip can be imaged inside
ized from infraclavicular (most common) or the SCV, dark venous blood can be aspirated in
supraclavicular approach. High-frequency linear the syringe attached to the needle. The syringe is
probes are commonly used to visualize subcla- then disconnected and the guidewire is intro-
vian vessels and for morbidly obese individuals, duced. The guidewire should be visualized in the
curvilinear probe with better penetration features superior vena cava and not in the SCA or the IJV
is recommended. The UST is positioned perpen- by placing the UST on the neck and tilting the
dicular to the midline, i.e., in a coronal/transverseprobe caudally. If the guidewire is found in the
orientation, medial end of the UST lying on the neck it is to be pulled back and negotiated in
midpoint of the clavicle. A little cephalad tilt of again. Only after confirmation of the guidewire
the UST allows imaging of the SCV and location in the superior vena cava, the needle is
SCA. The artery would be imaged lying posterior withdrawn.
to the vein. Before puncture, one should be sure Proceduralist can stand either at head end or
that both the artery and the vein can be imaged on the ipsilateral side for right subclavian vein
separately. Vein can also be identified by its col- cannulation. Ergonomics need to be ensured as it
lapsibility under pressure and that it expands is a demanding procedure.
upon valsalva maneuver and by a Trendelenberg A short-axis view can be also obtained at this
position. In case of difficulty of visualization, a location by rotating the UST by 90° and an out-­
more lateral site may be chosen. of-­plane approach for cannulation can be
An infraclavicular approach is a more tradi- performed.
tional approach, although a supraclavicular In either approach, it is mandatory to identify
approach has been described. and image the subclavian artery before beginning
the process of cannulation. Currently no evidence
Infraclavicular Approach of showing one method’s superiority over other
For infraclavicular approach, either SAX out-of-­ methods however a US-guided infraclavicular
plane or long-axis in-plane techniques have been approach is feasible to visualize SCV in most of
implemented with limitations of each approach the clinical settings (Figs. 2.8 and 2.9).
like in IJV cannulation. Compared to the land-
mark method, cannulation occurs more laterally. Supraclavicular Approach
The UST is held transversely at an angle of
90° to the midline just below the clavicle in a way Ultrasound Assessment of SCV in the Fossa
that the medial side of the ultrasound image Above the Clavicle
shows the clavicle and the shadow cast by it and The SCV runs from lateral to medial under the
the lateral side shows the subclavian vein on a clavicle, just anterior to the subclavian artery
little cephalad tilt of the transducer. The image is (SCA). As it approaches the heart, the SCV is
adjusted for depth, which is typically at 6 cm for joined by the IJV, forming the brachiocephalic
thin individuals. As the transducer is slided later- (innominate) vein. The supraclavicular approach
ally and tilted further cephalad, both subclavian attempts to cannulate the portion of the SCV just
vein and artery can be imaged. The angle of lateral to the clavicular head of the sternocleido-
insonation is important to observe because the mastoid muscle. The right SCV is preferred to the
needle for an in-plane cannulation would travel left since it forms a straighter angle with the IJV,
in the same plane. offering a shorter distance for wire passage into
2  Ultrasound-Guided Vascular Cannulation 27

Fig. 2.8  Infraclavicular approach long axis

Fig. 2.9  Infraclavicular approach short axis

the superior vena cava, avoiding proximity to the Ultrasound-Guided SCV Cannulation
thoracic duct, which drains into the left Once visualize the SCV, turn the probe in the lon-
SCV.  Place a high-frequency linear transducer gitudinal plane prior to needle puncture. Enter
(6–13  MHz) on the lateral neck just above the the SCV from the lateral aspect using in-plane
clavicle to locate the IJV and carotid artery. technique. Probe needs to be tilted more anteri-
Slowly trace the IJV by moving the probe cau- orly (caudally) and stabilized against the clavicle.
dally into the supraclavicular fossa until the This approach has an advantage of placing SCV
probe abuts the clavicle. While visualizing the even during CPR.  This approach has several
most caudal aspect of the IJV, angle the probe advantages over landmark technique. SCV can
anteriorly to visualize the confluence of the IJV also be accessed by short axis out-of-plane tech-
and SCV (Fig. 2.11). At this proximal location, nique (Fig. 2.11). However, long-axis approach is
the SCV lies anterior to the SCA, and the opera- preferred to avoid pleural puncture. Ultimately
tor should dynamically fan the probe from a pos- familiarity of SCV anatomy at this location and
terior to anterior position to identify both the operator skill determines the successful
vessels. cannulation.
28 A. Chakraborty et al.

2.3.3 Femoral Vein Cannulation 4. Confirm needle position in the vein, recom-
mended to use LAX to complete the track of
The femoral vein (FV) cannulation is employed the needle inside the vessel and to avoid pos-
for central venous access where the superior terior wall puncture.
extremity central venous access is otherwise 5. Confirm wire position in the vein and track all
ruled out and for temporary intravenous pacing. the way to IJ and SV junction if wire is placed
A high-frequency linear array UST is selected. in IJV to ensure wire enters the innominate
The patient lies supine and the proceduralist vein prior to dilation over the guidewire.
stands on the ipsilateral side. The UST is placed 6. Confirm catheter position in the vein.
transversely, just inferior to the inguinal crease.
The superficial femoral artery can be easily iden- 2.3.3.2 Femoral Vein Cannulation
tified as an anechoic pulsatile structure with the
FV lying medial and inferior to it. The FV appears Limitations of Ultrasound-Guided Central
as an oval-shaped anechoic object which is com- Venous Catheter Placement
pressible by the UST. After adjusting the UST, so Although US is noninvasive and thus does not
that the FV lies in the center of the image, the bear a risk to directly harm the patient, some lim-
block needle is inserted out of the plane. The itations and disadvantages of US during central
UST is slided up and down to follow the needle venous access are worth considering. One might
trajectory till the tip of the needle appears at the argue that the risk of catheter-related bloodstream
upper surface of the vein as a bright dot, appears infections might be higher if US is used for CVC
to indent on the vein wall and then pierces it. placement without applying a strict aseptic
Dark venous blood can be aspirated with the approach. US might give a false sense of security
syringe attached. Guidewire is passed down. and mislead him/her to neglect traditionally
The entire procedure should be performed under taught principles with regard to needle direction
direct ultrasound imaging. The needle is to be especially for inexperienced users. To overcome
advanced only when visualized without any doubt. these problems formal education and training
The same process can be applied for a femoral including simulation with a structured certifica-
artery cannulation. tion of US skills for vascular access and the
development of a consensus standard for these
2.3.3.1 Systematic Approach for USG training programs have been suggested.
Central Venous Catheter Sometimes insertion site-related specific issues
Placement when using US.  For example, shorter necks in
Irrespective of cannulation site, for clinical prac- morbid obese individuals might make LAX a
tice, the authors recommend a systematic challenge to visualize the IJV and might need
approach using six steps: curved-array probes even for IJ placement. For
subclavian access, because the angle of cannula-
1. To identify the anatomy of the insertion site tion is usually steeper when using US, it is espe-
and localization of the vein, it is ­recommended cially important to align and constantly visualize
to perform a baseline scan prior to sterile the needle to avoid pleural injury.
preparation. It also helps to know anatomical
variants.
2. To confirm patency of the vein by using vari- 2.3.4 Confirmation of the Correct
ous US features such as color Doppler, com- Position of the Central Venous
pression US or even spectral Doppler in rare Catheter Tip
occasions.
3. Use real-time US guidance for puncture of the Prevention of central venous catheter tip malpo-
vein, one should be familiar with the depth sition is of paramount importance, as it has been
and angular orientation prior to placing the associated with significant complications, includ-
needle. ing central venous or superior vena cava throm-
2  Ultrasound-Guided Vascular Cannulation 29

bosis, arrhythmias, cardiac tamponade, and ferent echocardiographic views have been tested:
hemodynamic monitoring inaccuracy. Moreover, the apical four-chamber view, the subcostal four
with malposition, appropriate treatment may be chambers view, the subcostal bi-caval view and
delayed with subsequent further related compli- the suprasternal/supraclavicular view. Both of the
cations. In this regard, the intracavitary electro- four chambers views allow only evaluation of the
cardiographic (IC-ECG) method is currently right atrium without visualization of the superior
recommended in international guidelines as vena cava or inferior vena cava. On the contrary,
accurate, well-tolerated, and cost-effective for the subcostal bi-caval view, the most studied
assessing the proper location of the central approach, allows visualization of the superior
venous catheter tip. However, this method is vena cava, cavo-atrial junction, right atrium, and
commonly considered to be applicable only when inferior vena cava. Four groups of researchers
there is a well-defined and identifiable P wave in have studied the suprasternal/supraclavicular
the ECG trace. Although a few studies have view, which allows the identification of the con-
recently suggested that IC-ECG might also be fluence between the two brachiocephalic veins,
used in patients with atrial fibrillation after some the superior vena cava, the right branch of the
appropriate modifications of the basic technique, pulmonary, artery and the aortic arch. These
in patients with a pacemaker or with other structures enable indirect identification of the
arrhythmias, IC-ECG is still considered to be not cavo-atrial junction. Ultrasound-based tip navi-
applicable. As bedside chest radiograph has been gation may be used during the procedure to help
shown to be inaccurate in identifying the catheter the operator in directing the guidewire.
tip location due to the inaccuracy of the radio-
logical landmark for the cavo-atrial junction, 2.3.4.1 The European Society
ultrasound imaging has been proposed as an of Anesthesiologists
alternative technique to IC-ECG and chest radio- Recommends [40]
graph for tip location. Indeed, the application of 1. When an intracardiac electrocardiogram is
ultrasound to vascular access should not be lim- not applicable, using real-time ultrasound to
ited to venepuncture but should be extended to detect and prevent central venous catheter
assist in all steps of the procedure. Specifically, in malposition, as it has been shown to be well-­
regard to the prevention of primary malposition, tolerated, feasible, quickly performed, and
ultrasound may play two roles as a “tip location” interpreted at bedside, and more accurate and
and a “tip navigation” technique. Ultrasound-­ faster than a chest radiograph (1C).
based tip navigation techniques can be used to 2. Combining vascular ultrasound for guidewire
confirm that the catheter or the guidewire is and central venous line tip navigation with
threading towards the cavo-atrial junction by transthoracic echocardiography for tip loca-
sonographic visualization throughout the ipsilat- tion (1C).
eral brachiocephalic vein, ruling out catheter 3. Performing pleural and lung ultrasound
misdirection into the ipsilateral IJV or other (PLUS) to rule out potential pleural-­
superior vena cava tributary veins (e.g., the con- pulmonary complications (mainly pneumo-
tralateral brachiocephalic vein). Sonographic tip thorax) soon after the procedure in any
navigation may be performed with the same lin- difficult puncture of the subclavian or axillary
ear probe used for the puncture. As a tip location vein and, particularly, if the patient complains
technique, ultrasound allows direct or indirect of shortness of breath or discomfort that wors-
visualization of the catheter tip or the J-guidewire ens after catheter placement (1B).
at the cavo-atrial junction, upper right atrium, or 4. Using PLUS to monitor the development of a
in the lower superior vena cava by means of confirmed pleural-pulmonary complication or
transthoracic echocardiography. Different for follow-up of treatment (1B).
approaches and different protocols have been 5. Ultrasound for diagnosis and follow-up of
described in the literature on this topic. Four dif- catheter-related thrombosis (1C).
30 A. Chakraborty et al.

2.4 Peripheral Venous 2.4.3 Confirm Vein


Cannulation
The blood vessel located should be confirmed as
Ultrasound-guidance is often useful for periph- a vein using the following principles-
eral venous cannulation in patients in whom
peripheral venous cannulation without guidance (a) Patent peripheral veins easily and completely
is difficult, e.g., burn patients, patients of cancer collapse with gentle compression with the
chemotherapy, patients with peripheral edema probe.
where the veins cannot be seen otherwise. Obese (b) Nonpulsatile.
patients also benefit from ultrasound guidance in (c) Color Doppler can be used.
peripheral venous cannulation by reducing the (d) Pulsed wave Doppler also can be used to
number of attempts and increasing first pass and demonstrate the pulsatile flow pattern in
overall success rate [41–48]. The technique can adjacent arteries and the nonpulsatile, phasic
be real-time ultrasound-guided or US-assisted— flow in veins.
where the proceduralist may mark the location of
the veins with the ultrasound and then proceed 2.4.3.1 Identify the Appropriate Vein
with cannulation with usual technique. The usual The appropriate vein should be identified by
targets are the antecubital vein and the saphenous applying the following principles
vein.
(a) Diameter, depth, path: The vein should be of
an adequately large diameter, at an achiev-
2.4.1 Indications able depth, in a straight path.
• Using a standard 48  mm angiocatheter,
• Difficulty in locating veins via visualization or success rate drops to 0% when vessel
palpation due to. depth is more than 1.6 cm.
–– Body habitus. • No significant difference between more
–– Edematous skin. superficial veins at different depths,
• Multiple unsuccessful blind insertion attempts. • The success rate increases with increasing
• Severe dehydration. venous diameter.
• Multiple previous cannulations (e.g., intrave- –– 56% when less than or equal to 0.3 cm
nous drug use, chemotherapy). –– 92% when greater than or equal to
0.6 cm.

2.4.2 Procedure Desirable targets are therefore found between


0.3  cm and 1.5  cm from the surface, with an
2.4.2.1 Initial Scout internal diameter of at least 0.4 cm
Using the ultrasound transducer survey potential
vessels for cannulation should be scanned. Start (b) Note and avoid venous valves.
in the antecubital fossa with a transverse probe
orientation. Targets in the upper limb are basilic, 2.4.3.2 Preparation
brachial, and/or cephalic veins. The basilic vein, • Clean probe after initial scout.
while variably present, lacks flanking arteries and • Place a cover directly on the clean probe.
nerves, and is usually the more superficial target. • Apply a tourniquet to the upper aspect of the
In contrast, the deep brachial vein is near ubiqui- patient’s arm.
tously present, but has nerves and arteries in • Prepare the skin over the previously identified
close proximity, and is found at a greater depth. venous target with alcohol/antiseptic wipe.
2  Ultrasound-Guided Vascular Cannulation 31

2.4.3.3 Needle Insertion –– Process of moving ultrasound probe for-


• Don sterile gloves. ward off the needle tip, stabilizing and then
• US probe is held in the nondominant hand moving the needle further forward into the
with a stable grip. ultrasound’s view.
• Apply sterile gel.
• Check probe orientation: Touch one end of the 2.4.3.4 Confirmation of Cannulation
probe and watch for reaction on the monitor. 1. Visualization of cannula and needle within the
• Align for use on the patient so that the medial lumen of the vessel.
of the screen is medial of the patient and lat- (a) On transverse orientation: Bull’s eye
eral is lateral. sign,
• Locate the target, adjust depth. (b) On longitudinal orientation: needle seen
• Transducer orientations-. entering and lying within the lumen,
–– Transverse: 2. A positive “saline flush test.”
Advantages: Improved ability to center (a) Identify cannula in long axis and push
needle to the midline of vessel. 5–10 mL of saline,
Disadvantages: Loss of direct needle (b) Positive test is direct visualization of bub-
tip visualization each time the probe or bles within the lumen,
the needle is moved. (c) May be aided by color Doppler 4,
–– Longitudinal: 3. Flashback of blood through the cannula.
Advantages: Entire needle visualized
throughout the procedure with a better The ultrasound probe can be put down at this
perception of depth within the vessel. point so that both hands can be used to advance
Disadvantages: Inability to identify if the catheter, remove the needle, attach the bung,
the needle is off the midline of the ves- flush the cannula, clean the surrounding skin,
sel. For narrower vessels, it is difficult to and secure it in place with a transparent
keep the transducer centered on the ves- dressing.
sel lumen in long axis.
–– Optional confirmation of position prior to 2.4.3.5 Complications
insertion of needle by placing needle In comparison to blinded techniques, complica-
between transducer and skin to illicit tions associated with peripheral IV insertion
shadow artifact. under ultrasound guidance are typically minor
–– Note the depth of the artery to approximate but include:
final insertion depth.
• Insert needle through skin at a 45° or lesser • Nerve injury: Median or median cutaneous
approach angle. nerve.
–– Concentrate on monitor after initial • Arterial cannulation: Highlights the impor-
insertion, tance of confirming venous characteristics on
–– Find needle tip through fanning or small ultrasound prior to cannulation.
movements of ultrasound prior to further
movement. Tilting the probe towards the
approaching needle in the short axis (trans- 2.5 Arterial Cannulations
verse orientation) is helpful to locate the
advancing tip without moving the probe 2.5.1 Scout Scan
from its position.
• Progressive targeted movement of needle After proper positioning, an initial scan is impor-
towards vessel. tant to locate the artery, confirm using color
–– Incremental movements of 1 mm at a time Doppler and or power Doppler. Note the depth
directed towards vessel. and diameter of the artery.
32 A. Chakraborty et al.

2.5.1.1 Preparation movement. Tilting the probe towards the


• Clean probe after initial scout. approaching needle in the short axis (trans-
• Place a sterile cover on the clean probe. verse orientation) is helpful to locate the
• Confirm position of the limb. advancing tip without moving the probe
• Prepare the skin over the previously identified from its position.
target with alcohol/antiseptic wipe. • Progressive targeted movement of needle
• A window drape may be placed. towards vessel.
• Select the cannula/catheter. –– Incremental movements of 1 mm at a time
directed towards vessel.
2.5.1.2 Needle Insertion –– Process of moving ultrasound probe for-
• Don sterile gloves. ward off the needle tip, stabilizing and then
• The US probe is held in the nondominant hand moving the needle further forward into the
with a stable grip. ultrasound’s view.
• Apply sterile gel.
• Check probe orientation: Touch one end of the 2.5.1.3 Confirmation of Cannulation
probe and watch for reaction on the monitor. 1. Visualization of cannula and needle within the
• Align for use on the patient so that the medial lumen of the vessel.
of the screen is medial of the patient and lat- (a) On transverse orientation: Bull’s eye
eral is lateral. sign,
• Locate the target, adjust depth. (b) On longitudinal orientation: Needle is
• Transducer orientations-. seen entering and lying within lumen,
–– Transverse: 2. A positive “saline flush test.”
Advantages: Improved ability to center (c) Identify cannula in long axis and push
needle to the midline of vessel. 5–10 mL of saline,
Disadvantages: Loss of direct needle (d) Positive test is direct visualization of bub-
tip visualization each time the probe or bles within lumen,
the needle is moved. (e) May be aided by color Doppler 4.
–– Longitudinal: 3. Flashback of blood through cannula.
Advantages: Entire needle visualized
throughout the procedure with a better The ultrasound probe can be put down at this
perception of depth within the vessel. point so that both hands can be used to advance
Disadvantages: Inability to identify if the catheter, remove the needle, attach the pres-
needle is off the midline of the vessel. sure monitoring line, flush the cannula, clean the
For narrower vessels, it is difficult to surrounding skin, and secure it in place with a
keep the transducer centered on the ves- transparent dressing or suture as deemed
sel lumen in long axis. necessary.
–– Optional confirmation of position prior to
insertion of needle by placing needle 2.5.1.4 Complications
between transducer and skin to illicit In comparison to blinded techniques, complica-
shadow artifact. tions associated with arterial cannulation under
–– Note the depth of the artery to approximate ultrasound guidance are negligible.
final insertion depth.
• Insert needle through skin at a 45° or lesser
approach angle. 2.5.2 Radial Artery Cannulation
–– Concentrate on monitor after initial
insertion, Radial artery cannulation is usually done without
–– Find needle tip through fanning or small ultrasound guidance but ultrasound-guided tech-
movements of ultrasound prior to further nique is an important skill that each trainee must
2  Ultrasound-Guided Vascular Cannulation 33

learn to be able to complete the task of radial 2.5.4 Arteria Dorsalis Pedis
artery cannulation even after a few missed Cannulation
attempts.
Radial artery is superficial and the diameter Arteria dorsalis pedis (ADP) cannulation is use-
of the lumen is about 3–4  mm. Being a small ful for head and neck surgery with plastic recon-
and superficial target, it is easy to miss. Out-of- struction that would need a radial free flap. It is a
plane approach is generally preferred, in-plane small and superficial target, easy to miss. A short-­
approach is possible too! When performing the axis view is to be obtained first to identify the
cannulation out-of-plane, slide the probe up artery. In contrast with most arteries, ADP is a
and down and tilt towards the advancing needle superficial artery which is easily collapsible by
to locate the tip. Once blood appears in the hub the pressure of the UST. Therefore, after identifi-
of the cannula, an attempt should be made to cation, use of color Doppler is warranted to con-
take a long-axis view and insert the cannula a firm the artery. Once confirmed, the UST should
few mm more inside the lumen. Then the cath- be rotated by 90° while keeping it centered on the
eter is slided in, keeping the stylet fixed with artery to obtain the long-axis view (LAX).
the other hand. Careful alignment of the UST is needed to get the
In the catheter-over-needle technique, as is complete LAX view. Colour Doppler is used
most commonly performed, often it happens that again to confirm (Fig.  2.10). In-plane approach
blood appears at the hub indicating that the can- can be taken to cannulate the artery if a good
nula has entered the artery, but upon advance- LAX view can be obtained. Alternatively, an out-­
ment of the catheter it gets kinked and it has to be of-­plane approach can be performed. When going
taken out and reinserted. The angle of the cannula out of plane, slide the probe up and down and tilt
should be kept acute, less than 30° so that after towards the advancing needle to locate the tip.
entering the artery, a few mm more can be
inserted to avoid kinking of the catheter.
2.5.5 Posterior Tibial Artery
Cannulation
2.5.3 Femoral Artery Cannulation
The posterior tibial artery (PTA) cannulation can
Femoral artery is cannulated for percutaneous be useful for head and neck surgeries and particu-
coronary intervention (PCI) and other intra-­ larly in children, in whom the radial or the ADP
arterial interventions such as thrombectomy and may be too small to cannulate. It is an important
recanalisation. It is sometimes used in cardiac skill to learn.
anesthesia and cardiothoracic intensive care units Has been used in small children as an alter-
for arterial pressure monitoring and intra-arterial native to the radial artery as the lumen
balloon pump (IABP). (ID>1.1  mm) can generally accommodate a
The process of cannulation is similar to the 20 G arterial cannula [49].
femoral vein cannulation. An out-of-plane
approach and a Seldinger technique is used, i.e., 2.5.5.1 Technique
a needle is inserted in the artery, upon obtaining Site: Midpoint between the medial malleolus and
pulsatile blood flow a guidewire is inserted, the the Achilles tendon.
needle is taken out followed by insertion of the Position: Ankle dorsiflexion and eversion sig-
catheter over the guidewire. For safety precaution nificantly increase the proximity to the skin sur-
against accidental dislodgement of the catheter face. An assistant may be asked to hold the foot
thus inserted, many centers practice suturing the in position for cannulation.
catheter hub with skin before taking the guide- Posterior tibial vessels lie in close proximity
wire out. to the tibial nerve in a neurovascular bundle
34 A. Chakraborty et al.

Fig. 2.10 Cannulation
1. Subclavian vein (SCV) and
of the subclavian vein artery (SCA) are identified
(SCV)—stepwise in the long axis view
approach. The needle
(Arrowheads) enter
lateral to medial.
Subclavian artery (SCA)
lies below the SCV

2. In-plane cannulation is
performed. Note the needle
tip (arrows) in the center of
the vein

3. Guidewire (arrows) is
inserted and imaged with
ultrasound

4. Ultrasound transducer is
placed above the clavicle
and tilted caudally to
observe the guidewire
(arrows) entering the
Superior vena cava (SVC).
The guidewire appears as a
shiny line moving with each
heartbeat.
2  Ultrasound-Guided Vascular Cannulation 35

Fig. 2.11  Supraclavicular approach short axis

(Figs. 2.11 and 2.12). Compression with the UST 2.7.2 Positioning the Patient
will collapse the veins, allowing the artery to be for Needle Insertion
identified, which lies flanked on both sides by the and Catheter Placement
veins. Color Doppler should be used to confirm
the artery. Once the PTA is identified a LAX view • Perform central venous access in the neck or
is obtained and in-plane cannulation is performed chest with the patient in the Trendelenburg
(Fig. 2.12). Out-of-plane cannulation can also be position when clinically appropriate and
performed, depending on the experience and feasible.
preference of the proceduralist (Figs. 2.13, 2.14,
2.15, 2.16, 2.17, 2.18, 2.19, and 2.20).
2.7.3 Needle Insertion, Wire
Placement, and Catheter
2.6 Recommendations Placement
for Ultrasound-Guided
Vascular Access [1] • Select catheter size (i.e., outside diameter) and
type based on the clinical situation and skill/
The 2020 practice guidelines from the ASA experience of the operator.
recommends-. • Select the smallest size catheter appropriate
for the clinical situation.
• For the subclavian approach select a thin-wall
2.7 Recommendations needle (i.e., Seldinger) technique vs a catheter-­
for Prevention of Mechanical over-­the-needle (i.e., modified Seldinger)
Trauma or Injury technique.
• For the jugular or femoral approach, select a
2.7.1 Catheter Insertion Site thin-wall needle or catheter-over-the-needle
Selection technique based on the clinical situation and
the skill/experience of the operator.
• Determine catheter insertion site selection • For accessing the vein before threading a dila-
based on clinical need and practitioner judg- tor or large-bore catheter, base the decision to
ment, experience, and skill. use a thin-wall needle technique or a catheter-­
• Select an upper body insertion site when pos- over-­the-needle technique at least in part on
sible to minimize the risk of thrombotic com- the method used to confirm that the wire
plications relative to the femoral site. resides in the vein (Fig. 2.21).
36 A. Chakraborty et al.

Fig. 2.12 IJV
cannulation 1. Identify anatomy of the insertion site and localization of the vein

a. Identify the vein,


artery and nearby
structures
b. Image in SAX and
LAX
c. Perform scan before
prepping

2. Confirm patency of the vein


a. Confirm compressability
of the vein
b. Scan up and down to
observe blood flow
c. Use colour Doppler

3. Use real time US guidance for puncture of the vein


a. Full sterile precautions
b. Use SAX or LAX or
both
c. Keep the needle tip
under “vision” all the
time.

4. Confirm needle position in the vein


a. Confirm that the needle
tip is placed centrally in
the vein in SAX before
inserting the guidewire.
b. Observe the needle in
LAX

5. Confirm wire position in the vein

a. Confirm position of
the guidewire inside
the vein.
b. Tilt the probe caudally
to notice the guidewire
entering the superior
vena cava

6. Confirm catheter position in the vein

a. Confirm the position


of the catheter
inside the vein in
SAX and LAX
b. Confirm free flow of
blood from the hubs
2  Ultrasound-Guided Vascular Cannulation 37

Femoral vein lies medial to the artery Obtain LAX view by rotating the
probe by 90 degrees
Identify anatomy of the insertion site and localization of the vein

The femoral vein lies


medial to the femoral
artery

Confirm patency of the vein

a. The vein should be


collapsible under
pressure from the UST
b. Obtain M mode image
of venous flow

Fig. 2.13  Femoral vein cannulation


38 A. Chakraborty et al.

Use real time US guidance for puncture of the vein

a. The FV can be
punctured in SAX or
LAX view.
b. Image the
needle tip in the center
of the vein (arrows).

Fig. 2.13 (continued)
2  Ultrasound-Guided Vascular Cannulation 39

Confirm wire position in the vein

Confirm catheter position in the vein

The catheter can be


imaged inside the vein in
LAX or SAX view.
In the SAX, a “target” sign
can be seen.

Fig. 2.13 (continued)

• The number of insertion attempts should be


based on clinical judgment.
• The decision to place two catheters in a single
vein should be made on a case-by-case basis.

2.7.4 Guidance of Needle, Wire,


and Catheter Placement

• Use real-time ultrasound guidance for vessel


Fig. 2.14  Venous cannulation in the long-axis view and localization and venipuncture when the inter-
in-plane approach; Arrowheads represent the advancing nal jugular vein is selected for cannulation
cannula (Fig. 2.21).
40 A. Chakraborty et al.

SV identified in Confirmed patency SV imaged in long In-plane


SAX with color doppler axis cannulation

Fig. 2.15  Cannulation of the saphenous vein (SV)

1 Artery is
identified in
SAX

2 LAX view
is obtained,
confirmed
using color
doppler

3 In plane
cannulation
is
performed.
Confirm by
observing
the blood
flashback
as well as
the cannula
images
inside the
artery

Fig. 2.16  Cannulation of the radial artery in 3 steps: 1. real-time imaging (note the blood in the hub and the can-
Identification of the artery in the short-axis view, 2. Long-­ nula inside the artery—arrows)
axis view with color Doppler, 3. In-plane cannulation with
2  Ultrasound-Guided Vascular Cannulation 41

Fig. 2.17 Cannulation
of the femoral artery
in short-axis view; note
the needle tip
(Arrow) inside the artery

Identification of the ADP in In-plane cannulation of ADP After blood flashback the
LAX with color doppler catheter is advanced in the
artery in-plane

Fig. 2.18  Arteria dorsalis pedis (ADP) cannulation

Fig. 2.19 Short-axis
view of the posterior
tibial artery with color
Doppler (note the red
color)
42 A. Chakraborty et al.

Tibial nerve and the posterior Posterior tibial artery in the In-plane cannulation of PTA
tibial vessels in the SAX view LAX with color doppler

Fig. 2.20  Cannulation of posterior tibial artery

Catheter-over the Needle


Thin-Wall Needle (Seldinger) Technique (Modified Seldinger) Technique
Use Real-Time Use Real-Time
Ultrasound Ultrasound

Needle appears to be in venous NO NO Needle appears to be in venous


system system

YES

DO
YES Slide catheter over needle into vessel
NOT
PROCEED

Confirm venous placement Confirm venous placement


(manometry, pressure measurement, NO NO (manometry, pressure measurement,
or ultrasound)† or ultrasound)

YES YES

Thread wire NO Thread wire

Any question
of difficulty
No
difficulty‡
Confirm venous residence of wire with
ultrasound, TEE, continuous ECG, or
fluoroscopy

YES

Proceed with dilator and catheter placement

Fig. 2.21  Algorithm for central venous insertion and veri- wall needle in the Seldinger technique could move out of
fication. This algorithm compares the thin-wall needle the vein and into the wall of an artery between the manom-
(i.e., Seldinger) technique vs the catheter-over-the needle etry step and the threading of the wire step. ECG, electro-
(i.e., modified Seldinger) technique in critical safety steps cardiography; TEE, transesophageal echocardiography.
to prevent unintentional arterial placement of a dilator or †
For neonates, infants, and children, confirmation of
large-bore catheter. The variation between the two tech- venous placement may take place after the wire is threaded.
niques reflects mitigation steps for the risk that the thin-­ ‡
Consider confirming venous residence of the wire [1].
2  Ultrasound-Guided Vascular Cannulation 43

–– When feasible, real-time ultrasound may • Verify that the wire has not been retained in
be used when the subclavian or femoral the vascular system at the end of the procedure
vein is selected. by confirming the presence of the removed
• Use static ultrasound imaging before prepping wire in the procedural field.
and draping for prepuncture identification of –– If the complete guidewire is not found in
anatomy to determine vessel localization and the procedural field, order chest radiogra-
patency when the internal jugular vein is phy to determine whether the guidewire
selected for cannulation. has been retained in the patient’s vascular
–– Static ultrasound may also be used when system.
the subclavian or femoral vein is selected.

2.8 Recommendations
2.7.5 Verification of Needle, Wire, for Management of Arterial
and Catheter Placement Trauma or Injury Arising
from Central Venous Access
• After insertion of a catheter that went over the
needle or a thin-wall needle, confirm venous • When unintended cannulation of an arterial
access. vessel with a dilator or large-bore catheter
–– Do not rely on blood color or absence of occurs, leave the dilator or catheter in place
pulsatile flow for confirming that the cath- and immediately consult a general surgeon, a
eter or thin-wall needle resides in the vein. vascular surgeon, or an interventional radiolo-
• When using the thin-wall needle technique, gist regarding surgical or nonsurgical catheter
confirm venous residence of the wire after the removal for adults.
wire is threaded. • For neonates, infants, and children, deter-
–– When using the catheter-over-the-needle mine on a case-by-case basis whether to
technique, confirmation that the wire leave the catheter in place and obtain consul-
resides in the vein may not be needed (1) tation or to remove the catheter
when the catheter enters the vein easily and nonsurgically.
manometry or pressure-waveform mea- • After the injury has been evaluated and a treat-
surement provides unambiguous confirma- ment plan has been executed, confer with the
tion of venous location of the catheter and surgeon regarding relative risks and benefits
(2) when the wire passes through the cath- of proceeding with the elective surgery vs
eter and enters the vein without difficulty. deferring surgery to allow for a period of
–– If there is any uncertainty that the catheter patient observation.
or wire resides in the vein, confirm venous
residence of the wire after the wire is
threaded; insertion of a dilator or large-­ 2.8.1 Resource Preparation
bore catheter may then proceed.
• After final catheterization and before use, con- • Perform central venous catheterization in an
firm residence of the catheter in the venous environment that permits use of aseptic
system as soon as clinically appropriate. techniques.
• Confirm the final position of the catheter tip as • Ensure that a standardized equipment set is
soon as clinically appropriate. available for central venous access.
–– For central venous catheters placed in the • Use a checklist or protocol for placement and
operating room, perform a chest radiograph maintenance of central venous catheters.
no later than the early postoperative period • Use an assistant during placement of a central
to confirm the position of the catheter tip. venous catheter.
44 A. Chakraborty et al.

2.8.2 Prevention of Infectious • For neonates, determine the use of transparent or


Complications sponge dressings containing chlorhexidine based
on clinical judgment and institutional protocol.
2.8.2.1 Intravenous Antibiotic • If a chlorhexidine-containing dressing is used,
Prophylaxis observe the site daily for signs of irritation,
• Do not routinely administer intravenous anti- allergy, or necrosis.
biotic prophylaxis.
2.8.2.7 Catheter Maintenance
2.8.2.2 Aseptic Preparation • Determine the duration of catheterization
• In preparation for the placement of central based on clinical need.
venous catheters, use aseptic techniques (e.g., • Assess the clinical need for keeping the cath-
hand washing) and maximal barrier precau- eter in place on a daily basis.
tions (e.g., sterile gowns, sterile gloves, caps, • Remove catheters promptly when no longer
masks covering both mouth and nose, full-­ deemed clinically necessary.
body patient drapes, and eye protection). • Inspect the catheter insertion site daily for
signs of infection.
2.8.2.3 Selection of Antiseptic Solution • Change or remove the catheter when catheter
• Use a chlorhexidine-containing solution for insertion site infection is suspected.
skin preparation in adults, infants, and children. • When a catheter-related infection is suspected,
–– For neonates, determine the use of a new insertion site may be used for catheter
chlorhexidine-containing solutions for skin replacement rather than changing the catheter
preparation based on clinical judgment and over a guidewire.
institutional protocol.
• If there is a contraindication to chlorhexidine,
povidone–iodine or alcohol may be used. 2.9 Aseptic Techniques Using
• Unless contraindicated, use skin preparation an Existing Central Venous
solutions containing alcohol. Catheter for Injection or
Aspiration
2.8.2.4 Catheters Containing
Antimicrobial Agents • Clean catheter access ports with an appropri-
• For selected patients, use catheters coated with ate antiseptic (e.g., alcohol) before each access
antibiotics, a combination of chlorhexidine and when using an existing central venous catheter
silver sulfadiazine, or silver-­platinum-­carbon– for injection or aspiration.
impregnated catheters based on risk of infec- • Cap central venous catheter stopcocks or
tion and anticipated duration of catheter use. access ports when not in use.
–– Do not use catheters containing antimicro- • Needleless catheter access ports may be used
bial agents as a substitute for additional on a case-by-case basis.
infection precautions.

2.8.2.5 Catheter Fixation 2.10 Recognized Benefits


• Minimize the number of needle punctures on and Concerns about
the skin. Ultrasound-Guided Vascular
Access
2.8.2.6 Insertion Site Dressings
• Use transparent bioocclusive dressings to pro- 2.10.1 Recognized Benefits
tect the site of central venous catheter inser-
tion from infection. Traditionally cannulation of vascular structures
• Unless contraindicated, dressings containing are based on palpatory and landmark-based tech-
chlorhexidine may be used in adults, infants, niques. Ultrasound(US)-guided vascular cannula-
and children. tion has shown to improve success rate in the first
2  Ultrasound-Guided Vascular Cannulation 45

attempt, reduce the number of overall attempts, tion of small hematomas was observed in 6% of
improve patient’s satisfaction and reduce overall cases [62, 63]. Similar findings were reported
procedure-related complications [50–54]. while subclavian venous cannulation under US
US aid leads to increased success rates of guidance in patients with deranged coagulation
catheter placement in the first attempt, both in parameters [64].
central venous catheters [55] and arterial cathe- US is useful in patients who are obese, edema-
ters, in adults and children [56]. Shiloh et al. [57] tous, have hematomas from previous attempts, or
reported an increase of 71% in the success rate of have weak or missing arterial pulsations. During
radial artery catheterizations. cardiopulmonary resuscitation, US can visualize
Estimates of the complication rate of central the vascular anatomy without relying on an arte-
venous catheterization without the use of US rial pulsation [65]. In patients with nonpulsatile
range between 5% and 19% [58, 59]. Us-guided ventricular assist device in situ, the peripheral
vascular punctures lead to reduced complications pulses are weak or absent and most of them are
associated with traditional techniques, such as anticoagulated, inadvertent arterial puncture dur-
failure of catheter placements, arterial puncture, ing central venous cannulation may lead to seri-
hematoma formation, hemothorax, and pneumo- ous complications [66].
thorax. Hind et al. [55] in a meta-analysis com- In addition, US can be used to screen for ves-
pared central venous catheterization using sel patency, vascular abnormalities, and anatomi-
US-guided puncture with those carried out by cal variations, which can be useful in patients
traditional methods. They reported a reduction of having variations in the relationship between the
86% in relative risk of catheter placement fail- internal jugular vein and carotid artery [67, 68].
ures using ultrasonography, 57% reduction was There are very few alternatives to using US
seen in relative risk of mechanical complications and obtaining visual feedback during vessel can-
(p = 0.02). nulation. Most of them, such as fluoroscopy, are
In another systematic review, which included not as readily available, require specialized train-
cannulation of various central venous structures, ing and certification, and cost.
it was reported that US aid leads to reduction of
82% of relative risk of failure of catheterization
and significantly less occurrence of arterial punc- 2.11 Concerns and Limitations
tures, local hematomas, pneumothorax, and of us-Guided Vascular Access
hemothorax had a reduction of relative risk of
75%, 70%, 79%, and 90%, respectively [60]. US itself is noninvasive and does not cause any
US-guided cannulations lead to cost saving. harm to the patient directly, but it guides an inva-
Calvert et al. estimated a saving of 2000 pounds sive procedure (needle/catheter insertion into a
for each thousand venous central catheterization vessel), and, thus, may result in complications.
guided by ultrasonography compared with those The following concerns can be identified while
guided by anatomic parameters [61]. Reduction using US for vascular puncture/cannulation.
in the average time spent in the procedure is also The operator is usually busy looking at the US
reported as fewer attempts are needed to success- screen searching for the needle, which is always
fully puncture the vessel [56]. not clearly visible especially during the out-of-­
US is found to be useful in patients with bleed- plane approach, which can lead to inadvertent
ing diathesis or coagulopathies. Using the tradi- carotid puncture. The needle also may exit the
tional method, Fisher et al. [57] reported a rate of posterior wall of the jugular vein and enter the
9% of minor complications (hematoma forma- carotid artery [69]. The long-axis approach may
tion and bleeding on puncture site) in patients not prevent this complication as the carotid artery
with abnormal platelet count or coagulation is not visualized side by side, and a slight oblique
parameters. In another study, no major complica- needle orientation may not allow proper needle
tions were reported in 133 patients with deranged visualization. Therefore, the incidence of carotid
coagulation. Bleeding on puncture site or forma- puncture during right IJV cannulation can be as
46 A. Chakraborty et al.

high as 4% [70] despite using US [71–73]. better safety profile, ultrasound (US)-assisted
Similarly during subclavian vein cannulation, vascular cannulation is being preferred over clas-
inadvertent puncture of the pleura may result in a sic landmark-based techniques. Literature regard-
pneumothorax. ing training, supervision, and competence needed
In patients with shorter neck anatomy, the regarding central venous access device (CVAD)
long-axis US view of the internal jugular vein insertion is scarce and not fully standardized. No
may be difficult to obtain. The use of US to punc- standard didactic or simulation training is
ture the subclavian vein results in a puncture site required before CVAD insertions by clinicians in
that is usually more lateral compared to the land- training, neither the role, experience, or compe-
mark puncture technique. The close proximity of tence of the supervisor is specified [77].
the vessels and the pleura must be kept in mind Ultrasound guidance CVAD insertion needs
because the angle of cannulation is usually education and training to ensure patient safety
steeper when using US, it is especially important and avoid major complications with the insertion
to align and constantly visualize the needle to of CVAD [78–82].
avoid pleural injury [74]. Understanding of basic anatomy, ultrasound
US guidance does not necessarily mean cor- physics and imaging, and infection prevention
rect placement of the guidewire in the vascular strategies are necessary [45, 83].
lumen, rather the visualization of the guidewire Multiple training techniques have been
in the venous lumen via peripheral ultrasonogra- described to aid the use of ultrasound for central
phy can be misleading as the wire may have venous cannulation [45, 84]. All forms of training
punctured the posterior wall and entered the should lay importance on developing proficiency
carotid artery [75]. in both cognitive and psychomotor skill sets and
Other concerns of US-guided vascular access must include image acquisition, interpretation,
are infections if a proper aseptic technique is not real-time use of ultrasound for vessel puncture
applied, procedural delays due to nonavailability and cannulation, and an experienced instructor
of the equipment, and the costs involved with the who demonstrates to the trainee how to translate
buying and maintaining the US equipment [76]. 2D imaging to perform a 3D task [85].
Routine US use may result in a “de-skilling” In 2013, an international evidence-based con-
of the landmark-based techniques as they are less sensus task force was established through the
taught and practiced nowadays, which might World Congress of Vascular Access (WoCoVA)
result in higher complication rates when US is to provide definitions and recommendations for
not available [76]. training and insertion of CVADs.
To achieve adequate skills for using US skills The task force proposed 16 recommendations.
high-quality care, formal education and training
(including simulation) with a structured certifica- 1. Anatomy and physiology of relevant body
tion of US skills for vascular access and the system.
development of a consensus standard for these 2. Ultrasound for insertion and assessment.
training programs have been suggested. 3. Central venous device tip location.
4. Infection control and sterile technique.
5. Device selection and indications.
2.12 Training 6. Insertion procedures, complication preven-
tion, evaluation, and management.
2.12.1 Training for Ultrasound-­ 7. Care and maintenance practices.
Guided Vascular Cannulations 8. Qualification and competency.
9. Simulation training.
Vascular cannulation is an important aspect of 10. Anatomical models.
patient care for the administration of fluids and 11. Objective grading and proficiency.
medications and for monitoring purposes. Due to 12. Examination and competency.
2  Ultrasound-Guided Vascular Cannulation 47

13. Supervised instruction. undergo initial basic US training varying in


14. Didactic or web-based training. length from 1 to 2 days followed by at least 25
15. Developing clinical competence. documented and reviewed cases of primary appli-
16. Education for children and neonates. cations including US-guided procedures such as
central venous cannulations.
Simulation-based education has recently There is a need for establishing consistency in
emerged as an important learning tool in several the development of training programs and mea-
areas of surgical expertise [86, 87] and it is partic- suring competency through completion of didac-
ularly useful for teaching invasive procedures such tic lessons, simulation, examination, and
as CVAD insertion, which require eye-hand coor- supervised practice. More research is necessary
dination and ambidextrous maneuvers [88, 89]. to establish stronger recommendations and
Simulation allows residents-in-training to repeat- clearer directives.
edly practice a procedure prior to performing it on
a living patient and also has potential benefits to
improve provider performance, to reduce errors, 2.13 Conclusion
and, ultimately, to enhance patient safety.
Multiple studies have assessed the learning Using ultrasound for vascular cannulations is an
effect of a standardized simulation-based teach- important skill a physician should learn and keep
ing program on ultrasound-guided cannulation in on practicing. Using ultrasound increases success
a low-cost cadaver tissue model. Some of them rate, reduces complications, and thereby reduces
have used avian muscle, chicken or turkey thigh the cost of health care. In the near future ultra-
and have inserted fluid-filled latex/elastic tubes, sound is going to become more and more avail-
into cadaver tissue to mimic vessels to create a able for all healthcare workers making the skill of
model for central venous catheterization, while ultrasound-guided vascular cannulations essen-
others have used latex, gelatin, or silicone rubber tial for a successful physician.
models, where plastic structures simulating cen-
tral vessels are inserted. Successful attempts,
number of failed attempts, and insertion times
were evaluated as end-points of the training POCUS Pearls
module. • A shallower angle of the needle is
There is a paucity of literature to specifically required for cannulating superficial and
suggest the number of procedures necessary to smaller vessels.
develop competence in performing real-time • Deeper vessels are better cannulated
ultrasound cannulation. One has to be mindful of with a steep needle angle to facilitate
the fact that clinicians acquire knowledge and location of the needle tip.
develop dexterity for the technique at different • While cannulating the radial artery,
rates. The opinion among expert users with make sure the BP cuff is on the other
>10 years of experience with this technique has hand or the automatic cycle timer is off
suggested that training includes a minimum of 10 if it is on the same hand.
procedures performed under the guidance of an • Check the BP before arterial cannula-
experienced user. tion. If the pressure is low, expect diffi-
There are no clear recommendations on the culty and consider administration of a
total number of hours that trainees must practice short acting vasoactive agent such as
to acquire US-guided CVC skills. In fact, The phenylephrine to increase BP
American College of Emergency Physicians rec- temporarily.
ommends that all US practitioners should
48 A. Chakraborty et al.

 elf Assessment Questions: (Answer


S D. Scan up and down the IJV to identify the
Keys at the End of the Chapter) point of puncture where there is mini-
mum overlap with common carotid artery
1. When using ultrasound to assist in cannula- (CCA)
tion of the radial artery, which of the follow- E. All of the above
ing techniques are recommended 5. During cannulation of the femoral vein,
A. Artery can be identified by color Doppler which of the following steps should be done
B. Angle of the entry is usually about 30° or to ensure proper placement of the central
less venous catheter?
C. Distance from the center of the probe to A. Confirm guidewire position on short-axis
point of entry varies between individuals view (SAX) with the ultrasound probe
D. Out-of-plane technique is usually used placed parallel to the inguinal crease
E. All of the above B. Ensure free flow of blood from the
2. When using ultrasound to identify structures cannula
for vascular cannulation, which of the fol- C. Place ultrasound probe on the inguinal
lowing options can be used to confirm that a crease
blood vessel is vein? D. Slide ultrasound probe up and down to
A. Color Doppler showing blue Doppler follow the needle tip and advance only
flow when needle is visualized
B. Compressibility under the gentle pres- E. Visualize the femoral vein to be lateral to
sure of ultrasound probe the superficial femoral artery
C. Presence of hypoechoic content within 6. When cannulating the IJV, which one of the
the lumen and posterior wall showing following statements is true
enhancement artifact A. CCA lies directly posterior to IJV hence
D. Pulsed wave Doppler showing pulsatile when both vessels are observed to be
waveform overlapping significantly, cannulation
E. Identifying surrounding structures can still occur at that position
3. No flashback is seen in the hub during can- B. In short-axis view (SAX), the echogenic
nulation of the peripheral vein under ultra- point seen is always the tip of the needle
sound guidance. What is the most likely C. Long-axis view of the IJV can prevent
reason for this? carotid artery puncture and hence always
A. Low-frequency ultrasound probe is being required prior to cannulation
used D. Once placement of guidewire is con-
B. Patient is hypotensive firmed on ultrasound, arterial puncture is
C. The vein is thrombosed unlikely
D. Venous valve is preventing any E. None of the above
flashback 7. During cannulation of the subclavian veins,
E. Wrong depth is being targeted which one of the following statements is true
4. During internal jugular vein (IJV) cannula- A. In an infraclavicular approach, the ultra-
tion, which of the following steps can be sound probe is first placed perpendicular
taken to minimize the risk of inadvertent to the clavicles to identify all the vascular
arterial puncture? structures
A. Avoid hyperextension and sideway rota- B. In-plane approach is usually taken for
tion of head at the time of positioning cannulation to avoid pleura puncture
B. Identify the lateral and medial wall of the C. Subclavian artery must be visualized
IJV before entering the skin prior to cannulation
C. Note the depth of the needle entering the D. Subclavian vein lies superior to the sub-
skin at any time clavian artery
2  Ultrasound-Guided Vascular Cannulation 49

E. All of the above C. When transduced, central venous wave-


8. When cannulating radial artery, which of the forms are shown
following statement is true D. When the catheter enters the vein easily
A. Ensure the patient is not hypotensive with no resistance
B. Blood pressure cuff should not be applied E. All of the above
onto the same arm 12. When inadvertent arterial trauma occurs

C. Needle often enters at a shallow angle after a dilator or large-bore catheter is intro-
D. When using needle in catheter technique duced during central venous catheter place-
and flashback is obtained, a few mm ment, what would be the immediate next
more can be inserted to avoid kinking of step?
the catheter during advancement A. Do not proceed further
E. All of the above B. Flush the large-bore catheter
9. After placement of the needle and flashback C. Put back the guidewire through the
obtained during cannulation of the central dilator
vein, what would be the immediate next D. Place patient in Trendelenburg position
step? E. Remove the dilator immediately
A. Insertion of guidewire through the 13. Which of the following methods is the most
needle effective in minimizing central line associ-
B. Confirmation of needle position within ated bloodstream infection?
the vascular lumen via ultrasound A. Aseptic placement technique at the time
C. Dilation of vein to admit the central line of placement
catheter B. Assess clinical need for catheter daily
D. Flush with saline and observe them for and remove catheter promptly when no
bubbles within the lumen on ultrasound longer deemed clinically necessary
E. Draw blood to test for blood gas C. Clean catheter access ports with an
10. When performing infraclavicular subclavian appropriate antiseptic solution before
vein insertion, which of the following com- each access
plications can be avoided if needle tip is con- D. Routine use of intravenous antibiotics
stantly visualized prophylaxis
I. Central line catheter entry into common E. Use of chlorhexidine-containing solution
carotid artery for skin preparation during placement of
II. Central line catheter entry into IJV CVL
III. Pleura puncture and pneumothorax 14. Which of the following option determines
IV. Subclavian artery puncture the angle of entry of the needle during
A. I ultrasound-­guided CVL placement?
B. I and II A. Distance from footprint of the ultrasound
C. II probe to the point of entry
D. III and IV B. Depth of the target vein
E. All of the above C. Location of the CVL
11. When using catheter-over-needle technique, D. Weight of the patient
ultrasound confirmation of guidewire in the E. All of the above
vein is not necessary when the following 15. When planning for subclavian vein cannula-
occurs: tion, which of the following statement is true
A. Blood gas performed through the cathe- with regard to the anatomical relations of the
ter unambiguously shows venous origin subclavian vein?
B. Guidewire can enter the vein without any A. Axillary vein can be seen joining the sub-
difficulty clavian vein
50 A. Chakraborty et al.

B. Internal jugular vein can be seen joining A. LAX in-plane approach is preferred as
the subclavian vein above the clavicle the depth of the needle tip can be seen
C. Pleura can be seen inferior to the subcla- and hence reducing the risk of arterial
vian vein puncture
D. Subclavian artery lies posterior to the B. LAX in-plane approach allows the entire
subclavian vein needle to be visualized and hence suc-
E. All of the above cess rate is higher
16. Difficult cannulation is expected for an obese C. SAX out-of-plane approach allows better
individual who requires CVL placement. visualization of vein in relation to the
Which of the following should be considered artery and helps to avoid accidental arte-
during the preparation? rial puncture
A. High-frequency linear transducer with a D. SAX out-of-plane approach allows the
large footprint entire needle to be visualized and hence
B. Low-frequency curvilinear probe with a reducing the risk of arterial puncture
large footprint E. SAX out-of-plane approach allows nee-
C. Hockey stick transducer with a small dle tip to be visualized and can help to
footprint minimize the risk of posterior wall
D. High-frequency linear transducer with a puncture
small footprint 20. Which of the following statement is true with
E. None of the above regard to limitations of US-guided CVL
17. When cannulating the femoral for CVL
insertion
placement, which of the following statement A. Arterial puncture is not possible if CVL
is true with regard to the anatomical relations is inserted under US guidance
of the femoral vein? B. Central line infection rate is higher when
A. Femoral artery is the most medial US is used for CVL insertion
structure C. In obese patients with short neck, the use
B. Femoral vein lies lateral to the femoral of linear probe and long-axis view is nec-
artery essary in order to increase change of suc-
C. Femoral vein lies medial to the femoral cessful puncture
artery D. When using US in CVL insertion, an
D. Femoral vein lies next to the femoral inexperienced user might focus on the
nerve US images and neglect traditional prin-
E. None of the above ciples with regard to needle direction
18. Which of the following options can accu- E. None of the above
rately assess IJV CVL position tip?
I. Chest radiography
II. Vascular ultrasound imaging References
III. Intracavitary electrocardiography
IV. Transthoracic echocardiography 1. Practice Guidelines for Central Venous Access.
A. III An Updated Report by the American Society of
B. I and II Anesthesiologists Task Force on Central Venous
Access. Anesthesiology. 2020;2020(132):8–43.
C. II and IV https://doi.org/10.1097/ALN.0000000000002864.
D. II, III, and IV 2. Bansal R, Agarwal SK, Tiwari SC, Dash SC. A pro-
E. I, II, III, and IV spective randomized study to compare ultrasound-­
19. Which of the following statement is true with guided with nonultrasound-guided double lumen
internal jugular catheter insertion as a temporary
regard to long-axis (LAX) in-plane and hemodialysis access. Ren Fail. 2005;27:561–4.
short-axis (SAX) out-of-plane approach to 3. Cajozzo M, Quintini G, Cocchiera G, Greco G, Vaglica
ultrasound-guided CVL insertion? R, Pezzano G, Barbera V, Modica G. Comparison of
2  Ultrasound-Guided Vascular Cannulation 51

central venous catheterization with and without ultra- 17. Grebenik CR, Boyce A, Sinclair ME, Evans RD,
sound guide. Transfus Apher Sci. 2004;31:199–202. Mason DG, Martin B.  NICE guidelines for central
4. Dolu H, Goksu S, Sahin L, Ozen O, Eken venous catheterization in children: is the evidence
L.  Comparison of an ultrasound-guided technique base sufficient? Br J Anaesth. 2004;92:827–30.
versus a landmark-guided technique for internal 18. Airapetian N, Maizel J, Langelle F, Modeliar SS,
jugular vein cannulation. J Clin Monit Comput. Karakitsos D, Dupont H, Slama M.  Ultrasound-­
2015;29:177–82. guided central venous cannulation is superior to
5. Mallory DL, McGee WT, Shawker TH, Brenner M, quick-look ultrasound and landmark methods among
Bailey KR, Evans RG, Parker MM, Farmer JC, Parillo inexperienced operators: a prospective randomized
JE. Ultrasound guidance improves the success rate of study. Intensive Care Med. 2013;39:1938–44.
internal jugular vein cannulation: a prospective, ran- 19. Turker G, Kaya FN, Gurbet A, Aksu H, Erdogan
domized trial. Chest. 1990;98:157–60. C, Atlas A.  Internal jugular vein cannula-
6. Milling TJ Jr, Rose J, Briggs WM, Birkhahn R, Gaeta tion: an ultrasound-­ guided technique versus a
TJ, Bove JJ, Melniker LA.  Randomized, controlled landmark-­ guided technique. Clinics (Sao Paulo).
clinical trial of point-of-care limited ultrasonography 2009;64:989–92.
assistance of central venous cannulation: the third 20. Zanolla GR, Baldisserotto M, Piva J.  How useful
sonography outcomes assessment program (SOAP-3) is ultrasound guidance for internal jugular venous
trial. Crit Care Med. 2005;33:1764–9. access in children? J Pediatr Surg. 2018;53:789–93.
7. Shrestha BR, Gautam B. Ultrasound versus the land- 21. Sulek CA, Blas ML, Lobato EB. A randomized study
mark technique: a prospective randomized compara- of left versus right internal jugular vein cannulation in
tive study of internal jugular vein cannulation in an adults. J Clin Anesth. 2000;12:142–5.
intensive care unit. JNMA J Nepal Med Assoc. 22. Gualtieri E, Deppe SA, Sipperly ME, Thompson
2011;51:56–61. DR.  Subclavian venous catheterization: greater suc-
8. Slama M, Novara A, Safavian A, Ossart M, Safar cess rate for less experienced operators using ultra-
M, Fagon JY.  Improvement of internal jugular vein sound guidance. Crit Care Med. 1995;23:692–7.
cannulation using an ultrasound-guided technique. 23. Fragou M, Gravvanis A, Dimitriou V, Papalois A,
Intensive Care Med. 1997;23:916–9. Kouraklis G, Karabinis A, Saranteas T, Poularas
9. Teichgräber UK, Benter T, Gebel M, Manns MP.  A J, Papanikolaou J, Davlouros P, Labropoulos N,
sonographically guided technique for central venous Karakitsos D.  Real-time ultrasound-guided subcla-
access. AJR Am J Roentgenol. 1997;169:731–3. vian vein cannulation versus the landmark method in
10. Troianos CA, Jobes DR, Ellison N. Ultrasound-guided critical care patients: a prospective randomized study.
cannulation of the internal jugular vein: a prospective, Crit Care Med. 2011;39:1607–12.
randomized study. Anesth Analg. 1991;72:823–6. 24. Oh AY, Jeon YT, Choi EJ, Ryu JH, Hwang JW,
11. Agarwal A, Singh DK, Singh AP. Ultrasonography: a Park HP, Do SH.  The influence of the direction of
novel approach to central venous cannulation. Indian J-tip on the placement of a subclavian catheter: real
J Crit Care Med. 2009;13:213–6. time ultrasound-­ guided cannulation versus land-
12. Denys BG, Uretsky BF, Reddy PS.  Ultrasound-­ mark method: a randomized controlled trial. BMC
assisted cannulation of the internal jugular vein: a pro- Anesthesiol. 2014;14:11.
spective comparison to the external landmark-guided 25. Aouad MT, Kanazi GE, Abdallah FW, Moukaddem
technique. Circulation. 1993;87:1557–62. FH, Turbay MJ, Obeid MY, Siddik-Sayyid
13. Palepu GB, Deven J, Subrahmanyam M, Mohan SM.  Femoral vein cannulation performed by resi-
S.  Impact of ultrasonography on central venous dents: a comparison between ultrasound-guided and
catheter insertion in intensive care. Indian J Radiol landmark technique in infants and children undergo-
Imaging. 2009;19:191–8. ing cardiac surgery. Anesth Analg. 2010;111:724–8.
14. Verghese ST, McGill WA, Patel RI, Sell JE, Midgley 26. Kusminsky RE.  Complications of central venous
FM, Ruttimann UE. Comparison of three techniques catheterization. J Am Coll Surg. 2007;204(4):681–96.
for internal jugular vein cannulation in infants. 27. Jenssen C, Brkljacic B, Hocke M, Ignee A, Piscaglia
Paediatr Anaesth. 2000;10:505–11. F, Radzina M, Sidhu PS, Dietrich CF.  EFSUMB
15. Verghese ST, McGill WA, Patel RI, Sell JE, Midgley guidelines on interventional ultrasound (INVUS),
FM, Ruttimann UE.  Ultrasound-guided internal part VI  - ultrasound-guided vascular interventions.
jugular venous cannulation in infants: a prospective Ultraschall Med. 2016;37(5):473–6.
comparison with the traditional palpation method. 28. Bhutta ST, Culp WC. Evaluation and management of
Anesthesiology. 1999;91:71–7. central venous access complications. Tech Vasc Interv
16. Karakitsos D, Labropoulos N, De Groot E, Radiol. 2011;14(4):217–24.
Patrianakos AP, Kouraklis G, Poularas J, Samonis 29. Brass P, Hellmich M, Kolodziej L, Schick G,
G, Tsoutsos DA, Konstadoulakis MM, Karabinis Smith AF.  Ultrasound guidance versus anatomical
A. Real-time ultrasound-guided catheterisation of the ­landmarks for internal jugular vein catheterization.
internal jugular vein: a prospective comparison with Cochrane Database Syst Rev. 2015;1(1):CD006962.
the landmark technique in critical care patients. Crit 30. Lamperti M, Bodenham AR, Pittiruti M, Blaivas M,
Care. 2006;10:R162. Augoustides JG, Elbarbary M, Pirotte T, Karakitsos
52 A. Chakraborty et al.

D, Ledonne J, Doniger S, Scoppettuolo G, Feller-­ gency department patients with difficult intravenous
Kopman D, Schummer W, Biffi R, Desruennes E, access. Ann Emerg Med. 1999;34(6):711–4.
Melniker LA, Verghese ST.  International evidence-­ 42. Costantino TG, Parikh AK, Satz WA, et-al.
based recommendations on ultrasound-guided vas- Ultrasonography-guided peripheral intravenous
cular access. Intensive Care Med. 2012;38:1105–17. access versus traditional approaches in patients
31. Dietrich CF, Horn R, Morf S, Chiorean L, Dong Y, with difficult intravenous access. Ann Emerg Med.
Cui XW, Atkinson NS, Jenssen C. Ultrasound-guided 2005;46(5):456–61. https://doi.org/10.1016/j.
central vascular interventions, comments on the annemergmed.2004.12.026.
European Federation of Societies for Ultrasound in 43. Panebianco NL, Fredette JM, Szyld D, Sagalyn
Medicine and Biology guidelines on Interventional EB, Pines JM, Dean AJ. What you see (sonographi-
Ultrasound. J Thorac Dis. 2016;8:E851–68. cally) is what you get: vein and patient characteris-
32. Wilson JG, Berona KM, Stein JC, Wang R. Oblique-­ tics associated with successful ultrasound-guided
axis vs. short-axis view in ultrasound-guided peripheral intravenous placement in patients with
central venous catheterization. J Emerg Med. difficult access. Acad Emerg Med Off J Soc Acad
2014;47:45–50. Emerg Med. 2009;16(12):1298–303. https://doi.
33. Troianos CA, Hartman GS, Glas KE, Skubas NJ, org/10.1111/j.1553-­2712.2009.00520.x.
Eberhardt RT, Walker JD, Reeves ST. Special articles: 44. Gottlieb M, Sundaram T, Holladay D, Nakitende
guidelines for performing ultrasound guided vascu- D.  Ultrasound-guided peripheral intravenous line
lar cannulation: recommendations of the American placement: a narrative review of evidence-based best
Society of Echocardiography and the Society of practices. West J Emerg Med. 2017;18(6):1047–54.
Cardiovascular Anesthesiologists. Anesth Analg. https://doi.org/10.5811/westjem.2017.7.34610.
2012;114:46–72. 45. Lamperti M, Bodenham AR, Pittiruti M, Blaivas M,
34. Aithal G, Muthuswamy G, Latif Z, Bhaskaran V, Haji Augoustides JG, Elbarbary M, et  al. International
Sani HS, Shindhe S, Manap NBA, Vadaje KS, Dato evidence-based recommendations on ultrasound-­
Paduka Buntar WS, Daiwajna RG.  An alternate in-­ guided vascular access. Intensive Care Med.
plane technique of ultrasound-guided internal jugular 2012;38(7):1105–17.
vein cannulation. J Emerg Med. 2019;57(6):852–8. 46. Troianos CA, Hartman GS, Glas KE, Skubas NJ,
35. Blaivas M, Brannam L, Fernandez E. Short-axis ver- Eberhardt RT, Walker JD, et  al. Special articles:
sus long-axis approaches for teaching ultrasound-­ guidelines for performing ultrasound guided vascu-
guided vascular access on a new inanimate model. lar cannulation: recommendations of the American
Acad Emerg Med. 2003;10:1307–11. Society of Echocardiography and the Society of
36. Chittoodan S, Breen D, O’Donnell BD, Iohom Cardiovascular Anesthesiologists. Anesth Analg.
G. Long versus short axis ultrasound guided approach 2012;114(1):46–72.
for internal jugular vein cannulation: a prospec- 47. Costantino TG, Parikh AK, Satz WA, Fojtik
tive randomised controlled trial. Med Ultrason. JP.  Ultrasonography-guided peripheral intravenous
2011;13:21–5. access versus traditional approaches in patients
37. Stone MB, Moon C, Sutijono D, Blaivas M. Needle with difficult intravenous access. Ann Emerg Med.
tip visualization during ultrasound-guided vascular 2005;46(5):456–61.
access: short-axis vs long-axis approach. Am J Emerg 48. Keyes LE, Frazee BW, Snoey ER, Simon BC,
Med. 2010;28:343–7. Christy D.  Ultrasound-guided brachial and basilic
38. Brass P, Hellmich M, Kolodziej L, Schick G, Smith vein cannulation in emergency department patients
AF.  Ultrasound guidance versus anatomical land- with difficult intravenous access. Ann Emerg Med.
marks for subclavian or femoral vein catheterization. 1999;34(6):711–4.
Cochrane Database Syst Rev. 2015;1:Cd011447. 49. Kim E-H, Lee J-H, Song I-K, Kim J-T, Lee W-J, Kim
39. Lalu MM, Fayad A, Ahmed O, Bryson GL, Fergusson H-S.  Posterior tibial artery as an alternative to the
DA, Barron CC, Sullivan P, Thompson C. Ultrasound-­ radial artery for arterial cannulation site in small chil-
guided subclavian vein catheterization: a system- dren: a randomized controlled study. Anesthesiology.
atic review and meta-analysis. Crit Care Med. 2017;127:423–31. https://doi.org/10.1097/
2015;43:1498–507. ALN.0000000000001774.
40. Lamperti M, Biasucci DG, Disma N, Pittiruti M, 50. Hind D, Calvert N, McWilliams R, Davidson A,
Breschan C, Vailati D, Subert M, Traškaitė V, Paisley S, Beverley C, et  al. Ultrasonic locating
Macas A, Estebe JP, Fuzier R, Boselli E, Hopkins devices for central venous cannulation: meta-analysis.
P.  European Society of Anaesthesiology guide- BMJ. 2003;327(7411):361.
lines on peri-operative use of ultrasound-guided 51. Schindler E, Schears GJ, Hall SR, Yamamoto
for vascular access (PERSEUS vascular access). T. Ultrasound for vascular access in pediatric patients.
Eur J Anaesthesiol. 2020;37(5):344–76. https://doi. Paediatr Anaesth. 2012;22(10):1002–7.
org/10.1097/EJA.0000000000001180. Erratum in: 52. Shiloh AL, Savel RH, Paulin LM, Eisen
Eur J Anaesthesiol 2020;37(7):623 LA.  Ultrasound-guided catheterization of the radial
41. Keyes LE, Frazee BW, Snoey ER, et-al. Ultrasound-­ artery: a systematic review and meta-analysis of ran-
guided brachial and basilic vein cannulation in emer- domized controlled trials. Chest. 2011;139(3):524–9.
2  Ultrasound-Guided Vascular Cannulation 53

53. McGee DC, Gould MK.  Preventing complications 67. Stone MB, Hern HG. Inadvertent carotid artery can-
of central venous catheterization. N Engl J Med. nulation during ultrasound guided central venous
2003;348(12):1123–33. catheterization. Ann Emerg Med. 2007;49(5):720.
54. Merrer J, De Jonghe B, Golliot F, Lefrant JY, Raffy 68. Blaivas M, Adhikari S. An unseen danger: frequency
B, Barre E, et  al. Complications of femoral and of posterior vessel wall penetration by needles dur-
subclavian venous catheterization in critically ill ing attempts to place internal jugular vein central
patients: a randomized controlled trial. JAMA. catheters using ultrasound guidance. Crit Care Med.
2001;286(6):700–7. 2009;37(8):2345–9. quiz 2359
55. Wu S, Ling Q, Cao L, Wang J, Xu M, Zeng W. Real-­ 69. Saugel B, Scheeren TWL, Teboul J-L.  Ultrasound-­
time two-dimensional ultrasound guidance for central guided central venous catheter placement: a structured
venous cannulation: a meta-analysis. Anesthesiology. review and recommendations for clinical practice.
2013;118(2):361–75. Crit Care Lond Engl. 2017;21(1):225.
56. Calvert N, Hind D, McWilliams RG, Thomas SM, 70. Mahmood F, Sundar S, Khabbaz K.  Misplacement
Beverley C, Davidson A.  The effectiveness and of a guidewire diagnosed by transesophageal
cost-effectiveness of ultrasound locating devices for echocardiography. J Cardiothorac Vasc Anesth.
central venous access: a systematic review and eco- 2007;21(3):420–1.
nomic evaluation. Health Technol Assess Winch Engl. 71. Hessel EA.  Con: we should not enforce the use of
2003;7(12):1–84. ultrasound as a standard of care for obtaining cen-
57. Fisher NC, Mutimer DJ.  Central venous cannula- tral venous access. J Cardiothorac Vasc Anesth.
tion in patients with liver disease and coagulop- 2009;23(5):725–8.
athy—a prospective audit. Intensive Care Med. 72. Moureau N, Lamperti M, Kelly LJ, Dawson R,
1999;25(5):481–5. Elbarbary M, van Boxtel AJH, et al. Evidence-based
58. Tercan F, Ozkan U, Oguzkurt L.  US-guided place- consensus on the insertion of central venous access
ment of central vein catheters in patients with disor- devices: definition of minimal requirements for train-
ders of hemostasis. Eur J Radiol. 2008;65(2):253–6. ing. Br J Anaesth. 2013;110(3):347–56.
59. Della Vigna P, Monfardini L, Bonomo G, Curigliano 73. Overview | Guidance on the use of ultrasound locat-
G, Agazzi A, Bellomi M, et  al. Coagulation disor- ing devices for placing central venous catheters |
ders in patients with cancer: nontunneled central Guidance | NICE [Internet]. NICE; [cited 2021 May
venous catheter placement with US guidance—a 30]. https://www.nice.org.uk/guidance/ta49
single-institution retrospective analysis. Radiology. 74. O’Grady NP, Alexander M, Burns LA, Dellinger EP,
2009;253(1):249–52. Garland J, Heard SO, Lipsett PA, Masur H, Mermel
60. Hilty WM, Hudson PA, Levitt MA, Hall JB.  Real-­ LA, Pearson ML, Raad II. Guidelines for the preven-
time ultrasound-guided femoral vein catheterization tion of intravascular catheter-related infections. Clin
during cardiopulmonary resuscitation. Ann Emerg Infect Dis. 2011;52(9):e162–93.
Med. 1997;29(3):331–6. discussion 337 75. Marschall J, Mermel LA, Classen D, Arias KM,
61. Weiner MM, Geldard P, Mittnacht AJC. Ultrasound-­ Podgorny K, Anderson DJ, et  al. Strategies to pre-
guided vascular access: a comprehensive review. J vent central line-associated bloodstream infections in
Cardiothorac Vasc Anesth. 2013;27(2):345–60. acute care hospitals. Infect Control Hosp Epidemiol.
62. Gordon AC, Saliken JC, Johns D, Owen R, Gray 2008;29(Suppl 1):S22–30.
RR. US-guided puncture of the internal jugular vein: 76. Eisen LA, Narasimhan M, Berger JS, Mayo PH,
complications and anatomic considerations. J Vasc Rosen MJ, Schneider RF. Mechanical complications
Interv Radiol JVIR. 1998;9(2):333–8. of central venous catheters. J Intensive Care Med.
63. Alderson PJ, Burrows FA, Stemp LI, Holtby HM. Use 2006;21(1):40–6.
of ultrasound to evaluate internal jugular vein anat- 77. Feller-Kopman D.  Ultrasound-guided internal
omy and to facilitate central venous cannulation in jugular access: a proposed standardized approach
paediatric patients. Br J Anaesth. 1993;70(2):145–8. and implications for training and practice. Chest.
64. Parsons AJ, Alfa J. Carotid dissection: a complication 2007;132(1):302–9.
of internal jugular vein cannulation with the use of 78. Bodenham AR.  Editorial II: ultrasound imaging by
ultrasound. Anesth Analg. 2009;109(1):135–6. anaesthetists: training and accreditation issues. Br J
65. Augoustides JG, Horak J, Ochroch AE, Vernick WJ, Anaesth. 2006;96(4):414–7.
Gambone AJ, Weiner J, et  al. A randomized con- 79. Skippen P, Kissoon N. Ultrasound guidance for cen-
trolled clinical trial of real-time needle-guided ultra- tral vascular access in the pediatric emergency depart-
sound for internal jugular venous cannulation in a ment. Pediatr Emerg Care. 2007;23(3):203–7.
large university anesthesia department. J Cardiothorac 80. Chenkin J, Lee S, Huynh T, Bandiera G. Procedures
Vasc Anesth. 2005;19(3):310–5. can be learned on the web: a randomized study
66. Adachi YU, Sato S. Four cases of inadvertent arterial of ultrasound-guided vascular access training.
cannulation despite ultrasound guidance. Am J Emerg Acad Emerg Med Off J Soc Acad Emerg Med.
Med. 2010;28(4):533. 2008;15(10):949–54.
54 A. Chakraborty et al.

81. Troianos CA, Hartman GS, Glas KE, Skubas NJ, 85. Ault MJ, Rosen BT, Ault B. The use of tissue mod-
Eberhardt RT, Walker JD, et  al. Guidelines for els for vascular access training. Phase I of the pro-
performing ultrasound guided vascular cannula- cedural patient safety initiative. J Gen Intern Med.
tion: recommendations of the American Society of 2006;21(5):514–7.
Echocardiography and the Society of Cardiovascular 86. Eason MP, Goodrow MS, Gillespie JE.  A device to
Anesthesiologists. J Am Soc Echocardiogr Off Publ stimulate central venous cannulation in the human
Am Soc Echocardiogr. 2011;24(12):1291–318. patient simulator. Anesthesiology. 2003;99(5):1245–6.
82. Reznick RK, MacRae H.  Teaching surgi- 87. Pérez-Quevedo O, López-Álvarez JM, Limiñana-­
cal skills—changes in the wind. N Engl J Med. Cañal JM, Loro-Ferrer JF.  Design and applica-
2006;355(25):2664–9. tion of model for training ultrasound-guided
83. Bastos EM, Silva RDP.  Proposal of a synthetic vascular cannulation in pediatric patients. Med
ethylene-­vinyl acetate bench model for surgical Intensiva. 2016;40(6):364–70.
foundations learning: suture training. Acta Cir Bras. 88. Kendall JL, Faragher JP.  Ultrasound-guided central
2011;26(2):149–52. venous access: a homemade phantom for simulation.
84. Evans LV, Dodge KL, Shah TD, Kaplan LJ, Siegel CJEM. 2007;9(5):371–3.
MD, Moore CL, et al. Simulation training in central 89. Denadai R, Toledo AP, Bernades DM, Diniz FD, Eid
venous catheter insertion: improved performance in FB, de Moura Lanfranchi LMM, et  al. Simulation-­
clinical practice. Acad Med J Assoc Am Med Coll. based ultrasound-guided central venous cannu-
2010;85(9):1462–9. lation training program1. Acta Cirúrgica Bras.
2014;29:132–44.
Point of Care Ultrasound
of the Airway 3
Deborah Khoo

Abbreviations
(d) Front of neck access—cricothyroidotomy,
percutaneous dilatational tracheostomy
CTM Cricothyroid membrane (PDT).
DLT Double lumen tube
(e) Airway nerve blocks—superior laryngeal
ETT Endotracheal tube nerve block.
FONA Front of neck access (f)
Extubation—leak test, predicting post-­
PDT Percutaneous dilatational tracheostomy extubation stridor, successful extubation.
SGA Supraglottic airway

3.2 Normal Anatomy

3.1 Indications 3.2.1 Ultrasound Appearance


of Airway Structures
Point of Care Ultrasound of the airway is a valu-
able tool that has the following uses: Air conducts ultrasound waves poorly hence the
clinician must bear in mind that structures deep
(a) Pre-intubation screening—especially for into the air-filled structures will not be well visu-
predicted difficult intubations. Used to select alized. The linear ultrasound probe (13–6 MHz) is
endotracheal tube (ETT) size, assess sub- usually used in adult patients. The air within the
mandibular soft tissue, evaluate vocal cord airway also causes reverberation artifacts. The air-
movement and examine airway integrity. mucosa interface is delineated by a bright, hyper-

(b) Tracheal intubation—Confirm ETT place- echoic appearance. The thyroid and cricoid
ment and depth. cartilage appear homogeneously hypoechoic,
(c) Supraglottic Airway (SGA)—to detect laryn- compared to muscle and connective tissue which
geal mask malrotation. are hypoechoic but heterogeneous. As with any-
where else in the body, bony structures (mentum,
mandible rami, hyoid, and sternum) appear hyper-
echoic and bright, with a hypoechoic acoustic
shadow below. Fluid-filled spaces are anechoic
D. Khoo (*) and cast a posterior acoustic enhancement. The
Department of Anaesthesia, National University
Hospital, Singapore, Singapore esophagus is seen as a multilayered structure
e-mail: [email protected] lying to the left of the trachea (Fig. 3.1).

© The Author(s), under exclusive license to Springer Nature Singapore Pte Ltd. 2022 55
A. Chakraborty, B. Ashokka (eds.), A Practical Guide to Point of Care Ultrasound (POCUS),
https://doi.org/10.1007/978-981-16-7687-1_3
56 D. Khoo

Cartilage: Homogenously hypoechoic

Muscle and connective tissue:


Heterogeneously hypoechoic

Air-Mucosa Interface:
Bright hyperechoic line
Carotid
Thyroid gland Artery Esophagus

Reverberation artefact
(posterior acoustic shadowing)

Fig. 3.1  Pattern recognition in airway ultrasound

Fig. 3.2  Transducers commonly used for airway ultrasound

3.2.2 Normal Ultrasound of sufficient ultrasound gel is important to


Appearance of the Airway obtaining a good image. In a spontaneously
breathing patient, the breathing movements
3.2.2.1 Transducer Selection may result in intermittent visualization of the
desired structures. In an awake patient, a
• The medium to high-frequency linear probe is gentle yet steady technique is beneficial to
suitable for visualizing structures close to the the patient’s comfort and cooperation
skin, usually to a depth of less than ~4 cm. (Fig. 3.2).
• The low-frequency curvilinear probe is used
for investigating deeper structures, such as the Normal ultrasound appearances of the airway
tongue and floor of mouth. at various levels are summarized in the table
Given the uneven and relatively unyield- below [1–3] (Table 3.1).
ing nature of the anterior neck surface, usage
3  Point of Care Ultrasound of the Airway 57

Table 3.1  Normal airway anatomy and views


Level of airway Appropriate manner of
that probe beam holding ultrasound to obtain
will be traversing view Normal ultrasound appearance Application
Orophar- A.  A. Tongue (blue
ynx—Sub- arrow): Look for
mentum size, movement,
and lesions
B. Tonsils (Orange
arrows): Size,
abscess. Best seen
by asking patient
to press tongue
Blue arrow shows the B. 
(blue arrow)
direction of ultrasound
against hard palate
probe beam
C. Epiglottis (blue
arrow): Assess for
inflammation and
masses
D. Retropharyngeal
space: Evaluate
C.  for retropharyn-
geal abscess and
Position of curvilinear probe patency of airway
to examine tongue and
submental structures

D. 

Position of probe (slightly


off-­midline) for evaluation
of retropharyngeal space

Larynx— Thin, bright line due


Hyoid to bony nature of
hyoid. (green arrows)
Aids administration of
the superior laryngeal
nerve block

Blue arrow shows the


direction of ultrasound
probe beam

(continued)
58 D. Khoo

Table 3.1 (continued)
Level of airway Appropriate manner of
that probe beam holding ultrasound to obtain
will be traversing view Normal ultrasound appearance Application
Larynx— The thyroid cartilage
Thyroid is pyramidal in shape
cartilage and and is the largest carti-
vocal cords laginous structure in
the airway ultrasound
exam. May be more
prominent in males
and calcified in the
elderly
The vocal cords can
also be seen at this
Transverse view of the airway at the level of
level. Look for vocal
the thyroid cartilage: AC = anterior
cord asymmetry
commissure; PC = posterior commissure;
(indicating vocal cord
VC = vocal cords; SHM = sternohyoid
palsy), lesions, and
muscle; STM = sternothyroid muscle;
cysts
ArC = arytenoid cartilage; TC = thyroid
Extubation—Air
cartilage
column width
difference at the level
of the vocal cords
before and after ETT
cuff deflation helps to
establish the presence
of air leak and
minimize the risk of
post-extubation stridor
Larynx—Cri- The cricothyroid
cothyroid membrane (CTM) is
membrane seen as a bright
hyperechoic line
(green arrow).
Locating and marking
out of the CTM can be
done rapidly and with
more accuracy than by
the palpation method,
especially in patients
who are obese or have
impalpable structures
Pre-procedural or
real-time ultrasound
guidance for
cricothyroidotomy is
also useful for
avoidance of
unintended airway
injury. Look out for
vessels (e.g., thyroid
artery) crossing the
intended puncture site
3  Point of Care Ultrasound of the Airway 59

Table 3.1 (continued)
Level of airway Appropriate manner of
that probe beam holding ultrasound to obtain
will be traversing view Normal ultrasound appearance Application
Larynx—Cri- The airway at the
coid cartilage level of the cricoid
cartilage is the
narrowest part of the
larynx in pediatric
patients. In both
adults and paediatric
patients, measurement
of the diameter of
airway (orange arrow)
can be used to assist
endotracheal tube
(ETT) sizing
according to the
external diameter of
the ETT
Air creates a bright
white air-mucosa
interface with a
posterior acoustic
shadow
The thyroid gland and
thyroid isthmus may
be seen here or at the
level of the tracheal
rings
Trachea Tracheal rings are less
rings—Trans- prominent and less
verse view superficial compared
to the cricoid cartilage
Use of color doppler
at this level is useful
to check for vascular
structures (e.g., a
high-riding
innominate artery)
that may be crossing
the intended site of a
tracheostomy, as well
as to assess the
optimal site and
spacing between
tracheal rings for a
tracheostomy
Suprasternal Correlates to sizing of
notch double lumen tubes.
Blood vessels may
also be seen in this
area

(continued)
60 D. Khoo

Table 3.1 (continued)
Level of airway Appropriate manner of
that probe beam holding ultrasound to obtain
will be traversing view Normal ultrasound appearance Application
Trachea— Longitudinal midline
Longitudinal ultrasound view of the
view trachea
The hypoechoic
tracheal rings give the
classic “string of
pearls” appearance.
The CTM can again
be identified between
the thyroid and cricoid
cartilage

3.3 Airway Devices

3.3.1 Endotracheal Tube (ETT)

Selection of ETT size (according to the outer


diameter) can be performed by measuring the
transverse diameter of the trachea along the
level of the cricoid cartilage to the suprasternal
notch (Fig. 3.3). Correct sizing of the ETT can
help to avoid airway leaks and insufficient ven-
tilation if too small, and pressure necrosis and
subglottic stenosis (Fig. 3.4) if too large, par-
ticularly in the pediatric population. This
Fig. 3.3  Transverse diameter of trachea
method has been shown to be more accurate
than height or weight-­based methods of ETT
sizing [1, 4]. In smaller sized adult patients and
patients with known subglottic stenosis, care-
ful examination of the entire airway to deter-
mine the narrowest point can help avoid
intubation, ventilation, and even extubation
difficulties.
The mid-tracheal level (between the cricoid
cartilage and suprasternal notch) may be identi-
fied in the longitudinal view and is useful in
ensuring appropriate insertion depth placement
of an endotracheal tube. This helps to avoid endo-
bronchial intubation if too deep or accidental
extubation if too shallow [5].
Successful placement of an ETT can be docu-
mented, either in real time or after intubation.
The air column within the ETT will cause poste-
rior acoustic shadowing. The ETT may appear as Fig. 3.4  Airway abrasion and necrosis from a larger than
a rounded structure within the trachea, or a “dou- required ETT, which may lead to subglottic stenosis
3  Point of Care Ultrasound of the Airway 61

ble track” appearance parallel to the air-mucosa airflow. The ratio of the air column width
interface (Figs. 3.5 and 3.6) [6]. (Fig.  3.7) pre-extubation (with cuff deflated) to
the air column width immediately post-intuba-
3.3.1.1 Extubation tion, is a potential predictor of post-­extubation
Post-extubation stridor (PES) is a concern in stridor. A ratio of <0.8 is predictive of post-­
patients who have been intubated for an extended extubation stridor [8, 9]. If the immediate
period, or who may have airway edema for vari- ­post-­intubation air column width is not available,
ous reasons. Cuff-leak tests are widely used to the difference in the air column width with the
predict if the patient may be successfully extu- cuff inflated and deflated may be a possible pre-
bated, and have good specificity, but low sensitiv- dictor, although cut-off values in adults have not
ity, as well as inter-individual variability. This yet been established [10].
means that even if a leak is present, the patient
must be closely monitored for PES post-­
extubation [7, 8]. PES often results in reintuba- 3.3.2 Supraglottic Airway (SGA)
tions, prolonged intubation, and its associated
comorbidities. Supraglottic airways may be visualized in the
Pre-extubation ultrasound can allow for transverse orientation, at the submental level.
assessment of the vocal cords, larynx, and ease of Appearance may differ depending on the make
and model of the SGA, but generally would have
a rounded, spade-like shape. Ultrasound may
help in troubleshooting a mispositioned
SGA.  Ultrasound may also be used to confirm
intraesophageal position of a gastric tube inserted
via the gastric port of second-generation SGAs
(Fig. 3.8).

3.3.3 Double Lumen Endotracheal


Tube (DLT)

Sizing of a left-sided double lumen tube can be


performed by measuring the transverse diameter
Fig. 3.5  Rounded ETT appearing in airway (orange
arrow) of the trachea at the level of the suprasternal

Fig. 3.6  Double track lines (blue arrows) can also help to confirm endotracheal ETT placement
62 D. Khoo

notch [11]. This is particularly useful in smaller


sized patients or patients who have had previous
airway surgery or injury. The double lumen tube
exhibits the same “double track” sign as a single
lumen ETT, but with slight midline depression in
transverse and parallel lines in the longitudinal
orientation (Fig. 3.9).

Fig. 3.7  Air column width (dotted yellow line) in an intu-


bated patient with cuff deflated

a b

Fig. 3.8 (a) Central, well-positioned SGA (blue arrow), seen in the transverse view at the submentum level. (b) SGA
tip folded and off-midline (orange arrow)

Fig. 3.9  Ultrasound appearance of a double lumen tube (blue arrows) in the trachea
3  Point of Care Ultrasound of the Airway 63

Transverse approach Paramedian approach

Direction of
Direction of needle needle entry
Thyrohyoid
membrane
entry and spread of
Hyoid bone local anaesthetic
Thyrohyoid
muscle

Superior Thyroid
laryngeal cartilage
nerve Superior
Superior
laryngeal
Greater cornu nerve laryngeal

of hyoid Superior Hyoid artery

laryngeal shadow
artery

Fig. 3.10  Ultrasound visualization and techniques for superior laryngeal nerve blocks

3.4 Airway Nerve Block • Identify and mark out the CTM—in prepara-
tion for emergency front of neck access if it
The superior laryngeal nerve (SLN) block is a use- becomes necessary. Recommended to do
ful adjunct to facilitate awake intubations or rigid before undertaking potentially difficult air-
bronchoscopy. The internal branch of the SLN ways such as awake intubations, trauma, or
runs beneath or lateral to the greater horn of the obese airways [1, 14, 15].
hyoid, before penetrating the thyrohyoid mem- • Identify the location of airway structures in
brane to provide innervation to the larynx. patients who may have tracheal deviation or lim-
Ultrasound-guided superior laryngeal nerve blocks ited neck mobility, e.g., previous radiotherapy or
allow for target visualization, as well as ensuring surgery to the neck, cervical spine contractures,
spread of local anesthetic and avoidance of vascu- neck masses, and thoracic and spine deformities.
lar (e.g., the superior laryngeal artery) and other • Right-sizing of airway device and selection of
anterior neck structures (Fig. 3.10) [12, 13]. a safe tracheal interspace or location of CTM
for front of neck access.

3.5 Clinical Pearls Stepwise approach to focused rapid ultra-


and Applications sound of the upper airway.

3.5.1 Focused Rapid Ultrasound 1. Identify the mentum and sternal notch as the
Review of the Upper Airway cephalad and caudal ends of the airway,
respectively.
3.5.1.1 Goals 2. Holding the ultrasound probe in a transverse
manner, begin from the mentum and move the
• To inform of obstructions to securing the air- ultrasound probe in a caudal direction.
way in potentially difficult airways. 3. Identify the thin, bright line of the hyoid bone,
• To check for injuries to the airway, especially looking for continuity of the bone and pres-
in patients who have sustained trauma. ence of soft tissue swelling.
64 D. Khoo

4. Move the probe in a caudal direction and 9. To obtain the longitudinal midline view of the
identify the triangular thyroid cartilage and trachea, slide the ultrasound probe laterally
vocal folds. In a spontaneously breathing until medial edge of the probe is positioned
patient, look for symmetry of vocal cord such that only half of the trachea is in view at
shape and movement. If trauma is suspected, the edge of the ultrasound screen. Rotate the
look for discontinuity on the external surface ultrasound probe so that the lateral edge is
of the cartilage as well as soft tissue now cephalad in the sagittal plane. Identify
swelling. the thyroid and cricoid cartilages, and the

5. Sliding the probe caudally, identify the CTM in between. Mark out the CTM as well
bright white line that denotes the air-mucosa as the midline of the airway.
interface of the cricothyroid membrane
(CTM).
6. Confirm the position of the CTM by continu- 3.5.2 Potential Ultrasound
ing to slide the probe caudally in a transverse Predictors of Laryngoscopy
orientation, seeing the change to the homoge-
neously hypoechoic cricoid cartilage. Airway ultrasound is being increasingly used to
7. Return the probe to the level of the CTM and identify potentially difficult airways, alongside
mark its position, as well as that of the mid- clinical examination and patient history [2]. It is
line, on the patient. important to note that no single criteria predict
8. Continue to scan caudally, observing the airway difficulty in totality, and greater accuracy
change from the cricoid cartilage to the tra- can be achieved with combination of predictors
cheal rings until it reaches the suprasternal and tailoring to the patient’s history and anatomy.
notch. Observe for pulsatile arterial structures Figure 3.11 and Table 3.2 summarize the param-
in this area. eters [4, 16–19].

Fig. 3.11  Levels at


which measurement of
airway parameters may
predict difficult
laryngoscopy. TT tongue
thickness, MH
mentohyoid distance,
SH skin to hyoid Nasal
distance, SE skin to passage
epiglottis distance, SVC
skin to anterior Oral passage
Nasopharynx
commissure of vocal
cords distance, ST skin Tongue
to thyroid isthmus Oropharynx TT
distance, SSN skin to
Epiglottis
suprasternal notch MH Mentum
distance Larynx Hyoid bone
SH
Glottis and vocal folds
SE
Subglottic space
SVC
Esophagus Thyroid cartilage
Trachea Cricothyroid membrane
ST
Cricoid cartilage
Tracheal rings
SSN

Sternum
3  Point of Care Ultrasound of the Airway 65

Table 3.2  Summary of ultrasound airway parameters that may predict a difficult airway
Tongue width:
How to obtain image: Linear probe
in transverse orientation at
submental level
Interpretation: A width of >28 mm
corresponds to a higher probability
of obstructive sleep apnoea and
difficult laryngoscopy

Mentohyoid distance and tongue


thickness:
How to obtain image: Curvilinear
probe in sagittal orientation between
mentum and hyoid
Mentohyoid (MH) distance:
Interpretation:
 1. An MH distance of <52 ± 6 mm
indicates a possible difficult
airway, while >65 ± 4 mm
indicates easy intubations
 2. Calculate the ratio of the MH
distance with the patient’s head in
hyperextension to that in neutral
position. A ratio of <1.1 is
predictive of difficult laryngos-
copy and intubation
These measurements also give an
indication of the degree of cervical
spine mobility
Tongue thickness (TT):
Interpretation: Measure the maximal
thickness of the tongue between the
floor of mouth and hard palate. A
distance of more than 60 mm is
indicative of macroglossia and may
be suggestive of obstructive sleep
apnoea (OSA)
Skin to hyoid (SH) distance:
How to obtain image: Linear probe
in transverse orientation at level of
hyoid bone
Interpretation: An SH distance of
>16.9 mm may predict a difficult
airway

(continued)
66 D. Khoo

Table 3.2 (continued)
Skin to epiglottis (SE) distance:
How to obtain image: Linear probe
in transverse orientation just below
level of hyoid bone
Interpretation: An SE distance of
>24 mm may indicate a difficult
airway

Skin to anterior commissure of


vocal cords (SVC) distance:
How to obtain image: Linear probe
in transverse orientation at level of
thyroid cartilage
Interpretation: An SVC distance of
>28 mm may indicate a difficult
airway

Skin to thyroid isthmus (ST)


distance:
How to obtain image: Linear probe
in transverse orientation at level of
the cricoid, looking for the point
where the thyroid gland isthmus is
maximally seen
Interpretation: An ST distance of
>34.7 mm may indicate a difficult
airway

Skin to suprasternal notch


distance:
How to obtain image: Linear probe
in transverse orientation at level of
the suprasternal notch
Interpretation: Not a proven
predictor, but may be useful as a
screening tool to predict a difficult
airway [20, 21]

Blue arrows show skin to SSN distance in obese vs normal patients in


longitudinal orientation
3  Point of Care Ultrasound of the Airway 67

3.6 Summary  CQs: Chose the Single Best


M
Answer
Ultrasound of the airway is a very useful skill to
incorporate into airway evaluation and manage- (Read the questions with a book or a paper cover-
ment. While not yet standard of care, it is gaining ing the answer side, try to answer on your own,
increasing acceptance and use in both adults and and then check with the answer)
pediatric patients [4, 22]. Consistent practice on a
wide range of patients will enable the user to rec-
ognize common and abnormal morphology, as
well as to add to current knowledge and practice.

Questions Answers
1. A pre-scan prior to performance of a cricothy- Answer: A. The cricothyroid membrane (CTM) appears as a
roidotomy reveals the following structures bright white line due to the air-mucosa interface, with a
   Which of the following accurately labels reverberation artifact posteriorly. The CTM is 13–15 mm
structures 1, 2, and 3? long in adults. In the transverse orientation, scanning from
the thyroid cartilage, then to the CTM, followed by the
cricoid, and finally back to the CTM helps to visualize the
transition and ascertain the identity of the CTM [23]

1 2 3
A Cricothyroid Reverberation Sternohyoid
membrane artifact muscle
B Cricoid Reverberation Thyroid gland
cartilage artifact
C Thyroid Endotracheal Sternocleido-
cartilage tube mastoid
muscle
D Cricothyroid Endotracheal Sternothyroid
membrane tube muscle
2. A patient who has sustained a penetrating head Answer: D. Subcutaneous emphysema indicates discontinu-
and neck trauma develops difficulty breathing ity in the airway or aerodigestive tract. Nonintervention may
and is desaturating. Ultrasound of his airway miss the window of opportunity for management before the
reveals subcutaneous emphysema. Appropriate patient becomes more unstable. Positive pressure ventilation
management includes should be minimized until the airway is secured. Careful
 A. Monitor the patient closely but do not ultrasound examination of the airway is useful to rapidly
manipulate the airway at this point as it may delineate distorted anatomy and formulate the safest location
worsen the situation to secure the patient’s airway, whether by intubation or front
 B. Support the patient with bag-mask ventila- of neck access. Bronchoscopy evaluation is indicated as well
tion where possible. Cervical spine immobility is likely to add to
 C. Intubate the patient via the oral or nasal route the complexity
 D. Identify the level of injury and proceed to
secure the airway
68 D. Khoo

Questions Answers
3. After a difficult intubation, this image is seen on Answer: B. This ultrasound image shows an oesophageal
ultrasound of the airway. Appropriate immediate intubation. The air column in the ETT produces a “drop-out”
management should include: dark area below the air-ETT interface. Stylets or bougies may
produce the same, albeit smaller, effect. Monitoring of an
intubation in real time using ultrasound allows for immediate
identification of esophageal intubation

  A Ventilate the patient with appropriate tidal


volumes
 B Remove and reinsert the endotracheal tube
(ETT)
 C Check for ETCO2 using a colorimeter or
capnograph
 D Deepen the patient using inhalational or
intravenous agents
4. Prior to performing a percutaneous dilatational Answer: C. The innominate artery is also known as the
tracheostomy (PDT), a vascular structure is seen brachiocephalic artery. It is the most proximal branch of the
at the level of the fourth tracheal ring. This aortic arch, originating medial and anterior to the left
vascular structure is likely to be the: common carotid artery. This short artery courses superiorly
 A. Thyroid artery posterior to the aortic arch, bifurcating into the right
 B. Superior laryngeal artery subclavian artery and right common carotid artery at the level
 C. Innominate artery of the sternoclavicular joint. It crosses from left to right
 D. Tonsillar artery anterior to the trachea, usually around the ninth tracheal ring.
However, it has a very variable location, and can sometimes
be observed at or above the suprasternal notch as a pulsatile
structure
Avoidance of this artery is paramount as cases of fatal
bleeding, or trachea-innominate artery fistulas post tracheos-
tomy, have been described. It should be located prior to both
surgical and percutaneous dilatational tracheostomies
3  Point of Care Ultrasound of the Airway 69

Questions Answers
5. In a preoperative assessment for an elective Answer: C. There is a vocal cord cyst at the anterior
surgery, you note that your patient has a hoarse commissure. On the ultrasound, posterior acoustic enhance-
voice. Ultrasound examination reveals the ment of the area below the fluid cyst hints at its nature.
following Careful documentation and preoperative evaluation, as well
as counselling of patient for potential worsening of voice
hoarseness is indicated. The vocal cords are abducted hence
vocal cord paralysis is unlikely. Depending on how
time-sensitive the surgery is, investigation into the cause of
the patient’s hoarse voice is useful but should not prohibit
urgent surgery from proceeding

Appropriate management includes:


 A. Continue with surgery with careful
intubation technique
 B. Assure patient and document voice
hoarseness before proceeding with surgery
 C. Referral to otolaryngologist for further
evaluation
 D. Investigate the patient for left recurrent
laryngeal nerve palsy
6. A patient is noted to have a left-sided neck Answer: A. Thyroglossal duct cysts are the most common
fullness just below his chin. Bedside ultrasound congenital neck mass. They are thin-walled, fluid-filled,
revealed the following: usually close to the midline, anterior to the thyroid cartilage,
and splaying the strap muscles. They are generally close to
the hyoid bone and can be traced to a tail diving under the
hyoid. On ultrasound, they appear anechoic, with posterior
acoustic enhancement. Stranding within the cyst or thickened
walls may indicate inflammation. Thyroid tissue would
exhibit a homogenously hypoechoic appearance but can be
found in very variable locations. A vocal cord cyst would
appear posterior to the thyroid cartilage and branchial cleft
cysts tend to be located more laterally. Ultrasound is a useful
The lesion shown is most likely to be: initial investigation modality due to its noninvasiveness,
 A. A thyroglossal duct cyst accessibility, relatively low cost, and avoidance of exposure
 B. Ectopic thyroid tissue to radiation
 C. A vocal cord cyst
 D. A branchial cleft cyst
7. Ultrasound features that correlate to a higher Answer: D. Together with patient history and physical
likelihood of obstructive sleep apnoea (OSA) examination, airway ultrasound can help improve the
include all EXCEPT: accuracy of OSA screening. Airway ultrasound features
  A. Tongue thickness > 60 mm include the tongue thickness and width, as well as thickening
  B. Tongue width > 28 mm of the lateral pharyngeal wall. Non-airway parameters
 C. Thickened lateral pharyngeal walls include carotid plaque formation and carotid intimal
  D. Mentohyoid distance < 52 mm thickening as surrogate markers [24]
70 D. Khoo

Questions Answers
8. In a patient with known right recurrent Answer: C. The recurrent laryngeal nerve (RLN) branches
laryngeal nerve palsy, you would expect to see from the vagus nerve and supplies all the intrinsic muscles of
the following on ultrasound in the transverse the larynx, except the cricothyroid muscles. Complete paraly-
orientation at the level of the thyroid cartilage sis of the RLN results in a partially adducted vocal cord,
during inspiration: while partial paralysis results in a fully adducted vocal cord
Right vocal Left vocal cord
cord
A Abducted Abducted
B Abducted Adducted
C Adducted Abducted
D Adducted Adducted
9. A victim of a road traffic accident where the Answer: D. This unfortunate patient will have all the
vehicle caught fire was brought into the attendant issues of trauma and smoke inhalation, not to
emergency department with severe burns to his mention burns and injuries to his face and upper airway
face, although his neck is relatively unscathed.
The potential airway issues include:
 A. Lower airway edema
 B. Potential cervical spine instability
 C. Upper airway injury
 D. All of the above
10. Advantages of a cricothyroidotomy over a Answer: D. In an emergency situation, a cricothyroidotomy
tracheostomy in an emergency front of neck offers the fastest and least complicated means of front of
access include: neck airway access. However, it is thought to be associated
 A. Relatively avascular structure with an increased risk of subglottic stenosis, and hence
 B. More superficial and palpable cricothyroid creation of a formal tracheostomy is often performed within
membrane compared to the deeper tracheal 72 h of an emergency cricothyroidotomy
rings
 C. Shorter insertion time required
 D. All of the above
11. Postsurgical tracheostomy creation, a ward Answer: A. Fluid, such as blood, appears anechoic on
nurse notices swelling around the tracheos- ultrasound and will cast posterior acoustic enhancement
tomy site. Ultrasound evaluation reveals (bright area). Air will cause posterior shadowing (dark area).
anechoic areas in the subcutaneous and Ultrasound of the airway is a useful bedside examination that
peri-muscular planes is portable, can be done rapidly and noninvasively, and gives
The cause of the swelling is likely to be: immediate information which can be critical to preventing
 A. Blood adverse patient outcomes
 B. Subcutaneous emphysema
 C. Soft tissue swelling
 D. An overly tight tracheostomy strap
12. Structures that may overlie the cricothyroid Answer: C. All the other structures may be found in variable
membrane (CTM) DO NOT include: positions overlying cricothyroid membrane and care should
 A. The cricothyroid artery and vein (in the be taken in the event a cricothyroidotomy needs to be
upper 1/3) performed
 B. The pyramidal lobe of the thyroid gland
 C. Recurrent laryngeal nerve
 D. Lymph nodes
3  Point of Care Ultrasound of the Airway 71

Questions Answers
13. Differences between the ultrasound appear- Answer: B. The narrowest part of the pediatric airway is at
ance of the pediatric airway compared to that the level of the cricoid cartilage, and airway measurement
of an adult include all EXCEPT: should take place here. Ultrasound sizing of the airway is
 A. A cartilaginous and hypoechoic hyoid bone more accurate compared to age-based and height-based
 B. Measurement of the tracheal diameter for formulas in estimating ETT size (outer diameter) for
endotracheal tube sizing at the level of the pediatric patients [25, 26]
thyroid cartilage
 C. Thinner and less echogenic cricothyroid
membrane
 D. An elliptical rather than circular subglottic
airway
14. A 12-year-old child with cerebral palsy and Answer: B. This patient is an airway challenge because of his
severe scoliosis requires a spine operation to limited range of motion. Ultrasound will be difficult because
improve his respiratory function. He has very of his inability to extend his neck, and such patients are
limited cervical extension. Which of the generally small for their age. Ultrasound is a reliable tool to
following ultrasound techniques may help in predict ETT size for pediatric patients with thoracic or
this situation? lumbar scoliosis. However, pediatric patients with cervical
lateral bending will need an ETT smaller than the size
predicted by ultrasonography [27]. Real-time imaging of the
airway during intubation can also help to ensure that the ETT
is placed at an appropriate depth, but this requires a second
operator

  A. Assessment of the tracheal diameter for


endotracheal tube (ETT) sizing at the level
of the sternal notch
 B. Use of ultrasound to perform a superior
laryngeal nerve block for an awake
intubation
 C. Use of a curvilinear probe to assess the
mentohyoid distance
 D. Use of a small “hockey stick probe” to
identify the cricothyroid membrane
15. Ultrasound features of an intubated airway Answer: A. The air column within the endotracheal tube
include all except: causes a linear posterior shadow within the airway. Radial
 A. Reverberation artifacts in the airway comet-tail artifacts may also be seen. Double track lines
 B. Double track lines in the sagittal plane indicate the presence of a tube within the airway
 C. Posterior shadow in the airway
 D. Comet-tail artifacts in the airway
72 D. Khoo

Questions Answers
16. A 45-year-old patient attempted suicide by Answer C: The patient likely has upper airway edema and is
drinking a large volume of corrosive fluid and agitated from a combination of intoxication and hypoxia. He
alcohol. He presents to the emergency is in respiratory distress and his airway needs to be secured.
department. He is extremely agitated, trashing, He is likely to be difficult to manage without sedation but
and has pulled out his intravenous cannulas. should be kept spontaneously ventilating where possible. He
His breathing is stridorous and his pulse will not be able to cooperate with an awake intubation
oximetry reading is 88%. What is the most Rapid focused ultrasound assessment of the airway should be
reasonable plan of action? done to identify the cricothyroid membrane as well as to
evaluate airway edema and soft tissue swelling. Serial
ultrasounds can also be used to monitor progress

  A. Avoid sedation, attempt awake broncho-


scopic intubation
 B. Avoid sedation, perform percutaneous
dilatational tracheostomy
 C. Sedate with IM or inhalational agents,
perform cricothyroidotomy
 D. Sedate with IM or inhalational agents,
paralyze patient, and attempt nasal or oral
intubation
17. Which of the following is true about ultra- Answer: A
sound appearance of airway structures? The air-mucosa interface, such as that at the cricothyroid
 A. Cartilage appears homogenously membrane, appears as a bright line because of the significant
hypoechoic difference between soft tissue and air, resulting in a strong
 B. The cricothyroid membrane appears as a reflection of soundwaves
bright line because it is relatively solid Reverberation artifacts are generated due to the reflective
compared to cartilage properties of the air-mucosa interface and appear as regularly
 C. Reverberation artifacts appear as hyper- spaced parallel hyperechoic lines deep to the air-mucosa
echoic radial lines interface
 D. Air causes posterior acoustic enhancement Air columns, such as that within an endotracheal tube, will
cast an acoustic shadow, obscuring structures deep into it
3  Point of Care Ultrasound of the Airway 73

Questions Answers
18. A patient presents with the following lesion on Answer: A. This patient has a large retropharyngeal abscess.
CT scan and is being brought to the operating Airway anatomy may be distorted, and ultrasound can help
theatre for intubation and emergency surgery especially with ensuring accurate endotracheal tube sizing, as
well as with the identification of critical airway structures

During a pre-intubation ultrasound scan of the


patient’s airway, which area do you expect to see
the lesion in?

19. The best level to measure the airway diameter Answer: C. The suprasternal notch provides an easily
to select a left-sided double lumen endotra- visualized and measured point of the airway for the purposes
cheal tube size is at the: of sizing of a double lumen endotracheal tube, according to
 A. Hyoid its outer diameter
 B. Thyroid cartilage
 C. Suprasternal notch
 D. Cricoid cartilage
20. Which of the following ultrasound features Answer: D. The skin to tracheal distance at the suprasternal
may predict a difficult airway? notch is not proven to be a marker of airway difficulty,
 A. Skin to tracheal distance of 23 mm at the although a larger distance would probably correspond to a
suprasternal notch prominent chest which may interfere with the handle of a
 B. Tongue width of 27 mm standard laryngoscope
 C. Skin to epiglottis distance of 22 mm It is important to remember that no single parameter can
 D. Mentohyoid distance of 42 mm predict airway difficulty well. Multiple ultrasound airway
parameters, together with clinical assessment, can help to
increase the rate of identification of difficult airways and
enable management by informing the clinician of the level at
which they are most likely to encounter difficulty
74 D. Khoo

Clinical exam scenarios Answers


A young man was brought into ED after a bar 1.  What are your airway concerns for this patient?
fight. He is intoxicated and unable to give a   • Cervical spine injury
consistent history, but you note some mild   • Airway patency and continuity
bruising and swelling over his neck. He has no   • Soft tissue swelling
other injuries 2.  Likely diagnosis?
His vital signs are as follows:   • Cricoid cartilage injury, indicated by the loss of
 Blood pressure: 114/74 mmHg continuity and depression of the cricoid cartilage
 Heart rate: 90 beats/min   • There may be accompanying soft tissue swelling and
 Pulse oximetry: 94% on room air edema
Physical examination: No noisy breathing/stridor,   • Likely cause in this patient is blunt laryngeal trauma
not cyanotic, not in respiratory distress 3. Management?
1. What are your airway concerns for this patient?   Laryngeal trauma can be categorized as penetrating or
2. Ultrasound reveals the following. There are no blunt laryngeal trauma. Management can be guided by the
other abnormalities. What is the likely Schaefer Classification System of Severity of Laryngeal
diagnosis? Injuries in the table below [28, 29]
Severity Management
Group 1: Minor endol- Conservative; Consider medical
aryngeal hematomas or adjunctive management such as
lacerations without steroids, antibiotics, anti-reflux
detectable fractures medication, humidification, and
voice rest
Group 2: More severe Conservative initially but with
edema, hematoma, minor serial examination as injuries
mucosal disruption may progress over time. Medical
without exposed cartilage, adjuncts as for Group 1
or nondisplaced fractures
Group 3: Massive edema, Direct laryngoscopy or
3. How would you manage this patient? large mucosal lacerations, esophagoscopy should be
exposed cartilage, performed in the operating room.
displaced fractures, or Tracheostomy, surgical
vocal cord immobility exploration, and repair are often
required
Group 4: Same as group Direct laryngoscopy and
3, but more severe, with esophagoscopy must be
disruption of anterior performed emergently.
larynx, unstable fractures, Tracheostomy mandated and
two or more fractures surgical repair may involve stent
lines, or severe mucosal placement
injuries
Group 5: Complete Patient will present with severe
laryngotracheal separation respiratory distress, necessitating
urgent airway evaluation and
management. Altered anatomy
may make airway management
difficult. Close communication
and joint management between
the surgical and anesthetic teams
are critical
Schaefer Classification System of Severity of Laryngeal
Injuries and Suggested Management
3  Point of Care Ultrasound of the Airway 75

Clinical exam scenarios Answers


An unknown elderly male was knocked down by a 1. What are your treatment priorities in this patient?
motorcycle while crossing the road. He sustained    This patient has sustained polytrauma, and it is
lower limb fractures and focused assessment with imperative to rapidly secure his airway, oxygenate him and
sonography in trauma (FAST) shows free fluid in improve his hemodynamics with fluid, blood, and pressor
his abdomen. He is hypotensive and tachycardic, support
and pulse oximetry is 86% on bag-mask 2. Ultrasound of airway—findings and implications
ventilation with 100% oxygen. He is noted to have    The airway of this patient appears to terminate at the
a tracheostomy, which was dislodged in the mid-tracheal level. This implies that the patient likely had a
accident, as well as bilateral neck scars. Medical total laryngectomy and that the upper airway is not
records are unavailable as he was not carrying any continuous with the lower airway. Hence, oral intubation
identity documents will not be possible
1. What are your treatment priorities in this 3.  Plan of management
patient?    Careful but rapid exploration of the tracheostomy site
2. Bag-mask ventilation does not seem to be can be performed. A bougie or flexible suction catheter may
improving the patient’s oxygenation, and he be gently passed into the tracheotomy track. Real-time
has become unconscious. Attempts at ultrasound monitoring can help to ascertain intratracheal
recanalization of the tracheostomy have failed, placement and avoid creating a false passage. A tracheos-
and the clinical team decides to intubate orally. tomy tube or armored endotracheal tube can then be
Suspecting a difficult airway, an airway railroaded into the trachea, and ultrasound is used to
ultrasound is performed and reveal the confirm placement, as well as the depth of insertion
following
   What is the implication of this ultrasound
finding?

3. What is the most reasonable plan of manage-


ment to secure the patient’s airway?
A patient was intubated 6 days ago for airway 1. How would you assess the patient’s fitness for extuba-
protection after an episode of torrential epistaxis, tion?
which has now resolved. The patient is a 40 kg    Acronym: MOVEC
elderly female of small build. A size 7.5 ETT was   • Mental status—GCS >8 (with some rare exceptions)
used. She is now ready to be extubated, but the   • Oxygenation—The patient should be able to maintain
clinical team is encountering resistance when adequate oxygenation with low levels of support. The
attempts are made to withdraw the ETT. The PaO2/FiO2 ratio should be more than 150, and FiO2 < 0.4
patient is otherwise well and breathing spontane-   • Ventilation—Positive End Expiratory Pressure should be
ously on FiO2 0.28. They request for the <10 cmH2O, and minute ventilation requirements <15 L/min
anesthetic team’s assistance in attempting   • Expectoration—Secretions should not be excessive or
extubation in the operating theatre cause mucous plugging
1. How would you assess the patient’s fitness for   • Cardiovascular—instability should not be present
extubation? 2. What are the risk factors for developing acquired
2. What are the risk factors for developing subglottic stenosis?
subglottic stenosis?   • Paediatric—Prolonged intubation, low birth weight [30]
3. What ultrasound airway parameters are useful   • Adults—High body mass index, diabetes [31], prolonged
in the evaluation of this patient for extubation? intubation
3. What ultrasound parameters are useful in evaluation of
this patient?
  • Air column width during cuff deflation at the level of the
cricothyroid membrane is a predictor of post-extubation
stridor (PES) [32]
  • Evaluation of vocal cord function and airway edema
  • Ultrasound of the lung and diaphragm can also help to
predict readiness for extubation
76 D. Khoo

Clinical exam scenarios Answers


A 6-year-old child presents to the emergency 1. How would you assess patency of this child’s airway?
department with a sorethroat, drooling, and   • Rapid assessment of patient’s airway, breathing, and
stridor. He is distressed and anxious circulation
1. How would you assess patency of this child’s   • Airway—accessory muscle use, tracheal tug, stridor
airway?   • Breathing—respiratory rate, breathing effort, and ability
2. How can ultrasound of the airway help? to move air in and out of lungs
3. What are the differential diagnoses?   • Circulation—presence of cyanosis, hypoxia, heart rate,
4. Ultrasound of the airway in the sagittal section consciousness
using a curvilinear probe at the submental level 2. How can ultrasound of the airway help?
shows the following appearance. What is the   • Assess patency of airway
most likely diagnosis?   • Identify cricothyroid membrane in the event emergency
front of neck access is needed
  • Right-sizing of an endotracheal tube or tracheostomy
tube
  • Evaluate potential causes for patient’s symptoms
  • Noninvasive, bedside test that can be done with parental
presence to encourage child’s cooperation
3. Differential diagnoses?
  • Infective—epiglottitis, viral croup, bacterial tracheitis
  • Allergy—anaphylaxis, angioedema
  • Foreign body inhalation
  • Trauma
  • Inhalation of corrosive or irritative gas
4. Ultrasound appearance of epiglottitis
  • Using a curvilinear probe in the longitudinal orientation
and at the level of the hyoid bone, the epiglottis appears
as the head of the “alphabet P,” and the acoustic shadow
cast by the hyoid bone forms the stem of the letter P. A
swollen and oedematous epiglottis is indicative of
epiglottis [33]
  • Evaluation can also be made in the transverse orienta-
tion, just below the hyoid, seeing the hypoechoic
cartilaginous epiglottis in the middle of the tongue [34]
3  Point of Care Ultrasound of the Airway 77

Clinical exam scenarios Answers


A 35-year-old male patient presents to the emer- 1. Initial goals of management
gency department with a penetrating neck wound   • Airway—maintain airway patency
across his neck. The patient is sitting upright.   • Breathing—maintain spontaneous ventilation and
Supplemental oxygen is being administered via a supplemental oxygen
non-rebreather mask, and oxygen saturation is   • Circulation—ensure good vascular access and
85–88%. His blood pressure is 95/50 mmHg and administer fluids, blood, and vasopressors as needed
heart rate is 102 beats per minute. The subcutaneous   • Disability—assess for other injuries
tissue below the mandible and oropharynx is grossly 2. How would you secure this patient’s airway?
traumatized and bleeding heavily. The patient is    This is an airway fraught with difficulties. There is
conscious and able to breath, but there is a bubbling likely trauma to the trachea itself, along with blood that
sound in his neck area with each breath will obscure visualization and soft tissue swelling which
1. What are your initial goals of airway will worsen
management in this patient?    The airway needs to be secured together with surgical
2. How would you secure this patient’s airway? expertise, and the management plan would need to be
3. How would you utilize ultrasound in this formulated according to the suspected level of injury.
scenario? The patient may need to be sedated for this, but
spontaneous ventilation should be maintained as far as
possible. Avoid a cannot intubate, cannot ventilate
situation
3. Use of ultrasound
   Combination of airway and lung ultrasound can help to
evaluate the success of endotracheal tube (ETT)
insertion, whether by the oral, nasal, or front of neck
access route. Tracheal ultrasound (either in real time or
immediately after intubation) can exclude the presence of
the ETT in the esophagus and possibly visualize the ETT
in the trachea. Confirm the presence of lung sliding on
the left, the absence of which may indicate right
endobronchial intubation. Finally, confirm the presence
of lung sliding on the right [35]
A 48-year-old patient, with a body mass index of 1. What are the ultrasound features that may suggest that
51 kg/m2 requires intubation with a double lumen the patient has obstructive sleep apnea (OSA)?
endotracheal tube for surgical removal of a right  •  Tongue width (between the lingual arteries) > 28 mm
lung mass. She has limited neck extension, a small  •  Retro-lingual space <37 mm and retropalatal space
mouth opening, and has hypertension and <30 mm
diabetes. She is unsure if she has symptoms of  • Distance between the lateral edge of the pharynx and
obstructive sleep apnea carotid artery, usually 30–50 mm. A greater distance
1. What are the ultrasound features that may correlates with greater severity of OSA [4, 24, 36]
suggest that the patient has obstructive sleep 2. Optimizing airway ultrasound examination
apnoea?    Ultrasound in morbidly obese patients may be challeng-
2. What may help make to optimize the ing, especially with limited neck extension. Placing the
performance of an airway ultrasound patient’s upper body in a ramped position may help to
examination in this patient? increase access to the patient’s neck, as well as improve
3. If double lumen tube intubation is not ventilation for the intubation process. Sufficient
possible, what are the alternatives to achieve ultrasound gel is also essentialThe cricothyroid
lung isolation? membrane (CTM) is often impalpable in morbidly obese
patients. Ultrasound localization of the CTM can be
performed faster than palpation. Tube sizing and
real-time monitoring of intubation help to avoid airway
injury [37]
3. If double lumen tube intubation is not possible, what
are the alternatives to achieve lung isolation?
   Insertion of a single lumen ETT, followed by the use of
a bronchial blocker is one method. Alternatively, the
single lumen ETT can be advanced to the left bronchus
as an intentional endobronchial intubation
78 D. Khoo

References membrane: a randomized comparison with the pal-


pation method in the morbidly obese. Br J Anaesth.
2015;114(6):1003–4.
1. Gottlieb M, Holladay D, Burns KM, Nakitende D,
16. Osman A, Sum KM.  Role of upper airway ultra-
Bailitz J.  Ultrasound for airway management: An
sound in airway management. J Intensive Care.
evidence-­based review for the emergency clinician.
2016;4(1):52.
Am J Emerg Med. 2020;38(5):1007–13.
17. Kristensen MS, Teoh WH, Graumann O, Laursen
2. Cherian A, Kundra P. Ultrasound imaging of the air-
CB.  Ultrasonography for clinical decision-making
way and its applications. Airway. 2018;1(1):17–24.
and intervention in airway management: from the
3. Adi O, Kok MS, Abdull Wahab SF.  Focused airway
mouth to the lungs and pleurae. Insights Imaging.
ultrasound: an armamentarium in future airway man-
2014;5(2):253–79.
agement. J Emerg Crit Care Med. 2019;3:31.
18. Srinivasarangan M, Akkamahadevi P, Balkal VC,
4. Zetlaoui PJ. Ultrasonography for airway management.
Javali RH.  Diagnostic accuracy of ultrasound mea-
Anaesth Crit Care Pain Med. 2021;40(2):100821.
surements of anterior neck soft tissue in determin-
5. Jang YE, Kim EH, Song IK, Lee JH, Ryu HG, Kim
ing a difficult airway. J Emerg Trauma Shock.
HS, et  al. Prediction of the mid-tracheal level using
2021;14(1):33–7.
surface anatomical landmarks in adults: clinical impli-
19. Abraham S, Himarani J, Mary Nancy S,
cation of endotracheal tube insertion depth. Medicine
Shanmugasundaram S, Krishnakumar Raja
(Baltimore). 2017;96(12):e6319.
VB. Ultrasound as an assessment method in predict-
6. Thomas VK, Paul C, Rajeev PC, Palatty
ing difficult intubation: a prospective clinical study. J
BU.  Reliability of ultrasonography in confirming
Maxillofac Oral Surg. 2018;17(4):563–9.
endotracheal tube placement in an emergency setting.
20. Komatsu R, Sengupta P, Wadhwa A, Akça O, Sessler
Indian J Crit Care Med. 2017;21(5):257–61.
DI, Ezri T, et al. Ultrasound quantification of anterior
7. Kuriyama A, Jackson JL, Kamei J. Performance of the
soft tissue thickness fails to predict difficult laryn-
cuff leak test in adults in predicting post-extubation
goscopy in obese patients. Anaesth Intensive Care.
airway complications: a systematic review and meta-­
2007;35(1):32–7.
analysis. Crit Care. 2020;24(1):640.
21. Ezri T, Gewürtz G, Sessler DI, Medalion B, Szmuk
8. Schnell D, Planquette B, Berger A, Merceron S,
P, Hagberg C, et  al. Prediction of difficult laryn-
Mayaux J, Strasbach L, et al. Cuff leak test for the diag-
goscopy in obese patients by ultrasound quanti-
nosis of post-extubation stridor: a multicenter evalu-
fication of anterior neck soft tissue. Anaesthesia.
ation study. J Intensive Care Med. 2019;34(5):391–6.
2003;58(11):1111–4.
9. Venkategowda PM, Mahendrakar K, Rao SM,
22. Daniel SJ, Bertolizio G, McHugh T.  Airway ultra-
Mutkule DP, Shirodkar CG, Yogesh H. Laryngeal air
sound: point of care in children-The time is now.
column width ratio in predicting post extubation stri-
Paediatr Anaesth. 2020;30(3):347–52.
dor. Indian J Crit Care Med. 2015;19(3):170–3.
23. You-Ten KE, Siddiqui N, Teoh WH, Kristensen
10. Ding L-W, Wang H-C, Wu H-D, Chang C-J, Yang
MS. Point-of-care ultrasound (POCUS) of the upper
P-C.  Laryngeal ultrasound: a useful method in pre-
airway. Can J Anaesth. 2018;65(4):473–84.
dicting post-extubation stridor. A pilot study. Eur
24. Singh M, Tuteja A, Wong DT, Goel A, Trivedi A,
Respir J. 2006;27(2):384–9.
Tomlinson G, et  al. Point-of-care ultrasound for
11. Sustić A, Miletić D, Protić A, Ivancić A, Cicvarić
obstructive sleep apnea screening: are we there yet? A
T. Can ultrasound be useful for predicting the size of
systematic review and meta-analysis. Anesth Analg.
a left double-lumen bronchial tube? Tracheal width
2019;129(6):1673–91.
as measured by ultrasonography versus computed
25. Stafrace S, Engelhardt T, Teoh WH, Kristensen
tomography. J Clin Anesth. 2008;20(4):247–52.
MS. Essential ultrasound techniques of the pediatric
12. Liao YC, Wu WC, Hsieh MH, Chang CC, Tsai
airway. Paediatr Anaesth. 2016;26(2):122–31.
HC.  Ultrasound-guided superior laryngeal nerve
26. Ahn JH, Park JH, Kim MS, Kang HC, Kim IS. Point
block assists in anesthesia for bronchoscopic
of care airway ultrasound to select tracheal tube and
surgical procedure: a case report of anesthesia
determine insertion depth in cleft repair surgery. Sci
for rigid bronchoscopy. Medicine (Baltimore).
Rep. 2021;11(1):4743.
2020;99(27):e20916.
27. Hao J, Zhang J, Dong B, Luo Z. The accuracy of ultra-
13. Sawka A, Tang R, Vaghadia H.  Sonographically
sound to predict endotracheal tube size for pediatric
guided superior laryngeal nerve block dur-
patients with congenital scoliosis. BMC Anesthesiol.
ing awake fiberoptic intubation. A A Case Rep.
2020;20(1):183.
2015;4(8):107–10.
28. Adi O, Sum KM, Ahmad AH, Wahab MA, Neri L,
14. Kristensen MS, Teoh WH.  Ultrasound identifica-
Panebianco N.  Novel role of focused airway ultra-
tion of the cricothyroid membrane: the new standard
sound in early airway assessment of suspected laryn-
in preparing for front-of-neck airway access. Br J
geal trauma. Ultrasound J. 2020;12(1):37.
Anaesth. 2021;126(1):22–7.
29. Omakobia E, Micallef A. Approach to the patient with
15. Kristensen MS, Teoh WH, Rudolph SS, Tvede MF,
external laryngeal trauma: the schaefer classification.
Hesselfeldt R, Børglum J, et al. Structured approach
Otolaryngology. 2016;6(230):2.
to ultrasound-guided identification of the cricothyroid
3  Point of Care Ultrasound of the Airway 79

30. Dankle SK, Schuller DE, McClead RE. Risk factors epiglottitis in the emergency department: a prelimi-
for neonatal acquired subglottic stenosis. Ann Otol nary study. J Ultrasound Med. 2012;31(1):19–22.
Rhinol Laryngol. 1986;95(6 Pt 1):626–30. 35. Senussi MH, Kantamneni PC, Latifi M, Omranian AP,
31. Nicolli EA, Carey RM, Farquhar D, Haft S, Alfonso Krveshi L, Barakat AF, et  al. Protocolized tracheal
KP, Mirza N. Risk factors for adult acquired subglot- and thoracic ultrasound for confirmation of endotra-
tic stenosis. J Laryngol Otol. 2017;131(3):264–7. cheal intubation and positioning: a multicenter obser-
32. El Amrousy D, Elkashlan M, Elshmaa N, Ragab vational study. Crit Care Explor. 2020;2(9):e0225.
A. Ultrasound-guided laryngeal air column width dif- 36. Isaiah A, Mezrich R, Wolf J. Ultrasonographic detec-
ference as a new predictor for postextubation stridor tion of airway obstruction in a model of obstructive
in children. Crit Care Med. 2018;46(6):e496–501. sleep apnea. Ultrasound Int Open. 2017;3(1):E34–e42.
33. Hung TY, Li S, Chen PS, Wu LT, Yang YJ, Tseng LM, 37. De Jong A, Molinari N, Pouzeratte Y, Verzilli D,
et al. Bedside ultrasonography as a safe and effective Chanques G, Jung B, et  al. Difficult intubation in
tool to diagnose acute epiglottitis. Am J Emerg Med. obese patients: incidence, risk factors, and compli-
2011;29(3):359.e1–3. cations in the operating theatre and in intensive care
34. Ko DR, Chung YE, Park I, Lee HJ, Park JW, You JS, units. Br J Anaesth. 2015;114(2):297–306.
et al. Use of bedside sonography for diagnosing acute
Point-of-Care Ultrasound
of the Lungs 4
Archit Sharma and Sudhakar Subramani

4.1 Introduction ultrasound (LUS) and proposed it as an effective


and accurate goal-directed diagnostic tool that can
Over the last two decades, ultrasound has emerged be applied in real time for the bedside assessment
as a new imaging technique for the thorax, even of patients with respiratory disorders [2].
though the possibility of exploring the lung by LUS has been shown to outperform physical
using ultrasound dates back to the 1960s [1]. examination and chest radiography for both diag-
Lung parenchyma is one of the challenging struc- nosis and monitoring of many pulmonary and
tures to evaluate using ultrasound, primarily due pleural conditions, especially in acute care set-
to high air/tissue ratio. However, in the early tings such as an emergency department and inten-
1990s, with the advent of descriptors like lung sive care units [2, 3]. In addition, suboptimal
sliding, and B-lines, it became possible to evalu- radiography may mask or mimic clinically sig-
ate lung in a systematic way. With a growing nificant abnormalities and at the same time, dif-
interest in the technique, many other features of ferentiation of pleural and parenchymal structures
lung ultrasound, such as artifactual images of nor- can be challenging in these acute settings.
mal aeration (i.e., A-lines), and the real images Moreover, chest ultrasound is increasingly used to
mirroring pathologic conditions (e.g., consolida- guide various interventional procedures such as
tions, pleural effusion) have been described in the placement of intercostal drains. Subsequently, the
comprehensive examination. Many societies have role of LUS expanded to many non-acute care set-
formulated a structured way of performing lung tings and widely used in the assessment of patients
with chronic diseases with lung involvement,
such as chronic heart failure, chronic kidney dis-
Supplementary Information The online version con-
tains supplementary material available at [https://doi. eases, and autoimmune diseases [4]. LUS has
org/10.1007/978-­981-­16-­7687-­1_4]. been established as one of the key diagnostic tools
in the neonatal and pediatric critical care setting
as well [5]. In this chapter, the authors have
A. Sharma
focused on a detailed description of the physics
Divisions of Cardiothoracic Anesthesiology, Solid
Organ Transplant and Critical Care, Department of and physiology of LUS, how to perform LUS,
Anesthesia, Iowa City, IA, USA description of sonographic appearances of normal
e-mail: [email protected] lung and in various pathological conditions, utili-
S. Subramani (*) zation in many clinical settings, limitations and
Department of Anesthesia, Division of Cardiothoracic pitfalls, and comparative outcomes with other
Anesthesiology, Iowa City, IA, USA
commonly used diagnostic modalities.
e-mail: [email protected]

© The Author(s), under exclusive license to Springer Nature Singapore Pte Ltd. 2022 81
A. Chakraborty, B. Ashokka (eds.), A Practical Guide to Point of Care Ultrasound (POCUS),
https://doi.org/10.1007/978-981-16-7687-1_4
82 A. Sharma and S. Subramani

4.1.1 Ultrasound Properties of Lung Overall, LUS images are based on the relative
amounts of air and fluid in the lung. If the lung is
Acoustic impedance (Z), a measure of the resis- highly fluid-filled, then it can be directly visual-
tance of particles in a medium to mechanical ized, whereas in case of pneumothorax, due to air
vibrations determines the appearance of LUS. If being present below the parietal pleura, one is
the density of the given medium and propagation unable to visualize the lung tissue directly.
velocity of US in the medium increases, the over- Although in the past significant attenuation of US
all resistance to sound signal increases. In the signals had discouraged clinicians to utilize US for
thoracic region, there are mediums with different the assessment of lung, in the current day and age,
impedances that cause some of the sounds to be a better understanding of physical properties and
transmitted and some to be reflected as an echo. new and improved ultrasound probes have played
The amount of reflection is based on the differ- a significant role in expanding the use of LUS.
ences in acoustic impedance: fluid appears as
black due to constant Z resulting in no echo, and
soft tissues have a minimal reflection. Majority of 4.1.2 Types of Ultrasound
the reflection happens between soft tissue and air Transducers for LUS
reflect 99.9% rendering this interface virtually
impenetrable to the US. This leads to inability to Table 4.1 shows properties, usefulness, and limi-
visualize normal aerated lung beneath the pleura tations of three commonly used probes for LUS
and only artifacts can be seen [6]. (Fig. 4.1).

Table 4.1  Comparison of three LU transducers


Curvilinear probe Phased array probe Linear probe
• Low frequency • Low frequency • Higher frequency
   (3–5 MHz)    (3–4.5 MHz)    (5–10 MHz)
• Higher penetration • Greater penetration • Poor penetration
• Larger sector width • Poor quality of image • Narrow sector width
• Not ideal for rapid movement • Small footprint allows to avoid • Not ideal for deeper structures
structures ribs • Great image quality for
• Large footprint need more superficial structures
angulation to avoid ribs • Large footprint needs significant
• Lung sliding can be easily angulation to avoid ribs
visualized • No ideal for rapid movement
• Effusions, consolidated lung, and structures
the diaphragm well imaged

Lung Ultrasound Transducers

Curvilinear Probe Phased Array Probe Linear Probe

Fig. 4.1  Types of LU transducers


4  Point-of-Care Ultrasound of the Lungs 83

4.1.3 Performing LUS First International Consensus Conference on


Lung Ultrasound (ICC-LUS) have provided
Transthoracic chest US can be performed with evidence-­based recommendations on “point-of-­
one of the three probes, as described in Fig. 4.1. care” LUS that predominantly focus on six major
Initially, a curvilinear probe can be used for a areas such as terminology, technology, technique,
quick survey of the chest wall, pleura, and lungs. clinical outcomes, cost effectiveness, and future
Once an abnormality has been identified, a high-­ research [2].
resolution linear probe can be used to provide a One of the simplest and quick ways of evaluat-
detailed depiction of any chest wall, pleural, or ing lungs is a three-point (upper anterior, lower
peripheral lung abnormality. In 2D images, the anterior, and postero-lateral) examination of each
echogenicity of a lesion can be compared with lung and is considered as a great starting point for
that of the liver and characterized as hypoechoic, a novice. To locate the three points, apply two
isoechoic, or hyperechoic. The color and quanti- hands side by side (without thumbs) over the ante-
tative Doppler can be used to identify vascula- rior chest with your wrists in the anterior axillary
ture. For improving the color Doppler imaging, line and your upper little finger resting along the
the sensitivity should be set to a low-velocity clavicle. Your lower little finger will be aligned
scale (typically 0.25 m/s), along with a wall filter with the lower border of the lung (the phrenic
to minimize rejection of small frequency shifts line) (Fig. 4.2). Upper anterior point corresponds
and to avoid interference from respiratory or car- to the base of the middle and ring fingers on the
diac movements. It is also suggested to increase upper hand and it lies over the upper lobe. Lower
the color Doppler gain until a light, but uniform, anterior point corresponds to the middle of the
background colored “snowstorm” is obtained. palm on the lower hand and it lies over the middle
The Doppler angle should be less than 60° for or lingular lobe. For postero-­lateral point, move
accurate evaluation of vascular flow and needs to the probe laterally and posteriorly as far as possi-
be repeated at least twice to ensure reproducibil- ble behind the posterior axillary line from the
ity of the spectral waveform. M-mode is used to lower anterior point and it lies over the lower lobe.
demonstrate certain artifacts to diagnose lung/ In the three-point examination, all views are
pleural abnormality. obtained in longitudinal orientation. Raising the
Prior to image acquisition, the operator should arm above the patient’s head increases the rib
ensure correct image orientation displayed on the space distance and facilitates scanning with the
screen such that the left side of the image should patient in erect or recumbent positions. The poste-
correspond with either the right side of the patient rior chest is best imaged with the patient sitting
(if transverse) or cephalad (if longitudinal). A upright, while the anterior and lateral chest may
comprehensive examination can be performed in be assessed in the lateral decubitus position.
supine position especially in acute care settings Maximum visualization of the lung and pleural
or in sitting or decubitus positions. We will dis- space is achieved by scanning along the intercos-
cuss a few of the protocols that have been tal spaces. US should be performed during quiet
described in the literature to examine the thorax respiration, to allow for assessment of normal
[7]. Growing application of LUS in different set- lung movement, and in suspended respiration,
tings has led to a difference in the approach and when a lesion can be examined in detail with 2D
the nomenclature. A panel of experts from the or color Doppler (Fig. 4.2).
84 A. Sharma and S. Subramani

Upper anterior point

Posterior-lateral alveolar pleural


Lower anterior point (PLAPS) point

Fig. 4.2  Three point examination

Intercostal spaces
Intercostal spaces
5th
3rd 4th
2nd 2nd
9 5 1 17 20 23
26
Anterior axillary line 13
12 3rd
11 6 2 18
21
10 14
10 24 27
9 4th
Mid-axillary line 19
3 22
7 25
16 15 11
5th 28
15 4
14 8
16 12
13
Parasternal lines Mid-clavicular Anterior axillary
line line

28 - point method
8 - point method
6 - point method
& 4 - point method

Fig. 4.3  Various scanning sites

4.1.4 Blue Protocol (Four-Point Other least commonly used methods are six-­
Method) point, eight-point, and 28 points. A six-point
method consists of two blue points and an addi-
Blue protocol is based on a four-points assess- tional scanning site on the fifth intercostal space at
ment, which is basically upper and lower ante- mid-axillary line [9]. In the eight-point method,
rior points, on both sides of the chest wall. A two anterior points between the sternum and the
positive point is defined as the presence of at anterior axillary line, and two lateral points
least three B-lines. For positive examination, between the anterior and the posterior axillary line
for certain acute respiratory disorders, at least are the scanning sites [10]. In the 28-points
three B-lines on each scanning site must be method, scanning sites are at 16 points on the right
present [8]. side and 12 points on the left as shown in Fig. 4.3
4  Point-of-Care Ultrasound of the Lungs 85

Fig. 4.4  LUS of normal


lung P

Subcutaneous
Tissue

Pleural Line

[11]. The supplementary videos 4.41–4.4 provide


an example of scanning the chest wall from vari-
ous positions. Video 4.1: Right upper anterior
point using phased array probe, Video 4.2: Right
Pleural Line
lower anterior point using a transverse probe,
Video 4.3: Right Posterior-lateral alveolar pleural A line
point, Video 4.4: Right costophrenic view.

4.1.5 Ultrasound Image


of Normal Lung
Fig. 4.5  LUS avoiding ribs shows multiple A-line
Visualization of the chest wall is achieved with
the placement of the transducer in sagittal orien-
tation in any intercostal space between second cortex should appear as a continuous smooth
and seventh space. All signs in LUS arise from echogenic line; (c) pleural line, caused by near-
the pleural line, except for subcutaneous emphy- complete reflection of the ultrasound beam at the
sema which will not display the pleural line due aerated lung and it appears as a hyperechoic
to the presence of air in the superficial tissue. homogeneous horizontal line between the pleura
The normal chest wall appears as a series of and aerated lung tissue, the visceral pleura usu-
echogenic soft tissue layers, representing the ally appears thicker than the parietal pleura, the
layers of muscles and the fascia planes. The fol- curvilinear probe unable to differentiate visceral
lowing structures and artifacts can be obtained and parietal pleura as it appears as echogenic
from normal lung ultrasound from superficial to bands measuring up to 2 mm thick; (d) multiple
deep: (a) subcutaneous tissues and intercostal hyperechoic horizontal lines called A-lines
muscles; (b) upper and lower ribs with acoustic appears below the pleural line, spaced at multi-
shadowing beneath the ribs due to near complete ples of the distance between the probe and the
reflection of the ultrasound beam at bone pleural line, A-lines are due to reverberation arti-
(Fig.  4.4 acoustic shadowing), if you turn the facts generated from the strong reflection of the
transducer along long axis of ribs, the anterior pleural line (Fig. 4.5).
86 A. Sharma and S. Subramani

4.1.6 Lung Sliding and Lung Pulse 4.1.7 Pneumothorax

The lung sliding and lung pulse are the two The key LUS findings associated with pneumo-
dynamic findings found in a normal lung ultra- thorax are the absence of lung sliding, absent
sound. These two features are caused by the B-lines, lung pulse, and presence of the lung
movement of the lung surface (visceral pleura) point. In supine patients, air, being nondependent
in relation to the innermost chest wall (parietal tends to collect anteriorly. In order to exclude or
pleura). The lung sliding represents air move- to diagnose pneumothorax, it is recommended to
ment during respiration and lung pulse indicates place the probe on the highest point of the ante-
the transmission of cardiac contractions through rior chest to demonstrate lung sliding. Absence of
the lung. Turning the probe transversely will lung sliding has a 95% sensitivity and 100% neg-
abolish the rib shadows so more of the pleural ative predictive value in predicting a pneumotho-
line can be seen. The danger of this is that an rax [12]. M-mode imaging will be helpful in
inexperienced user may interpret a rib as the certain challenging patients to diagnose pneumo-
pleural line and incorrectly diagnose absent thorax. It demonstrates only horizontal lines
lung sliding. M-mode imaging will be helpful to without sandy appearance, normal appears in
exclude pneumothorax as it shows seashore near field termed as “stratosphere sign” or bar-
sign, one of the characteristic features of lung code sign (Fig.  4.7) However, one should be
sliding. The horizontal line represents subcuta- aware of other conditions causing absence lung
neous tissue above the pleural line while the sliding such as pleural effusion or if both pleurae
movement of lung sliding appears as sand in far fused together due to underlying pneumonia,
field (Fig. 4.6). pleurodesis, or ARDS. Moreover, the lung sliding

Fig. 4.6  Seashore sign


4  Point-of-Care Ultrasound of the Lungs 87

Fig. 4.7  Stratosphere or barcode sign


Fig. 4.8  Curtain sign from hydropneumothorax
will be reduced with low tidal volume ventilation
or in hyperinflated lungs. Also, lung sliding will with LUS with the “curtain sign” describing
be absent if there is no respiration in one lung due reverberation artifacts originating from the air
to pneumonectomy or one lung intubation. within the pleura that obscures the underlying
In some situations, if one is unable to demon- effusion during inspiration (Fig.  4.8) [16]. The
strate absence of lung sliding in suspected simplified algorithm for pneumothorax is shown
patients, demonstrating B-lines arising at vis- in Fig. 4.9.
ceral pleura-lung boundary, will be helpful to Use of a combination of absent lung sliding
exclude pneumothorax. If there is no evidence and the loss of B-lines has a reported sensitivity
of lung sliding and B-lines, then one should of 100%, specificity of 96.5%, and negative pre-
look for lung point, the point at which the two dictive value of 100% [17].
pleural layers rejoin one another [13]. It is also
explained by the inspiratory increase of parietal
contact of the collapsed lung. Lung point is 4.1.8 Limitations for Identification
noticed in the lateral aspect of chest wall and to of Pneumothorax
demonstrate it, the ultrasound transducer needs
to be moved laterally. Detection by LUS has a Certain patient-specific factors such as obesity,
sensitivity is 66% for fully collapsed lungs and edema, or heavy musculature may degrade image
79% for occult pneumothorax [14]. It is also quality, making it hard to clearly demonstrate
helpful to indirectly judge the volume of air in pleural line. The presence of subcutaneous
the pleural cavity to determine appropriate emphysema or conditions causing pleural thick-
intervention. ening or calcifications can block visualization of
Finally, if the lung point cannot be demon- the pleural line [18]. In patients with ARDS, vig-
strated then the “lung pulse” should be sought. orous intercostal muscle contraction adjacent to
Cardiac pulsation will be transmitted to pleura by the pleura results in movement of the adjacent
lung only if the pleural layers are still adjacent to parietal pleural surface. There will be potential
one another causing a small amount of motion in operators to misdiagnose this abnormal move-
2D in relation with the heartbeat [15]. M-mode ment as the presence of lung sliding. One should
will be useful to demonstrate vertical lines run- be aware of the differences between the move-
ning from the pleural line to the bottom of the ment of the pleural line by intercostal muscle
image in time with cardiac pulsation called T contraction and from the shimmering movement
lines. Hydropneumothorax can also be identified of lung sliding [19].
88 A. Sharma and S. Subramani

Lung
Sliding No

Yes B Lines No

Yes
Lung
point Yes

NO PNEUMOTHORAX PNEUMOTHORAX
No

Lung
Yes No
pulse

Fig. 4.9  Simplified algorithm for pneumothorax.

4.1.9 Pulmonary Edema ther decreased, such as in lung consolidations,


and Interstitial Lung Disease the acoustic window on the lung becomes com-
pletely open and the lung may be directly visual-
Until recently, it was thought that ultrasound of ized as a solid parenchyma, like the liver or the
the lung tissue would be ineffective because the spleen. It has been reported that one can assess
air in the lung parenchyma would cause too much the space between B-lines to help determine fur-
acoustic impedance. However, this has been ther detail about the pathology of the airspace
shown to not be the case; and specifically, when disease [20].
the air content decreases (as in pulmonary edema A positive region is defined by the presence of
or any interstitial lung disease), an acoustic mis- three or more B-lines in a longitudinal plane
match needed to reflect the ultrasound beam is between two ribs. Multiple B-lines which are
created, and a signature ultrasound finding 7  mm apart characterize subpleural interlobular
appears (B-lines). This reflection of the beam septal edema or thickening (Fig.  4.10). Closely
creates some comet-tail reverberation artifacts, spaced (≤3  mm) and coalescent B-lines suggest
called B-lines or ultrasound lung comets. A subpleural fluid-filled alveoli which correspond to
B-line is a discrete, laser-like, vertical, hyper- ground glass opacities in CT. The term “B-pattern”
echoic image that arises from the pleural line, should be used in the description of multiple
extending to the bottom of the screen without B-lines in patients with interstitial syndrome [21].
fading, and moves synchronously with respira- In either case, the number and intensity of B-lines
tion. Multiple B-lines (>2) are the sonographic increase with the degree of loss of aeration. It is
sign of interstitial lung disease and their number important to realize that absence of normal arti-
increases along with decreasing air content as facts may also provide useful information. One
shown in Fig. 1.10. When the air content is fur- such artifact is what are termed A-lines, which
4  Point-of-Care Ultrasound of the Lungs 89

Fig. 4.10  B-lines in pulmonary edema and interstitial


lung disease

represent the reverberation artifact of lung pleura.


This artifact is a hyperechoic horizontal line that
is parallel to the pleural line. The distance from
A-line to pleural line is equal to the distance
between skin and pleural line, and this may repeat
further down the ultrasound image. The presence
of A-lines is a sign of normal aeration but may
also occur in the setting of pneumothorax. One
must always assess for pleural sliding when Fig. 4.11  Pleural effusion along with collapsed lung
A-lines are visualized. Also, since the presence of
A-lines is a sign of normal aeration (assuming
normal pleural sliding) one should not often see
A-lines and B-lines in the same image.

4.1.10 Pleural Effusion

The appearance of effusion between parietal and


visceral pleura in ultrasound images depends on
its nature. Most transudates, as well as some exu-
dates, appear as anechoic space (dark shadow)
between pleura because at pleura-effusion bound-
ary negligible echo is produced. This is well
demonstrated in Fig. 4.11. The presence of inter-
nal echoes (grey smoky shadows) in this anechoic
space is suggestive of exudate or hemorrhage
which can be confirmed by thoracentesis. The Fig. 4.12  Sinusoid sign caused by pleural movement
visceral pleura-lung boundary appears as a hyper-
echoic line in a normally aerated lung. movement of visceral pleura and called sinusoid
In a free effusion and aerated lung, the varia- sign, as is demonstrated in Fig. 4.12.
tion in interpleural distance during respiratory The posterior axillary line above the dia-
cycle is easily visualized in M-mode as sinusoid phragm is the optimal site for the detection of
90 A. Sharma and S. Subramani

non-loculated pleural effusion [22]. Various


ultrasound-guided approaches have been sug-
gested for assessment of volume of pleural effu-
sion. Interpleural distance of ≥50  mm between
posterior chest wall and lung is predictive of
pleural effusion ≥500 mL [23]. The LUS is supe-
rior to supine chest X-ray in the diagnosis of
pleural effusion and distinguishing it from other
causes of white opacities in chest X-ray. The
LUS is as good as CT in the diagnosis of pleural
effusion [2]. A hydropneumothorax can be iden-
tified by a dynamic air-fluid margin, with features
of pneumothorax above air-fluid level.

Fig. 4.13  Lung consolidation demarcated from normal


lung tissue
4.1.11 Consolidation

The appearance of LUS depends on the relative as the “shred sign.” Air within the consolidated
aeration of alveoli. Pneumonia is an inflamma- area may remain in small aerated patches of lung,
tory, and infectious process involving the lungs. or more commonly air remains within small bron-
Typically, the alveoli in intensely inflamed areas chi. The echogenic appearance of small air bub-
fill with inflammatory fluid or pus, and this is bles all lined up within a bronchus is known as the
known as consolidation. The changes may be sonographic air bronchogram.
widespread, patchy, or lobar. Ultrasound can
detect the pulmonary changes associated with
pneumonia as long as the process involves some 4.1.12 Atelectasis
of the pleural surface. Pneumonia progresses
through stages, and the ultrasound changes vary Subpleural echo-poor region or tissue-like
depending on the degree and extent of consolida- echotexture is seen in compression or obstructive
tion. In the appropriate clinical setting, a local- atelectasis. This appearance is due to the absence
ized patch of numerous B-lines, often with tiny of peripheral lung expansion. Compression atel-
areas of subpleural consolidation, suggests early ectasis caused by massive pleural effusion is seen
pneumonia. as floating wedge-shaped tissue-like structure, as
As the disease progresses, inflammatory and is demonstrated in Fig. 4.14 [21]. The absence of
purulent fluid fills the alveoli and the lung appears dynamic air/fluid in this tissue like echotexture
solid, with a homogenous relatively fine echotex- helps in differentiating it from pneumonia. Loss
ture similar to liver. The sonographic appearance of aeration in lung at various stages due to
of frank consolidation looks remarkably liver-­like ­gradually increasing pleural effusion can be seen
and is also termed hepatization. More commonly as compression atelectasis develops. Compression
there are smaller areas of consolidation, as dem- or resorptive atelectasis shows early and late
onstrated in Fig. 4.13. Where these abut the pleu- signs along with tissue-like echotexture. Early
ral surface, they are linear, but their deeper borders signs are abolished lung sliding with lung pulse,
usually demonstrate an irregular interface with standstill cupola. Residual trapped air with static
underlying aerated lung. This irregular junction air bronchogram is a late sign. Air is completely
between consolidated and aerated lung is known absorbed subsequently.
4  Point-of-Care Ultrasound of the Lungs 91

Fig. 4.15  Hyperinflated lung in a patient with COPD,


with A-line appearance

Fig. 4.14  Pleural effusion along with atelectatic lung The BLUE protocol has been proposed to help
floating in the pleural fluid in the diagnosis of various lung conditions, as
described before [26, 27]. Each region or zone is
4.1.13 Asthma/Chronic Obstructive then sonographically characterized. Integration
Pulmonary Disease of various LUS findings in all regions/zones
gives sonographic diagnosis. Finally, interpreta-
The presence of pleural line, lung sliding, tion of sonographic diagnosis is done in context
A-Lines in 2D, and seashore sign in M-mode in with history, clinical assessment, other investiga-
both lungs as characteristic LUS picture of aer- tions, and laboratory data. One algorithm is sug-
ated lung have been mentioned above. In condi- gested in Fig.  4.16. Table  4.2 summarizes the
tions like COPD, asthma and pulmonary major LUS findings in different pathological
embolism subpleural aeration are usually not states.
affected. Lung sliding may be reduced by hyper-
inflation [24]. These conditions usually present
with a similar LUS picture of aerated lung, as 4.1.15 Lung Ultrasound in COVID-19
shown in Fig.  4.15. The BLUE protocol has
described this LUS picture as A-profile and sug- POCUS has found its wide application in emer-
gested a decision tree to differentiate these gency and critical care medicine [2]. Many stud-
chronic conditions [25]. ies have shown that ultrasound imaging is more
sensitive than chest X-ray examination in the
diagnosis of ventilator-associated pneumonia
4.1.14 Pulmonary Embolism [28] and community-acquired pneumonia
(CAP) [29–31]. It can also make a sensitive
In the case of a pulmonary embolism, usually an diagnosis of viral pneumonia such as H1-N1 or
A-profile or A-pattern is seen in LUS. The BLUE H7-N9 earlier than chest X-ray examination
protocol suggests searching for venous thrombo- [32] and thus has been recommended as an
sis if A-profile is present. If venous thrombosis is alternative screening method for such pneumo-
present, diagnosis of pulmonary embolism is nia in endemic areas [33, 34]. However, ultra-
considered in patients with acute respiratory fail- sound identification between viral pneumonia
ure. Peripheral pulmonary embolism is seen as and bacterial pneumonia remains a huge chal-
multiple hypoechoic subpleural wedge-shaped lenge. Use of LUS is feasible in the diagnosis of
tissue-like echotexture. There can be local or COVID-19, and the ultrasound images are
basal pleural effusion also [26]. mainly manifested as B-line artifacts, rocket
92 A. Sharma and S. Subramani

The
BLUE
(upper and lower BLUE-points) protocol
Lung Sliding

Present any abolished

B-profile A-profile A/B or C profile B’ -profile A’ -profile

PULMONARY EDEMA sequential PNEUMONIA PNEUMONIA plus lung without lung


venous analysis point point

Thrombosed vein Free veins PNEUMOTHORAX Need for other


diagnostic
modalities
PULMONARY EMBOLISM Stage 3
(PLAPS-point)

This decision tree is not designed for


PLAPS no PLAPS providing 100% of diagnoses of acute
dyspnea. It has been simplified with the
target of 90.5% of overall accuracy.
PNEUMONIA COPD or ASTHMA

Fig. 4.16  The blue protocol

Table 4.2  Ultrasound findings of different pathologies


Lung pathology USG features Distribution
Normal lungs Ribs and rib shadows: Present Both lungs
Hyperechoic pleural line: Present In laterobasal areas
Lung sliding: Present A-profile
A-lines: Present
M-mode: Seashore sign present
Isolated B-line may be seen
Subcutaneous emphysema Ribs and rib shadows: Absent In affected areas
asthma/COPD E lines: Present In involved lung
A-profile
↓ Lung sliding, if hyperinflation is present
Pulmonary embolism A-profile Both lungs
Peripheral pulmonary embolism Focal/multifocal
Subpleural wedge-shaped tissue like echo may inhomogeneous
be seen
Focal/basal pleural effusion may be present
Pneumothorax Ribs and rib shadows: Present In affected lung
Hyperechoic pleural line: Present Focal
Lung sliding: Absent
A-lines: Present
B-lines: Absent characteristic
Lung point: Present
Lung pulse: Absent
M-mode: Stratosphere sign present
Pleural effusion Ribs and rib shadows: Present In affected hemithorax
Hyperechoic pleural line and lung sliding:
Absent
Anechoic (dark) space: Present
(internal echoes present, if it is transudate)
Hyperechoic lung line present if the underlying
lung is aerated
M-mode: Sinusoid sign, quad sign present, if
underlying lung aerated
4  Point-of-Care Ultrasound of the Lungs 93

Table 4.2 (continued)
Lung pathology USG features Distribution
Pulmonary edema Ribs and rib shadows: Present Both lungs
Hyperechoic pleural line and lung sliding: Homogeneous
Present Dependent areas
Pleural line and subpleural abnormalities:
Absent
A-lines: Present
B-lines: Significant and ≥2 bilateral regions +
ALI/ARDS Ribs and rib shadows: Present Both lungs
Pleural line: Present Inhomogeneous
Lung sliding: Present/may be ↓ Spared areas of normal
Pleural and subpleural abnormalities: Present Appearance seen
Anterior subpleural tissue like echo may be
seen
A-lines: Present
B-lines: Significant but irregularly spaced
Pulmonary fibrosis Ribs and rib shadows: Present Affected lung may be bilateral
Pleural line and lung sliding: Present or unilateral
Lung sliding: Normal/may be ↓ Focal/multifocal
Pleural line and subpleural abnormalities: Inhomogeneous
Present
A-lines: Present
B-lines: Significant and spaced
Lung contusion Ribs and rib shadows: Present In affected lung
Pleural line and lung sliding: Present May be bilateral or unilateral
Lung sliding: Normal/may be ↓ Focal/multifocal
Pleural line and subpleural abnormalities: Inhomogeneous
Present
A-lines: Present
B-lines: Significant and spaced/crowded
Interstitial/ Ribs and rib shadows: Present In affected lung
broncho-pneumonia Plural line and lung sliding: Present May be bilateral or unilateral
Lung sliding: Normal/may be ↓ Focal/multifocal
Pleural line and subpleural abnormalities: Inhomogeneous
Present
A-lines: Present
B-lines: Significant and spaced/crowded
Pneumonia consolidation Ribs and rib shadows: Present In affected lung
Pleural line and lung sliding: Absent May be unilateral or bilateral
Tissue like echotexture of consolidated lung Focal/multifocal
Dynamic air/fluid bronchogram: Present Inhomogeneous
Shred sign: Present
Atelectasis Ribs and rib shadows: Present In affected lung
Pleural line and lung sliding: Absent
Tissue like echotexture of atelectatic lung
Dynamic air/fluid bronchogram: Absent
Shred sign: Absent
Attendant signs of atelectasis: Present
Evolving atelectasis
Image characteristics depend on aeration
COPD chronic obstructive pulmonary disease, USG use of ultrasonography
* Correlate USG image with history, clinical examination, and clinical presentation
94 A. Sharma and S. Subramani

sign, and partially or completely diffused B-line.


Small consolidation and abnormal pleural lines
are detected in some cases. Ultrasound images
of CAP were often manifested as large and cir-
cumscribed consolidation accompanied by
bronchial gas phase or liquid phase, and pleural
effusion was more common in CAP patients
while their lung lesions were, to some extent,
Lung
limitedly distributed compared with those of Consolidation
COVID-19 patients. Consolidations are more
extensive in bacterial pneumonia because a
large number of fibrinogens permeated out from
blood vessel to alveoli as the permeability of the Fig. 4.17 Bilateral ground glass opacifications in a
blood vessel increases. A limited number of COVID-19 patient
studies [35] showed that the pathologic features
of COVID-19 were mainly manifested by dif- sonographic features of COVID-19 were as
fused interstitial inflammation alveolar damage, f­ ollows: (i) thickening, blurred and irregular, and
including apoptosis and shedding of alveolar fragmented pleural line; (ii) dispersed B-line and
epithelial cells, formation of pulmonary hyaline rocket sign, partially diffused B-line, completely
membrane, obviously widened lobar space, and diffused B-line for white lung or waterfall sign,
infiltration of inflammatory cells composed of pulmonary consolidations or subpleural focal
lymphocytes and monocytes. lesions, generally less than 1.0 cm and shown as
The similar pathologic features among a C-line sign, were often seen among severe
COVID-19, SARS, and Middle East Respiratory COVID-19 patients, while large pulmonary con-
Syndrome [36, 37] were the pathologic basis for solidation was uncommon. Fig.  4.17 demon-
the formation of lung ultrasound signs among strates the characteristics of the lung in a
patients with such diseases. The unique ultra- COVID-positive patient.
sound signs of interstitial pneumonia included
the disappearance of pleural line and the overlap-
ping of B-line, as well as white lung, waterfall 4.1.16 Role of LUS in Thoracic
sign, and small consolidations in patients with Interventions/Procedures
severe interstitial pneumonia. Combined with
other studies, this study indicated that different Ultrasound can be used as a guide during thora-
viral pneumonias had similar sonographic fea- centesis procedures or for the placement of chest
tures, and there was still no evidence of obvious tubes in patients with pleural effusions (inflam-
identification characteristics among them. matory, infectious, neoplastic) or inflammatory
However, compared with pulmonary edema and infiltrates involving the lung parenchyma or chest
interstitial lung fibrosis, diffuse B-line patterns wall. Imaging can also be used to collect samples
were more commonly detected in COVID-19 of expanding bone lesions of the ribs, which are a
patients. The reasons might be that the more frequent site of distant metastases and involve-
severe pulmonary inflammation and more abun- ment from disease arising in other contiguous
dant mucus were conducive to the formation and structures. For complicated, antibiotic-resistant
fusion of B-line in such patients [38, 39]. effusions which can form fibrous septa, ­ultrasound
Unexpectedly, large consolidation was rarely can also be used as a guide for intracavitary
seen in COVID-19, and either bronchial fluid administration of fibrinolytic agents, which lyse
phase or gas phase was rarely seen in the consoli- adhesions and septa, improving the chances for
dation, whereas nearly all the pleural abnormali- effective drainage. Ultrasound-guided biopsy is
ties were detected in this study. The main indicated for Pancoast tumors, whose pleural and
4  Point-of-Care Ultrasound of the Lungs 95

extrapleural extension can be evaluated in part by exam. Exceptional cases provide difficult inter-
sonography. The use of color Doppler is funda- pretation, even for experts. Is lung ultrasound
mental for evaluating the vessels of this region. easy? Some experiences show high interob-
The complications commonly associated with server agreement [8]. A burgeoning literature,
interventional procedures involving the chest, up to a consensus conference, seems to confirm
regardless of the type of guidance used, are this accessibility [2]. A scientific assessment of
pneumo- and hemothorax, hemoptysis, vasovagal the learning curve remains to be done, not in
reactions, and if a malignant lesion is being biop- volunteers (creating a selection bias), but in
sied, neoplastic seeding along the needle tract. unselected physicians. Care should be taken to
When interventional procedures are performed confide training to experts choosing simplicity,
under ultrasound guidance, the rate of complica- although one can practice lung ultrasound with
tions is around 1% [40], which is lower than the any machine, any probe, and any teaching
rates reported for CT-guided procedures [41]. approach.
In conclusion, the scope of LUS in emer-
gency and critical care settings has expanded
4.1.17 Advantages of LUS extensively. This imaging modality is easily
available at bedside, real time, and free of
All intensivists prefer the least invasive tool, all else radiation hazards in comparison to conven-
being equal. Ultrasound is an answer to the long- tional imaging modalities of lung, in critically
standing dilemma: “Radiography or CT in the ill patients. It can be used easily at the bedside
ICU?” Radiography is a familiar tool that lacks to assess initial lung morphology in severely
sensitivity [42]: about 60–70% [43]. CT has high hypoxemic patients and can be easily repeated,
accuracy but severe drawbacks: cost, transportation allowing the effects of therapy to be moni-
of critically ill patients, delay between CT and the tored. Its use in the diagnosis of lung diseases
resulting therapy, renal issues, anaphylactic shock, is real but with some practical limitations.
and mainly high irradiation [44]. Ultrasound has Importance of history taking and clinical
quite similar performances to CT [12, 45] being on examination cannot be undermined, and one
occasion superior: better detection of pleural septa- should not jump to the conclusion using just
tions, necrotic areas, real-time measurement allow- the LUS findings.
ing assessment of dynamic signs: lung sliding, air
bronchogram [46], and diaphragm [47]. Ultrasound
should be considered as reasonable, bedside “gold  elf-Assessment Questions: Select
S
standard.” For all assessed disorders, it provides the Best Answer
quantitative data. Pleural effusions can be quanti-
fied [48]. Lung consolidation can be monitored, 1. The maximum reflection of ultrasound sig-
which is useful for those who want to increase end- nals during thoracic ultrasound happens at
expiratory pressure [49]. The volume and progres- A. Skin and subcutaneous
sion of a pneumothorax are monitored using the B. Subcutaneous and muscle layer
lung point location [50]. Lung ultrasound will favor C. Soft tissue and pleura
programs allowing decrease in bedside radiographs D. Pleura and lung parenchyma
and CTs in the next decades. 2. Following are the features of using a linear
ultrasound probe in LUS except
A. Deeper penetration
4.1.18 Limitations B. Has narrow sector width
C. Large footprint
Dressings and subcutaneous emphysema create D. Not recommended for rapidly moving
limitations to performing a good ultrasound structures
96 A. Sharma and S. Subramani

3. For better image quality using color Doppler 7. Following are advantages of using curvilin-
all are recommended except ear probe in LUS except
A. Using wall filter for rapidly moving A. Smaller footprint
structures B. Higher penetration
B. High scale velocity C. Ideal for assessment of diaphragm
C. Adjust color gain to get “snowstorm” D. Lower frequency
appearance 8. Blue protocol is one of the commonly
D. Doppler angle must be less than 60° employed methods for comprehensive
4. The structure highlighted in the following assessment of lung using ultrasound. A posi-
image represents tive blue point represents
A. Presence of two B-lines
B. Presence of one B-line
C. Presence of three B-lines
D. Presence of four B-lines
9. Absence of lung sliding is one of the essen-
tial features of LUS to diagnose pneumo-
thorax. Presence of the following
respiratory condition will cause false lung
sliding due to vigorous intercostal muscle
contraction
A. Emphysema
B. Chronic bronchitis
C. Acute respiratory distress syndrome
D. Bronchial asthma
10. The highlighted structure/artifact in the fol-
lowing ultrasound image represents

A. Pleural line
B. B-line
C. A-line
D. Z-line
5. The most appropriate technique to avoid
acoustic shadowing from ribs is
A. Using curvilinear probe
B. Scanning in sagittal plane
C. Phased array probe parallel to rib
D. Using linear probe A. A-line
6. A-lines are one of the common artifacts B. Pleural line
observed in LUS.  The true statement about C. T-line
A-line is D. B-line
A. Reflection from bony surface 11. You have high peak ventilation pressures

B. Refraction from pleural surface after intubating a patient with advanced
C. Reverberation from pleural line COPD. You find bilateral anterior B-lines but
D. Refraction from fascial plane cannot find lung sliding over the right ante-
4  Point-of-Care Ultrasound of the Lungs 97

rior thorax. What is the most likely cause of


high peak pressures?
A. Pleural effusion
B. Pulmonary edema
C. Pneumothorax
D. Pericardial tamponade
12. You are taking care of a trauma patient who
is admitted after a collision between the car
that he was an unrestrained driver in ran
into a tree. The patient has multiple rib frac-
tures and has a high oxygen requirement of
70% oxygen on a noninvasive BiPAP
machine to maintain oxygenation. He is
also complaining of dyspnea and chest full-
ness. You perform a lung ultrasound, and
A. COPD
the attached picture is displayed. What is
B. Pneumonia
the most likely reason for his respiratory
C. Pericardial effusion
condition?
D. Pneumothorax
14. A 75-year-old woman, with chronic myeloid
leukemia presents with dyspnea and fever.
She presented to her doctor a week ago with
fever and cough, for which she was treated
with azithromycin. She was feeling better for
a while but has been declining for the past
couple of days. Please interpret her lung
ultrasound findings

A. Pleural effusion
B. Pulmonary contusions
C. Pericardial effusion
D. Pneumothorax
13. A 50-year-old patient presents with a week-
long history of dyspnea. Fever, and cough.
His blood pressure is 90/50 and he is requir-
ing four liters of oxygen to maintain his satu-
ration. The attached picture represents the A. Pleural effusion
ultrasound exam with the probe held in place B. Pneumonia
at the base of the lung. What is the most C. Pericardial effusion
likely diagnosis? D. Pneumothorax
98 A. Sharma and S. Subramani

15. You are called on the floor to evaluate a


A. Pleural effusion
90-year-old man with diastolic dysfunction. B. Pneumonia
His diuretics have been held for the past week C. Pericardial effusion
due to renal dysfunction (currently his creati- D. Pulmonary edema
nine has plateaued at 4.2 mg/dL). He has been 16. A 29-year-old male, weighing 134  kg is
placed on BiPAP but continues having sub- brought to the emergency department after
stantial work of breathing. The following a motor vehicle accident. He has lost a lot
images are obtained from his right thorax. of blood and is hypotensive and anemic
What is the most likely cause of his pulmonary with a Hb of 6.2. An emergent central line
dysfunction, based on the ultrasound findings is placed in his right internal jugular and is
challenging to place, given his body habi-
tus. The patient becomes hemodynami-
cally unstable after line placement and
there is some concern that he might have
developed an iatrogenic pneumothorax
due to accidental puncture during line
placement. A lung ultrasound over the
right upper chest shows the following
picture

What is the most accurate statement? ated and fluid-filled parts of the lung
A. Patient does not have pneumothorax tissue
B. Patient does not have pneumothorax, at B. B-lines are evenly spaced horizontal
that point. A more comprehensive lung lines at integer multiples of the distance
exam is required from the skin to the pleural line
C. Patient needs an emergent chest tube C. B-lines indicate well-aerated lung
D. Patient has a hemothorax that needs to be D. B-lines are diagnostic with for
drained pneumothorax
17. Which one of the following statements about 18. Absence of pleural lung sliding can occur in
B-lines is TRUE? all of the following situations except
A. B-lines are reverberation artifacts arising A. Pneumothorax
from volumetric variations between aer- B. Apnea
4  Point-of-Care Ultrasound of the Lungs 99

C. Bronchial intubation References


D. Bronchial obstruction
E. Pleural adhesion 1. Ross AM, Genton E, Holmes JH. Ultrasonic examina-
F. All of the above tion of the lung. J Lab Clin Med. 1968;72(4):556–64.
2. Volpicelli G, Elbarbary M, Blaivas M, Lichtenstein
19. A 64-year-old female, with a history of HTN, DA, Mathis G, Kirkpatrick AW, International
HLD, and COPD has an onset of shortness of Liaison Committee on Lung Ultrasound (ILC-LUS)
breath and sudden chest pain for the past few for International Consensus Conference on Lung
hours, after a coughing spell. The physician Ultrasound (ICC-LUS), et al. International evidence-­
based recommendations for point-of-care lung ultra-
evaluates the heart and lungs with ultra- sound. Intensive Care Med. 2012;38(4):577–91.
sound. When looking at the lungs, the 3. Mojoli F, Bouhemad B, Mongodi S, Lichtenstein
physician notices multiple horizontal lines D.  Lung ultrasound for critically ill patients. Am J
between two hypoechoic columns as well as Respir Crit Care Med. 2019;199(6):701–14.
4. Gargani L, Volpicelli G. How I do it: lung ultrasound.
a lung point however he does not see any Cardiovasc Ultrasound. 2014 Jul;4(12):25. https://doi.
lung sliding. What is the most likely org/10.1186/1476-­7120-­12-­25.
diagnosis? 5. Singh Y, Tissot C, Fraga MV, Yousef N, Cortes RG,
A. Pleural effusion Lopez J, et  al. International evidence-based guide-
lines on point of care ultrasound (POCUS) for criti-
B. Pneumonia cally ill neonates and children issued by the POCUS
C. Pericardial effusion Working Group of the European Society of Paediatric
D. Pneumothorax and Neonatal Intensive Care (ESPNIC). Crit Care.
20. A 82-year-old male with a past history of 2020;24(1):65.
6. Aldrich JE. Basic physics of ultrasound imaging. Crit
CAD, systolic heart failure with EF of 30%, Care Med. 2007;35(Suppl):S131–7.
COPD, and a 3-year history of progressively 7. Buessler A, Chouihed T, Duarte K, Bassand A, Huot-­
worsening dyspnea on exertion finally pres- Marchand M, Gottwalles Y, et al. Accuracy of several
ents to his PCP for workup. The doctor lung ultrasound methods for the diagnosis of acute
heart failure in the ED: a multicenter prospective
notices distended jugular veins and upon study. Chest. 2020;157(1):99–110.
auscultation hears an S3 and a holosystolic 8. Lichtenstein D, Goldstein I, Mourgeon E, Cluzel P,
murmur over the apex. The physician per- Grenier P, Rouby JJ. Comparative diagnostic perfor-
forms a chest ultrasound exam and sees the mances of auscultation, chest radiography, and lung
ultrasonography in acute respiratory distress syn-
following. Based on this exam, which of the drome. Anesthesiology. 2004;100(1):9–15.
following is true? 9. Pivetta E, Goffi A, Lupia E, Tizzani M, Porrino G,
Ferreri E, SIMEU Group for Lung Ultrasound in
the Emergency Department in Piedmont, et al. Lung
ultrasound-implemented diagnosis of acute decom-
pensated heart failure in the ED: a SIMEU multi-
center study. Chest. 2015;148(1):202–10.
10. Volpicelli G, Mussa A, Garofalo G, Cardinale L,
Casoli G, Perotto F, et al. Bedside lung ultrasound in
the assessment of alveolar-interstitial syndrome. Am J
Emerg Med. 2006;24(6):689–96.
11. Jambrik Z, Monti S, Coppola V, Agricola E, Mottola
G, Miniati M, Picano E.  Usefulness of ultrasound
lung comets as a nonradiologic sign of extravascular
lung water. Am J Cardiol. 2004;93(10):1265–70.
12. Lichtenstein DA, Menu Y. A bedside ultrasound sign
ruling out pneumothorax in the critically ill. Lung
sliding. Chest. 1995;108(5):1345–8.
13. Lichtenstein D, Mezière G, Biderman P, Gepner
A. Patient has a pleural effusion A. The lung point: an ultrasound sign specific to pneu-
B. Patient has fluid in the hepatorenal recess mothorax. Intensive Care Med. 2000;4:1434–40.
14. Lichtenstein D, Mezière G, Lascols N, Biderman P,
C. Patient has free fluid surrounding the
Courret JP, Gepner A, Tenoudji-Cohen M. Ultrasound
bladder diagnosis of occult pneumothorax. Crit Care Med.
D. Patient is completely healthy 2005;4:1231–8.
100 A. Sharma and S. Subramani

15. Lichtenstein D, Lascols N, Prin S, Mezière G.  The ysis for the use of ultrasound versus radiology
lung pulse: an early ultrasound sign of complete atel- in diagnosing of pneumonia. Crit Ultrasound J.
ectasis. Intensive Care Med. 2003;4:2187–92. 2017;9:6.
16. Targhetta R, Bourgeois JM, Chavagneux R, 31. Reissig A, Copetti R, Mathis G, Mempel C, Schuler
Marty-Double C, Balmes P.  Ultrasonographic A, Zechner P, et al. Lung ultrasound in the diagnosis
approach to diagnosing hydropneumothorax. Chest. and follow-up of community-acquired pneumonia: a
1992;101:931–4. prospective, multicenter, diagnostic accuracy study.
17. Koh DM, Burke S, Davies N, Padley SP. Transthoracic Chest. 2012;142:965–72.
US of the chest: clinical uses and applications. 32. Testa A, Soldati G, Copetti R, Giannuzzi R, Portale
Radiographics. 2002;22(1):e1. G, Gentiloni-Silveri N. Early recognition of the 2009
18. Volpicelli G.  Sonographic diagnosis of pneumotho- pandemic influenza a (H1 N1) pneumonia by chest
rax. Intensive Care Med. 2011 Feb;37(2):224–32. ultrasound. Crit Care. 2012;16:R30.
19. Alrajhi K, Woo MY, Vaillancourt C. Test characteris- 33. Lissaman C, Kanjanauptom P, Ong C, Tessaro M,
tics of ultrasonography for the detection of pneumo- Long E, O'Brien A.  Prospective observational study
thorax: a systematic review and meta-analysis. Chest. of point-of-care ultrasound for diagnosing pneumo-
2012;141(3):703. nia. Arch Dis Child. 2019;104:12–8.
20. Copetti R, Soldati G, Copetti P. Chest sonography: a 34. Tsai NW, Ngai CW, Mok KL, Tsung JW. Lung ultra-
useful tool to differentiate acute cardiogenic pulmo- sound imaging in avian influenza a (H7 N9) respira-
nary edema from acute respiratory distress syndrome. tory failure. Crit Ultrasound J. 2014;6:6.
Cardiovasc Ultrasound. 2008;6:16. 35. Tsung JW, Kessler DO, Shah VP. Prospective appli-
21. Bouhemad B, Zhang M, Lu Q, Rouby JJ.  Clinical cation of clinician-performed lung ultrasonography
review: bedside lung ultrasound in critical care prac- during the 2009 H1 N1 influenza a pandemic: dis-
tice. Crit Care. 2007;11:205. tinguishing viral from bacterial pneumonia. Crit
22. Pneumatikos I, Bouros D.  Pleural effusions in criti- Ultrasound J. 2012;4:16.
cally ill patients. Respiration. 2008;76:241–8. 36. Shen MS, Yin T, Ji XL.  Pathological diagnosis and
23. Roch A, Bojan M, Michelet P, Romain F, Bregeon differential diagnosis of severe acute respiratory syn-
F, Papazian L, et  al. Usefulness of ultrasonogra- drome. J Clin Exp Pathol. 2003;19:387–9.
phy in predicting pleural effusions>500 mL in 37. Xu Z, Shi L, Wang Y, Zhang J, Huang L, Zhang C,
patients receiving mechanical ventilation. Chest. et  al. Pathological findings of COVID-19 associated
2005;127:224–32. with acute respiratory distress syndrome. Lancet
24. Via G, Storti E, Gulati G, Neri L, Mojoli F, Braschi Respir Med. 2020;8:420–2.
A.  Lung ultrasound in the ICU: from diagnostic 38. Zhu ZX, Lian XH, Zeng YM, Wu WJ, Xu ZR, Chen
instrument to respiratory monitoring tool. Minerva YJ, et al. Point-of-care ultrasound- a new option for
Anestesiol. 2012;78:1282–96. early quantitative assessment of pulmonary edema.
25. Lichtenstein DA, Mezière GA.  Relevance of lung Ultrasound Med Biol. 2020;46:1–10.
ultrasound in the diagnosis of acute respiratory fail- 39. Zeng LQ, Lyu GR, Lian XH, Zhu ZX, Chen YJ, Xu
ure: the BLUE protocol. Chest. 2008;134:117–25. ZR, Guo YN.  Study on the correlation between B
26. Reissig A, Kroegel C.  Transthoracic ultrasound line in ultrasound and severity of pulmonary edema.
of lung and pleura in the diagnosis of pulmonary Chinese J Ultrasound Med. 2019;35:272–4.
embolism: a novel non-invasive bedside approach. 40. Yang PC. Ultrasound-guided transthoracic biopsy of
Respiration. 2003;70:441–52. the chest. Radiol Clin N Am. 2000;38(2):323–43.
27. Wang XT, Liu DW, Zhang HM, He HW, Liu Y, Chai 41. Sheth S, Hamper UM, Stanley DB, Wheeler JH,
WZ, et  al. The value of bedside lung ultrasound in Smith PA.  US guidance for thoracic biopsy: a valu-
emergency-plus protocol for the assessment of lung able alternative to CT. Radiology. 1999;210:721–6.
consolidation and atelectasis in critical patients. 42. Hendrikse K, Gramata J, ten Hove W, Rommes J,
Zhonghua Nei Ke Za Zhi. 2012;51:948–51. Schultz M, Spronk P.  Low value of routine chest
28. Mongodi S, Via G, Girard M, Rouquette I, Misset B, radiographs in a mixed medical-surgical ICU. Chest.
Braschi A, Mojoli F, Bouhemad B. Lung ultrasound 2007;4:823–8.
for early diagnosis of ventilator-associated pneumo- 43. McGonigal MD, Schwab CW, Kauder DR,
nia. Chest. 2016;149:969–80. Miller WT, Grumbach K.  Supplemented emer-
29. Ho MC, Ker CR, Hsu JH, Wu JR, Dai ZK, Chen gent chest CT in the management of blunt torso
IC. Usefulness of lung ultrasound in the diagnosis of trauma. J Trauma. 1990;4:1431–5. https://doi.
community-acquired pneumonia in children. Pediatr org/10.1097/00005373-­199012000-­00001.
Neonatol. 2015;56:40–5. 44. Lauer MS. Elements of danger - the case of medical
30. Alzahrani SA, Al-Salamah MA, Al-Madani W, imaging. N Engl J Med. 2009;4:841–3. https://doi.
Elbarbary MA.  Systematic review and meta-anal- org/10.1056/NEJMp0904735.
4  Point-of-Care Ultrasound of the Lungs 101

45. Lichtenstein D, Hulot JS, Rabiller A, Tostivint I, ment of pleural effusion in critically ill patients
Mezière G. Feasibility and safety of ultrasound-aided by means of ultrasonography. Crit Care Med.
thoracentesis in mechanically ventilated patients. 2005;4:1757–63.
Intensive Care Med. 1999;4:955–8. 49. Bouhemad B, Brisson H, Le-Guen M, Arbelot C, Lu
46. Lichtenstein D, Mezière G, Seitz J.  The dynamic Q, Rouby JJ. Ultrasound assessment of positive end-­
air bronchogram. An ultrasound sign of alveo- expiratory pressure-induced lung recruitment. Am J
lar consolidation ruling out atelectasis. Chest. Respir Crit Care Med. 2011;4:341–7.
2009;4:1421–5. 50. Oveland NP, Lossius HM, Wemmelund K, Stokkeland
47. Lerolle N, Guérot E, Dimassi S, Zegdi R, Faisy C, PJ, Knudsen L, Sloth E. Using thoracic ultrasonogra-
Fagon JY, Diehl JL.  Ultrasonographic diagnosis cri- phy to accurately assess pneumothorax progression
terion for severe diaphragmatic dysfunction after car- during positive pressure ventilation. A comparison
diac surgery. Chest. 2009;4:401–7. with CT scanning. Chest. 2013;4(2):415–22.
48. Vignon P, Chastagner C, Berkane V, Chardac E,
Francois B, Normand S, et  al. Quantitative assess-
Point-of-Care Ultrasound
of the Heart: Transthoracic 5
Echocardiogram

Rohit Vijay Agrawal, Sudipta Mukherjee,
Chang Chuan Melvin Lee,
Arunangshu Chakraborty, and Manojit Lodha

5.1 Echocardiography phy is increasingly employed by nontraditional


practitioners of echocardiography, and a limited
5.1.1 Background structural assessment of the heart and inferior
vena cava (IVC) can be performed by acute medi-
Point-of-care ultrasonography (PoCUS) is cine practitioners such as emergency physicians,
increasingly employed in a variety of clinical set- intensivists, and anesthesiologists.
tings—particularly in emergency departments An evolving patient care landscape, coupled
and critical care units. A part of PoCUS is point-­ with increased availability of ultrasound has made
of-­care echocardiography which may be employed bedside echocardiography an important tool in
to provide rapid diagnosis for respiratory and cir- patient assessment. The role of point-of-­care testing
culatory failure or the etiology of cardiac arrest, improves efficiency and can improve patient out-
answer specific clinical questions, and facilitate comes [1]. The role of point-of-care echocardiogra-
goal-directed care, such as fluid management or phy is to answer specific clinical questions in a
inotropic support. Point-of-care echocardiogra- dichotomous manner in order to facilitate early
diagnosis and guide therapy. Point-of-care echocar-
diography is typically one part of a PoCUS proto-
R. V. Agrawal
National University Health System, col, and the findings should be considered together
Singapore, Singapore with the rest of the ultrasound examination, such as
Surgical Intensive Care Unit, National University lung and vascular ultrasound. The findings obtained
Health System, Singapore, Singapore from point-of-­care echocardiography also have to
S. Mukherjee be interpreted in conjunction with bedside clinical
Tata Medical Center, Kolkata, West Bengal, India examination findings and laboratory data.
C. C. M. Lee (*) While the conduct of a comprehensive diagnos-
National University Health System, tic echocardiogram is well beyond the scope of this
Singapore, Singapore book, this chapter aims to serve as a primer for
A. Chakraborty anesthetic and critical care trainees to understand
Department of Anaesthesia, Critical Care and Pain, essential sonographic principles underlying echo-
Tata Medical Center, Newtown, Kolkata, West cardiography, and point-of-care assessment of car-
Bengal, India
diac function to identify life-threatening pathology
M. Lodha and basic assessment of cardiac function to aug-
Tata Medical Center, Kolkata, West Bengal, India
ment the bedside cardiac examination and make
Vivekananda Institute of Medical Sciences, better-informed decisions for patient care.
Kolkata, West Bengal, India

© The Author(s), under exclusive license to Springer Nature Singapore Pte Ltd. 2022 103
A. Chakraborty, B. Ashokka (eds.), A Practical Guide to Point of Care Ultrasound (POCUS),
https://doi.org/10.1007/978-981-16-7687-1_5
104 R. V. Agrawal et al.

It is also important to understand the limita- ducers are required, but these are beyond the
tions of point-of-care echocardiography. Unlike a scope of this book.
comprehensive echocardiographic study, the Transducer orientation. All transducers have
point-of-care echocardiography does not acquire a marker on one side of the probe which corre-
images from all possible windows and views, it is sponds with the orientation marker on the image
mainly a qualitative assessment, just sufficient to produced on the ultrasound screen [4]. This is
answer the specific clinical query by detecting relevant as the transducer orientation marker is
pathological states relevant to the clinical setting. important in view acquisition during echocar-
Furthermore, the impact of inter-individual varia- diography. In addition, the operator may notice
tion cannot be discounted as the study is inter- that, by convention, the transducer orientation
preted by providers with a wide range of expertise marker on the ultrasound screen is on the left of
and skill levels. Lastly, the study shows a single the image, while this is reversed for echocardiog-
snapshot in the course of a patient’s care and raphy (i.e., on the right of the screen).
repeated reassessment to evaluate the effective- Spectral Doppler. Spectral Doppler imaging
ness of treatment or clinical trajectory may be is important in echocardiography, as it allows for
required. At present, most hemodynamic calcula- the measurement of blood flow velocity to calcu-
tions, such as stroke volume and ejection frac- late pressure gradients and flow. Two forms of
tion, require manual operator measurement or spectral doppler exist—pulsed wave Doppler
assessment, and thus if continuous cardiovascu- (PW) which uses short bursts of ultrasound, or
lar and hemodynamic assessment is required, it is range gating to measure velocities in a specific
not the optimal tool for monitoring. area or depth (sample volume). However, at high
velocities above the Nyquist limit range ambigu-
5.1.1.1 Sonography Principles ity, or aliasing occurs, precluding measurement
and Instrumentation by PW, which is the main disadvantage of
The fundamentals of knobology and ultrasound PW.  The Nyquist limit, defined as (Nyquist
physics have been covered elsewhere in this limit = pulse repetition frequency/2), is the maxi-
book, and this section will briefly cover 3 areas mum velocity that can be measured by PW. This
which are particularly relevant to bedside can be increased by adjusting the baseline shift
echocardiography. and scale. Continuous wave Doppler (CW) in
Phased array transducers. Echocardiogra- contrast, utilizes continuously emitting and
phy probes, unlike most other PoCUS methods, receiving crystals to measure velocities across
utilizes phased array transducers with adjustable the entire scan line. Generally speaking, CW dis-
focus and steering. These transducers have mul- plays the highest velocity along the scan line.
tiple piezoelectric elements which are electroni- While this allows it to have no maximum veloc-
cally separate [2]. In newer pocket-sized handheld ity, the signal is not gated, and localization of the
devices, the piezoelectric elements are replaced signal is not possible.
by a novel ultrasound-on-chip technology which
generates a voltage through a membrane that 5.1.1.2 Normal Transthoracic
generates ultrasound waves [3]. Individual ele- Echocardiography (TTE)
ments are fired sequentially to steer the ultra- The American Society of Echocardiography
sound beam by creating a radially propagating (ASE) divides transthoracic echocardiography
scan line. This allows a probe with a small foot- (TTE) into different categories—the ultrasound-­
print to allow imaging of a wider field of view assisted physician examination, cardiac PoCUS,
which is essential in cardiac imaging. The small critical care echocardiography, limited echocar-
square footprint also helps to be accommodated diography, and comprehensive echocardiography
within intercostal space and to avoid obstruction [5]. The focused TTE is incorporated as part of
by the ribs. In order to produce three-dimensional several protocols such as the Rapid Ultrasound
echocardiography, specialized matrix array trans- for Shock and Hypotension (RUSH) and
5  Point-of-Care Ultrasound of the Heart: Transthoracic Echocardiogram 105

a b

Fig. 5.1 (a) Probe positioning for the parasternal long-­ (lower half of the image) to the right ventricle, and the
axis view, with the transducer orientation marker pointing mitral and aortic valves can be seen towards the right of
towards the patient’s left shoulder. (b) The parasternal the image
long-axis view, with the left ventricle shown posterior

Sonography in Hypotension and Cardiac Arrest


(SHoC) protocols [6–9]. While each protocol has
its own philosophy and utilizes a different per-
mutation of views, they share the same basic
views covered below.
Parasternal long-axis view (PLAX). Cardiac
imaging traditionally commences with the para-
sternal windows. Although some echocardiogra-
phy laboratories perform echocardiography with
the patient in the left lateral decubitus position,
this is not practical in most scenarios, especially
in critically ill patients, where PoCUS is often
performed. Nonetheless, a semi-rotated position
Fig. 5.2  M-Mode echocardiography demonstrating the
towards the left, by placing a pillow under the E-Point Septal Separation (EPSS) acquired from the para-
patient’s right back and hip, may help improve sternal long-axis view
image acquisition. The initial probe position is
just to the left of the sternum, in the third or (Fig. 5.2) [10, 11]. The anteroseptal and infero-
fourth intercostal space (Fig.  5.1a). Acquisition lateral walls of the left ventricle are visualized in
of the PLAX view is obtained with the alignment this view, although the cardiac apex is usually
of the transducer orientation marker pointing poorly visualized. Tilting the probe towards the
towards the patient’s right shoulder. In this view patient’s right hip allows visualization of the
(Fig. 5.1b), the right ventricle is seen anteriorly, right ventricular inflow view in which the tricus-
and the left ventricle and atrium inferiorly. The pid valve and its leaflets can be visualized. From
mitral and aortic valves should be clearly visual- this view, Doppler interrogation of the tricuspid
ized towards the right of the image, and M-Mode valve can be performed.
echocardiography directed towards the anterior Parasternal short-axis view (PSAX).
mitral leaflet from this view can be used to Rotation of the probe 90° clockwise from the
acquire the E-Point Septal Separation (EPSS)—a PLAX view is probably the most efficient way of
semiquantitative measure of left ventricular sys- obtaining the PSAX view. The transducer orien-
tolic function covered later in this chapter tation marker should now point towards the
106 R. V. Agrawal et al.

patient’s left shoulder. The PSAX view demon- illary muscles. This view is attractive to PoCUS
strates different structures at different levels, of providers as it allows visual estimation of left
which there are five different scan planes. ventricular systolic function, myocardial thicken-
Generally, most PoCUS providers will focus on ing, and movement of all six wall segments at this
the mid-ventricular SAX view (Fig.  5.3a, b) level, as well as LV filling. From this view,
where LV is seen in cross-section with the 2 pap- M-Mode sonography can be aligned perpendicu-

a b

c d

Fig. 5.3 (a) Probe positioning for the parasternal short-­ (c) The parasternal short-axis view at the basal level,
axis view, with the transducer orientation marker point- demonstrating the “fish mouth” appearance of the mitral
ing towards the patient’s left shoulder. (b) The parasternal valve. (d) The aortic valve short-axis view. (e) The para-
mid-ventricular short-axis view, with the left ventricle sternal short-axis view at the level of the cardiac apex
shown in cross-section with the two papillary muscles.
5  Point-of-Care Ultrasound of the Heart: Transthoracic Echocardiogram 107

lar to the inferior wall passing through the center tation marker pointing towards the patient’s left
of the LV cavity allow assessment of LV end-­ shoulder (Fig. 5.4a). The probe is directed towards
diastolic and end-systolic diameter, which can the patient’s right shoulder and then moved later-
also be used to calculate fractional shortening ally until the cardiac apex is visualized in the
and estimate ejection fraction based on geometric middle of the upper part of the screen. The probe
formulae such as Teicholz’s formula or the modi- should then be tilted until the interventricular sep-
fied Quinone’s equation [12–14]. This is covered tum runs down the middle of the image. This view
further later in this chapter. shows all 4 cardiac chambers and allows the oper-
From the mid-ventricular view, the probe can ator to estimate the relative sizes of the right- and
be angled cephalad towards the base of the heart left-sided chambers, and visualize both atrioven-
and first produces the “fish mouth” appearance of tricular valves (Fig.  5.4b). Application of color
the mitral valve (Fig. 5.3c). The anterior and pos- flow Doppler allows the detection of mitral and/or
terior mitral valve leaflets can be visualized in tricuspid regurgitation, or turbulent flow sugges-
this view, with the anterior leaflet of the mitral tive of stenosis. Furthermore, this view allows for
valve towards the top of the screen (anteriorly) good alignment for continuous wave Doppler
and the posterior leaflet below. The anterior and interrogation of a tricuspid regurgitation jet for
posterior mitral valve leaflets are each divided estimation of right ventricular systolic pressure.
into three scallops (A1–A3 and P1–P3). A1 and Both left and right ventricular systolic function
P1 are on the right of the image and the A3 and can be estimated or measured in this view.
P3 scallops are on the left. This view is of limited Tricuspid annulus peak systolic excursion
clinical utility in the context of PoCUS. (TAPSE) (Fig. 5.4c), a surrogate measure of right
The probe is further directed cephalad to ventricular systolic function, can be measured in
obtain the aortic valve short-axis view (Fig. 5.3d), this view by aligning M-Mode ultrasound to the
in which the three cusps of the aortic valve appear lateral tricuspid annulus. In this view, the antero-
symmetrical. The tricuspid valve can be seen to lateral and inferoseptal walls of the left ventricle
the left of this image, and the pulmonary valve can be seen. The presence of any pericardial col-
anterior to the aortic valve. The anatomy of the lection can also be seen from this view.
aortic valve can be assessed in this view, as well A cephalad and leftward redirection of the probe
as the presence of any tricuspid regurgitation. from the A4C view produces the apical 5-chamber
Doppler interrogation of the tricuspid valve can view which allows for visualization of the left ven-
be performed in this view. Further cephalad tricular outflow tract and aortic valve, from which
direction of the probe from the aortic valve short-­ Doppler measurements can be performed.
axis view brings the main pulmonary artery into The A4C view is important as it forms the
view, although this is of limited clinical utility, basis from which additional views such as the
except in the rare case of a massive acute pulmo- apical 2-chamber, apical 3-chamber, and apical
nary embolism, in which a thrombus may be vis- 5-chambers views can be obtained, although
ible in the main pulmonary artery. these are generally not part of PoCUS protocol,
Caudal direction of the probe allows assess- but necessary for advanced measurements and
ment of the cardiac apex (Fig.  5.3e) for wall assessment of global left ventricular function.
motion abnormalities and ballooning, of which Apical 2-chamber (A2C). A 90° counter-
the latter may indicate Takotsubo cardiomyopa- clockwise rotation of the probe from the A4C view
thy. Otherwise, this view does not yield much generates the A2C view (pic), which allows assess-
clinical information for PoCUS providers. ment of the anterior and posterior walls of the left
Apical 4-chamber view (A4C). The A4C is ventricle for wall motion abnormalities, as well as
obtained at the cardiac apex—which can vary sig- left ventricular function. Furthermore, biplane
nificantly between individuals. This view is usu- measurements of left ventricular systolic function,
ally obtained by starting just inferolaterally to the such as Simpson’s biplane, require tracing end-
left nipple or left breast, with the transducer orien- diastolic and end-systolic endocardial borders of
108 R. V. Agrawal et al.

a b

Fig. 5.4 (a) Probe positioning for the apical 4-chamber tricuspid valve peak systolic excursion (TAPSE), mea-
view, with the transducer orientation marker pointing sured by M-Mode echocardiography aligned to the lateral
towards the patient’s left. (b) The apical 4-chamber view tricuspid annulus
with the right ventricle on the left of the screen. (c) The

the left ventricle in orthogonal views; thus, mea- supine position and legs bent at the knees to relax
surements performed in the A4C view will need to abdominal muscles. Subcostal window is particu-
be complemented by measurements in the A2C larly important in those patients where apical and
view. Further counterclockwise ­ rotation of the parasternal windows are difficult to obtain, like
probe from the A2C position generates the apical the patients on ventilator and COPD patients.
3-chamber (A3C) or apical long-axis view, analo- From this window, a 4-chamber view can be
gous to the PLAX view. In this view, the antero- obtained to visualize any pericardial collections
septal and inferolateral walls of the left ventricle, and allow gross estimation of chamber sizes, ven-
as well as the left ventricular outflow tract and aor- tricular filling, and systolic function (Fig. 5.5b).
tic valve can be visualized, and unlike the paraster- Alignment also usually does not allow for ade-
nal long axis, allows for alignment of a Doppler quate alignment for quantitative techniques
beam for pulsed wave and continuous-wave mea- which use pulsed wave or continuous wave
surements of flow across the left ventricular out- Doppler. The apex is also typically not well visu-
flow tract and aortic valve. Unlike the PLAX view, alized, only the septal and lateral walls of the left
the A3C view usually allows for visualization of ventricle, precluding assessment of regional wall
the apex of the left ventricle. motion abnormalities, and limiting the utility of
Subcostal window. The subcostal window this view to visual estimates of function.
can be accessed by placing the transducer in the Nonetheless, application of color flow Doppler in
subxiphoid space (Fig.  5.5a) with patient in this view can still allow for visualization of gross
5  Point-of-Care Ultrasound of the Heart: Transthoracic Echocardiogram 109

a b

Fig. 5.5 (a) Probe positioning for the subcostal view, screen), and the left ventricle and atrium seem posterior.
with the transducer orientation marker pointing to the The liver and inferior vena cava can be seen on the left of
patient’s left. (b) The subcostal 4-chamber view. The right the screen
ventricle and atrium and anterior (upper half of the

valvular pathology such as regurgitation, as well


as flow across the interatrial or interventricular
septum. As it is comparatively easier to obtain a
subcostal window compared to the parasternal or
apical windows, especially in ventilated patients,
modifications of other views are sometimes
obtained from this window. Rotation of the probe
counterclockwise produces a subcostal short-axis
view that cuts through the mid-papillary region
of the left ventricle that can allow for assessment
of volume status. A modified short-axis view of
the aortic valve can also be obtained from the
Fig. 5.6  The subcostal inferior vena cava view
subcostal window.
Inferior vena cava (IVC) view. This is a
modification of the subcostal window. A cepha- needs to be differentiated from, and can be easily
lad orientation of the transducer from subcostal done so by its pulsations relative to the QRS
long-axis view will create IVC view (Fig.  5.6). complex of the ECG, or the patient’s pulse based
The orientation marker is directed towards the on the arterial waveform or plethysmography.
head of the patient (cephalad). It will identify the Furthermore, the abdominal aorta has a thicker
IVC draining into the right atrium. Approximately wall, with less respiratory variability and does
3  cm from the junction of inferior vena cava not drain into the right atrium; this can be further
(IVC) and right atrium, the hepatic vein can be confirmed by color flow or spectral Doppler
visualized to drain into IVC.  This anatomical interrogation.
landmark is important as all the measurements of Suprasternal window. The suprasternal view
IVC in terms of volume status is supposed to be is usually not performed as part of PoCUS. It is
done just proximal to drainage point of hepatic acquired with the probe in the suprasternal notch
vein and is standardized at this level. and the transducer orientation marker pointing
Measurements of IVC diameter can be performed cephalad in a 12 o’clock position. In this view,
with M-Mode echocardiography (Fig.  5.7). The the ascending aorta, aortic arch, and part of the
abdominal aorta that is situated next to the IVC, proximal descending thoracic aorta can be clearly
110 R. V. Agrawal et al.

indications such as during open-heart surgery,


and catheter-based cardiac interventions. The
American Society of Echocardiography has put
forward a consensus statement detailing the 11
basic views required for a basic perioperative
TOE examination, which are listed below
(Table  5.1) with their approximate acquisition
angles [18]. The basic perioperative TOE exami-
nation is a noncomprehensive examination aimed
at assessing hemodynamic function, and exclu-
sion of life-­threatening pathology.

Fig. 5.7  Measurements of inferior vena cava diameter


5.1.1.4 Left Ventricular Systolic
using M-Mode echocardiography
Function Assessment
visualized, as well as the right pulmonary artery Left ventricular systolic function should be
inferior to the aortic arch, and the aortic arch assessed in the presence of undifferentiated shock
branches—the innominate artery, left common as it has important implications for treatment.
carotid artery, and left subclavian artery. This Patients with left ventricular systolic dysfunction
view allows for the identification of a proximal and a dilated left ventricle may benefit from ino-
aortic aneurysm or dissection. tropic support, rather than further fluid resuscita-
tion. Furthermore, the presence of left ventricular
5.1.1.3 Normal Transesophageal systolic dysfunction is of prognostic value, and
Echocardiography (TOE) its presence portends worse outcomes.
There is increasing interest in the use of trans- The assessment of left ventricular systolic
esophageal echocardiography (TOE) in both function is commonly assessed in the PLAX,
acute care and perioperative medicine, due to its PSAX, and A4C views. In patients with poor
ability to circumvent poor transthoracic windows transthoracic windows (chronic obstructive pul-
from positive pressure ventilation, chest wall monary disease, left-sided pneumothorax, left-­
injuries, and dressings, and provide good visual- sided thoracic surgery, etc.), the subcostal
ization of cardiac structures [15, 16]. 4-chamber view may also provide useful infor-
Unlike transthoracic echocardiography, the mation. Visual estimates of ejection fraction are
TOE probe consists of a single ultrasound array most commonly performed as part of PoCUS as
that can be rotated around the long axis of the traditional methods of quantifying ejection frac-
beam from 0 to 180°. The probe is inserted to tion, such as Simpson’s biplane or fractional area
various depths, and a multiplane angle is applied change are time-consuming, require training, and
to rotate the beam in order to view cardiac struc- a subject of interobserver variability. Furthermore,
tures across different scan planes [17]. studies have demonstrated the feasibility and
Due to its small risk of serious complications, accuracy of qualitative visual estimates of ejec-
formal training in probe handling and manipula- tion fraction in the management of the critically
tion is currently required by many centers. The ill [19].
indications for TOE are diverse. These range Quantitative measures of left ventricular sys-
from hemodynamic monitoring, such as in basic tolic function can be obtained from
perioperative TOE, to structural heart assessment PoCUS.  Fractional shortening (FS) of the left
of pathologies (such as valvular abnormalities) ventricle can be measured in the PLAX or PSAX
not achievable by TTE, to specific procedural views via M-Mode echocardiography which
5  Point-of-Care Ultrasound of the Heart: Transthoracic Echocardiogram 111

Table 5.1  The 11 basic views which constitute a basic perioperative transesophageal echocardiographic examination
Structures visualized and acquisition from previous
View Approximate depth and angle view (if any maneuvers required)
ME 4-chamber view ME, 30–40 cm Both atria and ventricles, atrioventricular valves
0–10° (mitral and tricuspid) interatrial septum, and
interventricular septum
Inferoseptal and anterolateral walls of the LV
Right-sided chambers are seen on the right side of
the image
ME 2-chamber view ME, 30–40 cm Left atrium and ventricle, left atrial appendage, and
00–100° mitral valve
Anterior and posterior walls of the LV are seen
ME long-axis view ME, 40–50 cm Left atrium and ventricle, mitral valve, aortic valve,
120–140° and left ventricular outflow tract
Aortic root and proximal ascending aorta
Anteroseptal and inferolateral walls of the LV
Right ventricle and right ventricular outflow tract in
the far field of the image
ME ascending aortic ME, 30–40 cm Transducer withdrawn slightly from the ME
long-axis view 90–110° long-axis view
The mid-ascending portion of the aorta and right
pulmonary artery is seen
ME ascending aortic ME, 30–40 cm The mid-ascending portion of the aorta and main
short-axis view 20° pulmonary artery and its bifurcation, as well as the
superior vena cava. The left pulmonary artery is not
well visualized
ME aortic valve short-axis ME, 30–40 cm Probe advanced from the ME ascending aortic
view 0–30° short-axis view
Aortic valve, interatrial septum, left main coronary
artery, both atria, pulmonary valve, and the right
ventricular outflow tract can be seen
ME RV inflow-­outflow view Me, 30–40 cm Aortic valve, interatrial septum, both atria,
60–80° pulmonary and tricuspid valves, and the right
ventricular outflow tract are visualized
ME bicaval view ME, 30–40 cm Left and right atria, interatrial septum, superior and
90–110° inferior vena cavae on the right and left of the
screen, respectively. The tricuspid valve may be seen
in the far field with increasing angles
TG mid-­papillary short-axis TG, 40–45 cm Probe advanced from the mid-oesophageal views
view 0–20° The LV, papillary muscles, and all six walls at the
mid-­ventricular level. Part of the RV may be seen
Descending aortic short-axis ME to TG, 30–40 cm The probe is withdrawn to a ME to TG level
view 0° The descending thoracic aorta and left hemithorax is
seen
Descending aortic long-axis ME to TG, 30—40 cm The descending thoracic aorta and left hemithorax is
view 90° seen
Abbreviations: ME midesophageal, RV right ventricle, TG transgastric, LV left ventricle

allows the calculation of LV end-diastolic and ening is a unidimensional assessment and is not
end-systolic dimensions (Fig.  5.8). Its normal clinically appropriate in the presence of regional
value is >30%. Usually, ejection fraction (EF) is wall motion abnormalities or septal dyskinesia.
considered to be double of FS. Fractional short- Fractional area change (FAC) can be measured in
112 R. V. Agrawal et al.

the PSAX view (mid-papillary segment). This is Simpson’s biplane method of disks (from tracing
a two-dimensional assessment of left ventricular endocardial borders). Both Teicholz’s formula
area change during end-systole and end-diastole, and the modified Quinone’s equation have their
the end-systolic area (ESA) and end-diastolic limitations as linear measurements which make
area (EDA), respectively. Normally it is 35–65%, assumptions regarding the shape of the left ven-
and is calculated by FAC = (EDA − ESA) × 100/ tricle and thus may be inaccurate in situations
EDA.  Ejection fraction, the difference between where there is asymmetrical left ventricular con-
end-diastolic volume and end-systolic volume of traction such as ischemic heart disease. An alter-
the left ventricle can be calculated by geometric native is to use Simpson’s biplane method of
formulae (Table 5.2), such as the Teicholz’s equa- disks, which requires tracing of LV endocardial
tion or the modified Quinone’s equation (from borders in orthogonal views (i.e., 4-chamber and
M-Mode echocardiography) or the modified 2-chamber views) in both end-diastole and end-­
systole. This method divides the LV cavity into
an a priori defined number of disks (usually 20),
and the volume of each disk is calculated based
on the tracing of the LV cavity [12–14, 20, 21].
Another semiquantitative measure of left ven-
tricular systolic function easily obtained by echo-
cardiography is the E-point septal separation
(EPSS) which is the minimum distance between
the tip of the anterior mitral valve leaflet endo-
cardial border of the interventricular septum.
The normal value of the EPSS is <7 mm, which
corresponds with a normal ejection fraction
[10, 11]. Based on magnetic resonance imaging
studies, ejection fraction may be estimated
Fig. 5.8  M-Mode echocardiography demonstrating the
left ventricular cavity. From this view, left ventricular end-­ from EPSS using the equation ejection
diastolic and end-systolic dimensions can be measured, fraction = 75.5 – (2.5 × EPSS).
and calculations of fractional shortening or ejection frac- Apart from the estimate of ejection fraction,
tion performed the operator should also consider wall motion

Table 5.2  Teicholz’s formula and the modified Quinone’s equation [12–15]
Formula Description
Teicholz’s formula:
Vol = 7D3/(2.4 + D), where D is the LV diameter (mm) Calculates LV volume using only LV diameter (D), and
LVEF = (EDV − ESV)/EDV thus its accuracy is based on geometric assumptions
regarding the shape of the LV
Measurements are performed in end-diastole and
end-systole in order to calculate the EDV and ESV,
respectively
Modified Quinone’s equation:
%ΔD2 = (LVEDD2 − LVESD2)/LVEDD2 Calculates LVEF without planimetry by performing 8
LVEF = (%ΔD2) + [(1 − %ΔD2)(%ΔL)] averaged measurements of the LV internal diameter at
Where LVEDD is the averaged LV end-diastolic different levels in the parasternal long-axis, apical 4
dimension (mm), and LVESD is the averaged LV chamber, and long-axis views at end-diastole (LVEDD)
end-systolic dimension (mm) and end-systole (LVESD). The formula further corrects
%ΔL: Correction for apical contraction for apical contraction visualized in the long axis (%ΔL)
+15% Normal apex as with Teicholz’s formula, the modified Quinone’s
+5% Hypokinetic apex equation is a linear measurement based on geometric
+0% Akinetic apex assumptions, which may be inaccurate in the context of
−5% Slightly dyskinetic apex LV distortion (e.g., asymmetrical wall motion
−15% Frankly dyskinetic apex abnormalities, focal aneurysms)
5  Point-of-Care Ultrasound of the Heart: Transthoracic Echocardiogram 113

Apical view:
17 17 17
14 15 13 14
16 16

11
12 10 Perfusion of segments:
7 8
9 1 – LAD
5 2 – LAD
4 2 3 – RCA
3 6 1
4 – RCA
5 – Cx or RCA
4-chamber 2-chamber 3-chamber 6 – Cx or LAD
7 – LAD
8 – LAD
9 – RCA & LAD
Parasternal short axis (psax): 10 – RCA
11 – Cx or RCA
1 7
13 12 – Cx or RCA
6 8 12 13 – LAD
14 – LAD
2 15 – LAD
14 16
16 – Cx or LAD
3 17 – LAD
9 11
5 15
4 10
Base Mid-vantricular Apex

Fig. 5.9  The 17-segment model and its correspondence model and coronary arterial anatomy using contrast-­
to perfusion by coronary arteries depicts the fundamental enhanced cardiac magnetic resonance imaging.
methodological approach. Ortiz-Pérez JT, Rodríguez J, Cardiovasc Imaging. 2008; 1:282–93 [23]
Meyers SN, et al. Correspondence between the 17-­segment

abnormalities. The wall segments of the left changes and rise in cardiac markers, but the
ventricle should exhibit symmetrical systolic characteristic ECHO finding of apical balloon-
excursion towards the center of the cavity, and ing is distinctive. Regional wall motion abnor-
the myocardium should increase in thickness by malities may be briefly evaluated with the PSAX
>30% during systole. These findings are easily view at the apical, mid-ventricular, and basal
assessed from the PSAX view, as all six seg- levels. However, a comprehensive evaluation
ments are visible. Evaluation for the presence of also requires assessment with the A4C, A2C,
regional wall motion abnormalities (RWMA) is A3C, and PLAX views. Depending on the
important for the diagnosis of acute coronary occluded vessel (left anterior descending, right
syndrome. Rise in cardiac biomarkers is delayed coronary, or left circumflex artery), different
and time is the key in critically ill patients. areas of the left ventricular wall may show
Furthermore cardiac biomarkers, such as hypokinesia, dyskinesia, or akinesia (Fig.  5.9;
Troponin I can be elevated from other causes Table 5.2) [22, 23].
such as acute kidney injury, pulmonary embo-
lism, and sepsis which need to be distinguished 5.1.1.5 Left Ventricular Diastolic
from acute coronary syndrome. ECG changes Function
can also be a delayed finding, that makes echo- When considering left ventricular function, far
cardiography important for early evaluation of more attention is paid to systolic function com-
acute coronary syndrome. The presence of pared to diastolic function. In diastolic dysfunc-
RWMAs may necessitate early coronary angi- tion, there is impaired relaxation and
ography and intervention. In critically ill noncompliance of the left ventricle (lusitropy).
patients stress cardiomyopathy can be similar to The long-standing consequence is an elevated left
the presentation as ACS with similar ECG ventricular filling pressure, left atrial pressure,
114 R. V. Agrawal et al.

and left atrial dilatation. Measurement of diastolic This results in atrial systole contributing more to
dysfunction can be easily performed by using ventricular filling and a consequently larger A
pulsed wave Doppler to measure mitral inflow wave. The E/A ratio is reversed and E is <  A
velocities. The sample volume is placed at the (E/A < 0.75). The deceleration time is prolonged
coaptation point on an A4C view. This generates (>240 ms) congruent with the impaired lusitropy
an E wave which corresponds to opening of the [24–27].
mitral valve and filling of the LV from the left In grade II diastolic dysfunction, pseudonor-
atrium down a pressure gradient [24]. Diastasis malization is seen as a result of elevated left atrial
then occurs as the left ventricular and atrial pres- pressures. The increased left atrial pressure
sures equilibrate. Atrial systole then generates the results in a pressure gradient that reestablishes
A wave due to reacceleration of transmitral flow flow once the mitral valve opens and the E wave
[24]. Identification of the E and A waves can be is larger due to this increased blood flow. This
done by either observing the relative position to restores a “normal” E/A ratio of >1.0 [24–27].
an ECG trace on the sonography platform (if Grade II diastolic dysfunction should be sus-
applied) or the systolic wave on pulsed wave pected in patients with unexplained dilated LA
Doppler. The A wave occurs after the P wave on and can be unmasked by valsalva maneuver or
the ECG and precedes the systolic wave on pulsed during tissue Doppler interrogation.
wave Doppler. Measurement of time interval from Further diastolic dysfunction results in restric-
the peak of the E wave to its extrapolated return to tive ventricular filling. Due to the elevated left
baseline is known as the deceleration time and is atrial pressures filling a stiff, noncompliant ven-
representative of the time required for left ven- tricle, a high inflow velocity is seen with diastasis
tricular and atrial pressures to equalize [24]. achieved rapidly. Atrial contraction contributes
In normal diastole, the mitral E wave is usu- little to ventricular filling against the elevated left
ally slightly higher than the A wave with a resul- ventricular pressures. The E wave is usually
tant ratio of the E and A waves exceeding 1.0 twice the peak velocity of the A wave (E/A > 2)
(E/A 1–1.5). A normal deceleration time of 180– and the deceleration time is short (<140  ms),
220 ms is seen [24–27]. reflecting the rapid pressure equilibrium and
In grade I diastolic dysfunction or impaired diastasis [24–27] (Table 5.3).
relaxation, the active process of ventricular relax- Grade IV Diastolic dysfunction is restrictive
ation is slowed, with a reduction in the E velocity. cardiomyopathy, which is sonologically the same

Table 5.3  Grading of diastolic dysfunction


Normal Mild (Grade I) Moderate (Grade II) Severe (Grade III)
Pathophysiology ↓ Relaxation ↓ Relaxation and ↑ ↓ Compliance ↑↑ LVEDP
LVEDP
E/A ratio 1–2 <0.8 0.8–2.0 ≥2.0
Valsalva ΔE/A <0.5 ≥0.5 ≥0.5
DT (ms) 150–200 >200 150–200 <150
e´ velocity ≥10 <8 <8 <5
(cm/s)
E/e´ ratio ≤8 ≤8 9–14 ≥15
IVRT (ms) 50–100 ≥100 60–100 ≤60
PVS/PVD =1 S > D S < D S < <D
PVS (ms) <0.35 >0.35 ≥0.35 ≥0.35
Bdur–Adur (ms) <20 <20 ≥30 ≥30
LA volume <34 ml/m2 Mildly enlarged Moderately enlarged Severely enlarged
Clinical Mild diastolic Pseudonormalization, Restrictive filling (reversible)
presentation dysfunction or Increased stiffness of High LAP, noncompliant
impaired relaxation the LV, elevated LAP LV. May be reversible with
phase reduction of preload (e.g.,
diuretics, Valsalva)
5  Point-of-Care Ultrasound of the Heart: Transthoracic Echocardiogram 115

as Grade III, the only difference being it is not However, tracing of endocardial borders
reversible with Valsalva maneuver. may not be practical at the bedside with porta-
ble sonography platforms. During systole, the
5.1.1.6 Right Ventricular Function right ventricular free wall contracts towards
Assessment the apex.
Generally, the left ventricle (LV) receives much Contractility of the right ventricular free wall
more attention during echocardiographic evalu- contributes minimally (15%) to the cardiac out-
ation, particularly the systolic function of the put. It is the vertical annular movement that is the
LV. However, the importance of the right ven- most important contributor of RV systolic func-
tricle (RV) and its assessment has garnered tion. Tricuspid Annular Plane Systolic Excursion
increased recognition. Right ventricular dys- (TAPSE) can be measured by M-Mode echocar-
function has important physiologic and prog- diography with the cursor aligned to the base of
nostic implications [28]. Conditions that may the lateral tricuspid valve annulus. Normal
result in RV dysfunction include primary RV TAPSE value is 16 mm or more. A TAPSE value
failure (e.g., myocardial infarction, severe sep- less than that indicates RV systolic dysfunction
sis) or increased RV afterload (e.g., Pulmonary [29, 30].
hypertension, pulmonary embolism, acute
respiratory distress syndrome). Assessment of 5.1.1.7 Pulmonary Artery Pressures
RV size can guide fluid resuscitation, and is Systolic, mean, and end-diastolic pulmonary
important, as further fluid loading in the con- artery pressures can be estimated by echocar-
text of RV dysfunction can further worsen RV diography.
function, create interventricular septal bowing Right ventricular systolic pressure (RVSP) is
that compromises LV filling, and worsen used as a surrogate measure of pulmonary artery
hemodynamics. systolic pressure (PASP). This is calculated by
Furthermore, certain causes of RV dysfunc- sum of the estimated or measured right atrial
tion necessitate specific therapy. Pulmonary pressure (RAP) and peak gradient (PPG) of the
emboli may require anticoagulation or thrombol- tricuspid regurgitation (TR) jet, expressed as
ysis, and massive saddle embolism with severe RVSP = TR PPG + RAP. The peak TR gradient
obstructive shock may require surgical is obtained by aligning a continuous wave (CW)
­embolectomy. In patients with pulmonary hyper- Doppler beam to the TR jet to measure the peak
tension, specific physiological and pharmacolog- velocity (TR Vmax) and hence gradient by
ical strategies may be required in addition to Bernoulli’s equation, across the tricuspid valve.
treating the underlying etiology—these may Thus, RVSP  =  4(TR Vmax)2  +  RAP [31].
include inhaled nitric oxide (iNO), inodilator Similarly, mean Pulmonary artery  =  Mean TR
administration (e.g., Milrinone, Dobutamine), or gradient (obtained by tracing the TR jet in
pulmonary vasodilator therapy. CW) + RAP. If there is insufficient TR to obtain
Fractional area change (FAC) is a commonly a complete CW Doppler envelope, RVSP(and
employed measure of RV systolic function. End-­ hence PASP) and mean PAP cannot be
diastolic area (EDA) and end-systolic area (ESA) estimated.
can be measured by tracing the endocardial bor- Diastolic PA pressure can be estimated by
ders of the right ventricle, allowing calculation of measuring the end-diastolic velocity of PR (Vpr).
FAC by the formula FAC = (EDA − ESA) × 100/ Diastolic PAP = 4(Vpr)2 + RAP.
EDA. Values of FAC correlate well with RV ejec- In the absence of adequate TR to estimate
tion fraction obtained by cardiac magnetic reso- RVSP, the acceleration time across the pulmonary
nance imaging (CMR) measurements, which are valve can help predict the likelihood of pulmonary
regarded as the gold standard in the quantifica- hypertension, and estimate mean pulmonary artery
tion of RV systolic function. Fractional area systolic pressure (MPAP) by Mahan’s equation. A
change values of <35% indicate RV systolic dys- short pulmonary acceleration time (<92  ms) is
function [29, 30]. suggestive of elevated MPAP.
116 R. V. Agrawal et al.

5.1.1.8 Volume Assessment neous respiration, the vessels collapse on inspira-


Volume status assessment is important to guide tion and expand on expiration; and the minimum
fluid therapy. Knowing when to intensify fluid (IVCDmin) and maximum (IVCDmax) diameters
administration is important. However, volume can be easily measured by M-Mode ultrasound or
loading is detrimental in patients with myocar- direct 2D measurements [35]. The IVC can be
dial dysfunction, or those who are already clini- imaged in the subxiphoid (or subcostal) long-axis
cally overloaded. Although simple clinical view. Measurements are typically performed
examination techniques such as the straight leg 2 cm inferior to the junction of the IVC and the
raise exist, these alone may not provide suffi- hepatic veins, although the IVC may collapse
cient information. Surrogate measures of fluid nonuniformly [36]. In spontaneously breathing
status such as central venous pressure or pulmo- patients, the IVC collapsibility index is used and
nary capillary wedge pressure (PCWP) are inva- calculated by: IVC collapsibility
sive, and may not be accurate. Furthermore, index  =  [(IVCDmax − IVCDmin)/IVCDmax]. An
derived measures of arterial waveform analysis IVC collapsibility index of >50% is associated
such as stroke volume variation can only be with reduced RAP and hypovolaemia [37]. In
interpreted accurately in certain physiological mechanically ventilated patients however the
states. positive intrathoracic pressure transmitted during
Left ventricular assessment. Hypovolaemia inspiration leads to reduced venous return and
may be indicated by a small, under-filled left ven- increased IVC diameter during this phase of the
tricular cavity. This can be visible on the para- respiratory cycle, and thus IVC distensibility is
sternal long- and short-axis views, as well as the used instead of collapsibility. The IVC distensi-
apical and subcostal 4 chamber views. On the bility index is calculated by the same formula,
parasternal short axis, kissing papillary muscles [(IVCDmax − IVCDmin)/IVCDmax]. A distensibility
may be seen. index of cut-off of >18% has been proposed to
Right ventricular assessment. During the indicate fluid responsiveness [38].
bedside echocardiographic examination, the size The diameter of the IVC has also been pro-
of the right ventricle should be noted, particularly posed as an indicator of fluid overload. However,
the approximate ratio of the right ventricle diam- the utility of this has a few caveats. Firstly, the
eter to that of the left ventricle (RV:LV ratio). The diameter of the IVC varies with position and may
RV:LV ratio should be less than 1:1, that is, the be dilated in healthy individuals, such as athletes
RV should not be the same size or larger than the [39, 40]. Nonetheless, a diameter > 2.5 cm may
left ventricle [32, 33]. A dilated right ventricle be considered dilated, and suggestive of elevated
should prompt caution with further fluid loading. right atrial pressure [29, 40–42].
However, the critically ill may have a dilated More about IVC assessment can be found in
right ventricle for other reasons, such as positive Chap. 6: Vascular POCUS.
pressure mechanical ventilation leading to an Superior vena cava assessment. Superior
increased right ventricular afterload, or pulmo- vena cava distensibility has been proposed as a
nary embolism. The latter may result in a distinc- measure of fluid responsiveness as the superior
tive echocardiographic finding of regional right vena cava (SVC) is subject to the same cyclical
ventricular dysfunction known as McConnell’s changes in intrathoracic pressure as the IVC [43,
sign, in which there is akinesia of the mid-free 44]. In their study, Vieillard-Baron et al. proposed
wall but normal motion at the apex [34]. a cut-off of 36% to indicate fluid responsiveness
Inferior vena cava assessment. Both the [43]. However, SVC variability is usually studied
inferior (IVC) and superior vena cavae (SVC) are by transesophageal echocardiography as views of
sensitive to fluid changes. They are collapsible, the SVC are difficult to acquire using transtho-
and their diameters undergo cyclical changes racic echocardiography, and even when obtain-
during both spontaneous respiration and positive able usually does not provide good alignment for
pressure mechanical ventilation. During sponta- M-Mode measurements.
5  Point-of-Care Ultrasound of the Heart: Transthoracic Echocardiogram 117

Doppler assessment. Stroke volume variation scan line, which will occur at the aortic valve
may be used to predict fluid responsiveness. In (and not the LVOT) thus leading to overestima-
one of its more primitive forms, this is visualized tion of SV. Once SV is calculated, a simple prod-
as variation or “swing” in the peak of an arterial uct against heart rate will yield the estimated
waveform that varies with the respiratory cycle. cardiac output.
Stroke volume can be quantified using pulsed
wave Doppler, with the sample volume placed in
the left ventricular outflow tract (LVOT) on the
apical 5-chamber or apical long-axis views.
References
Respiratory variation should ideally be mea- 1. The College of Emergency Medicine. Crowding in
sured over a single respiratory cycle. Although the emergency department. Revised ed. London: The
stroke volume (SV) is calculated using the prod- College of Emergency Medicine; 2014.
uct of the LVOT area and time-velocity integral 2. Lawrence JP.  Physics and instrumentation of ultra-
sound. Crit Care Med. 2007;35(8 Suppl):S314–22.
(VTI) of a pulsed wave Doppler measurement 3. Baribeau Y, Sharkey A, Chaudhary O, et al. Handheld
within the LVOT (i.e., SV = LVOTarea × VTILVOT), point-of-care ultrasound probes: the new gen-
the former does not change, and for the pur- eration of POCUS.  J Cardiothorac Vasc Anesth.
poses of PoCUS, measurement of VTI is ade- 2020;34(11):3139–45.
4. Moore CL.  Chapter 3. Ultrasound orientation. In:
quate. In some cases, simple visualization of the Carmody KA, Moore CL, Feller-Kopman D. eds.
peak velocity variation across the LVOT will Handbook of critical care and emergency ultra-
suffice. In mechanically ventilated patients, a sound. McGraw Hill; https://accessanesthesiology-­
12–20% variation in VTI or peak aortic flow mhmedical-­com.ezproxy.anzca.edu.au/content.aspx?
bookid=517&sectionid=41066789. Accessed 25 June
predicts fluid responsiveness [44, 45]. However, 2021
the stroke volume variation has a few caveats, 5. Kirkpatrick JN, Grimm R, Johri AM, Kimura BJ,
which also apply to non-echocardiographic et  al. Recommendations for echocardiography labo-
methods of quantification (e.g., pulse waveform ratories participating in cardiac point of care cardiac
ultrasound (POCUS) and critical care echocardiog-
analysis). Among these include the need for raphy training: report from the American Society
mechanical ventilation, a fixed tidal volume of of Echocardiography. J Am Soc Echocardiogr.
8 mL.kg−1, and sinus rhythm [44–48]. In addi- 2020;33(4):409–422.e4.
tion to static measurements of LVOT VTI vari- 6. Stickles SP, Carpenter CR, Gekle R, Kraus CK, et al.
The diagnostic accuracy of a point-of-care ultrasound
ability, a passive leg raise can be performed, and protocol for shock etiology: a systematic review and
the pre-leg raise and post-leg raise VTI values meta-analysis. CJEM. 2019;21(3):406–17.
are compared. The leg raise is akin to subjecting 7. Seif D, Perera P, Mailhot T, Riley D, Mandavia
the patient to a volume load, by returning the D. Bedside ultrasound in resuscitation and the rapid
ultrasound in shock protocol. Crit Care Res Pract.
pooled venous blood in the lower limb to the 2012;2012:503254.
right ventricle. A change of >12% is suggestive 8. Milne J, Atkinson P, Lewis D, et  al. Sonography in
of fluid responsiveness [49, 50]. hypotension and cardiac arrest (SHoC): rates of
abnormal findings in undifferentiated hypotension
and during cardiac arrest as a basis for consensus on a
5.1.1.9 Cardiac Output hierarchical point of care ultrasound protocol. Cureus.
Cardiac output can be calculated noninvasively 2016;8(4):e564.
by echocardiography and has been shown to cor- 9. Vaishnav M, Sedgwick J. Point-of-care echocardiog-
relate well with that obtained by thermodilution raphy – a road to future or a step backwards. Australas
J Ultrasound Med. 2019;22:26–31.
techniques [51]. SV can be obtained by the for- 10. McKaigney CJ, Krantz MJ, La Rocque CL, Hurst ND,
mula SV = LVOTarea × VTILVOT, described in the Buchanan MS, Kendall JL. E-point septal separation:
previous section. The apical 5 chamber and 3 a bedside tool for emergency physician assessment of
chamber views can be used to measure LVOT left ventricular ejection fraction. Am J Emerg Med.
2014;32(6):493–7.
VTI using pulsed wave doppler. Care has to be 11. Silverstein JR, Laffely NH, Rifkin RD.  Quantitative
taken not to use continuous wave doppler as this estimation of left ventricular ejection fraction from
modality measures the highest velocity along the mitral valve E-point to septal separation and compari-
118 R. V. Agrawal et al.

son to magnetic resonance imaging. Am J Cardiol. 24. Nishimura RA, Borlaug BA. Diastology for the clini-
2006;97(1):137–40. cian. J Cardiol. 2019;73(6):445–52.
12. Wandt B, Bojö L, Tolagen K, Wranne 25. Nagueh SF, Smiseth OA, Appleton CP, Byrd BF
B.  Echocardiographic assessment of ejection 3rd, Dokainish H, Edvardsen T, Flachskampf FA,
fraction in left ventricular hypertrophy. Heart. Gillebert TC, Klein AL, Lancellotti P, Marino P, Oh
1999;82(2):192–8. JK, Popescu BA, Waggoner AD.  Recommendations
13. Picard MH, Popp RL, Weyman AE.  Assessment for the evaluation of left ventricular diastolic function
of left ventricular function by echocardiography: by echocardiography: an update from the American
a technique in evolution. J Am Soc Echocardiogr. Society of Echocardiography and the European
2008;21(1):14–21. Association of Cardiovascular Imaging. J Am Soc
14. St John Sutton MG, Plappert T, Rahmouni Echocardiogr. 2016;29(4):277–314.
H. Assessment of left ventricular systolic function by 26. Silbiger JJ.  Pathophysiology and echocardiographic
echocardiography. Heart Fail Clin. 2009;5(2):177–90. diagnosis of left ventricular diastolic dysfunction. J
15. Roscoe A, Strang T.  Echocardiography in inten- Am Soc Echocardiogr. 2019;32(2):216–232.e2.
sive care. Contin Educ in Anaesth Crit Care Pain. 27. Schumacher A, Khojeini E, Larson D.  ECHO
2008;8(2):46–9. parameters of diastolic dysfunction. Perfusion.
16. Mayo PH, Narasimhan M, Koenig S.  Critical 2008;23(5):291–6.
care transesophageal echocardiography. Chest. 28. Anavekar NS, Skali H, Bourgoun M, Ghali JK, Kober
2015;148(5):1323–32. L, Maggioni AP, McMurray JJ, Velazquez E, Califf R,
17. Seward JB, Khandheria BK, Freeman WK, Oh JK, Pfeffer MA, Solomon SD.  Usefulness of right ven-
Enriquez-Sarano M, Miller FA, Edwards WD, Tajik tricular fractional area change to predict death, heart
AJ.  Multiplane transesophageal echocardiography: failure, and stroke following myocardial infarction
image orientation, examination technique, anatomic (from the VALIANT ECHO study). Am J Cardiol.
correlations, and clinical applications. Mayo Clin 2008;101(5):607–12.
Proc. 1993;68(6):523–51. 29. Rudski LG, Lai WW, Afilalo J, Hua L,
18. Reeves ST, Finley AC, Skubas NJ, Swaminathan Handschumacher MD, Chandrasekaran K, Solomon
M, Whitley WS, Glas KE, Hahn RT, Shanewise JS, SD, Louie EK, Schiller NB.  Guidelines for the
Adams MS, Shernan SK.  Council on Perioperative echocardiographic assessment of the right heart
Echocardiography of the American Society of in adults: a report from the American Society
Echocardiography; Society of Cardiovascular of Echocardiography endorsed by the European
Anesthesiologists. Basic perioperative transesopha- Association of Echocardiography, a registered
geal echocardiography examination: a consensus state- branch of the European Society of Cardiology, and
ment of the American Society of Echocardiography the Canadian Society of Echocardiography. J Am Soc
and the Society of Cardiovascular Anesthesiologists. Echocardiogr. 2010;23(7):685–713. quiz 786-8
J Am Soc Echocardiogr. 2013;26(5):443–56. 30. Dutta T, Aronow WS.  Echocardiographic evalua-
19. Hope MD, de la Pena E, Yang PC, Liang DH, tion of the right ventricle: clinical implications. Clin
McConnell MV, Rosenthal DN.  A visual approach Cardiol. 2017;40(8):542–8.
for the accurate determination of echocardiographic 31. Armstrong DW, Tsimiklis G, Matangi MF.  Factors
left ventricular ejection fraction by medical students. influencing the echocardiographic estimate of right
J Am Soc Echocardiogr. 2003;16(8):824–31. ventricular systolic pressure in normal patients and
20. Foley TA, Mankad SV, Anavekar NS, Bonnichsen clinically relevant ranges according to age. Can J
CR, Morris MF, Miller TD, et al. Measuring left ven- Cardiol. 2010;26(2):e35–9.
tricular ejection fraction-techniques and potential pit- 32. Frémont B, Pacouret G, Jacobi D, Puglisi R,
falls. European Cardiology. 2012;8(2):108–14. Charbonnier B, de Labriolle A.  Prognostic value of
21. Li J, Zhang L, Wang Y, Zuo H, Huang R, Yang X, Han echocardiographic right/left ventricular end-diastolic
Y, He Y, Song X. Agreement in left ventricular func- diameter ratio in patients with acute pulmonary embo-
tion measured by echocardiography and cardiac mag- lism: results from a monocenter registry of 1,416
netic resonance in patients with chronic coronary Total patients. Chest. 2008;133(2):358–62.
occlusion. Front Cardiovasc Med. 2021;8:675087. 33. Hendriks SV, Klok FA, den Exter PL, Eijsvogel M,
22. Iwaszczuk P, Kołodziejczyk B, Kruczek T, et  al. Faber LM, Hofstee HMA, Iglesias Del Sol A, Kroft
Ischemic versus non-ischemic (neurogenic) myo- LJM, Mairuhu ATA, Huisman MV.  Right ventricle-­
cardial contractility impairment in acute coro- to-­left ventricle diameter ratio measurement seems
nary syndromes: prevalence and impact on left to have no role in low-risk patients with pulmonary
ventricular systolic function recovery. Med Sci Monit. embolism treated at home triaged by Hestia criteria.
2018;24:3693–701. Am J Respir Crit Care Med. 2020;202(1):138–41.
23. Ortiz-Pérez JT, Rodríguez J, Meyers SN, et  al. 34. Oh SB, Bang SJ, Kim MJ. McConnell’s sign; a dis-
Correspondence between the 17-segment model and tinctive echocardiographic finding for diagnosing
coronary arterial anatomy using contrast-enhanced acute pulmonary embolism in emergency department.
cardiac magnetic resonance imaging. Cardiovasc Crit Ultrasound J. 2015;7(Suppl 1):A20. https://doi.
Imaging. 2008;1:282–93. org/10.1186/2036-­7902-­7-­S1-­A20.
5  Point-of-Care Ultrasound of the Heart: Transthoracic Echocardiogram 119

35. Yildizdas D, Aslan N.  Ultrasonographic inferior caval collapsibility as a gauge of volume status in
vena cava collapsibility and distensibility indices for ventilated septic patients. Intensive Care Med. 2004
detecting the volume status of critically ill pediatric Sep;30(9):1734–9.
patients. J Ultrason. 2020;20(82):e205–9. 44. Miller A, Mandeville J.  Predicting and measuring
36. Blehar DJ, Resop D, Chin B, Dayno M, Gaspari fluid responsiveness with echocardiography. Echo
R.  Inferior vena cava displacement during respi- Res Pract. 2016;3(2):G1–G12.
rophasic ultrasound imaging. Crit Ultrasound J. 45. Charron C, Fessenmeyer C, Cosson C, Mazoit JX,
2012;4(1):18. Hebert JL, Benhamou D, Edouard AR.  The influ-
37. Kircher BJ, Himelman RB, Schiller NB. Noninvasive ence of tidal volume on the dynamic variables of fluid
estimation of right atrial pressure from the inspira- responsiveness in critically ill patients. Anesth Analg.
tory collapse of the inferior vena cava. Am J Cardiol. 2006;102:1511–1517.Vie.
1990;66(4):493–6. 46. Feissel M, Michard F, Mangin I, Ruyer O, Faller JP,
38. Barbier C, Loubières Y, Schmit C, Hayon J, Ricôme Teboul JL. Respiratory changes in aortic blood veloc-
JL, Jardin F, Vieillard-Baron A. Respiratory changes ity as an indicator of fluid responsiveness in ventilated
in inferior vena cava diameter are helpful in predict- patients with septic shock. Chest. 2001;119:867–73.
ing fluid responsiveness in ventilated septic patients. 47. Heenen S, De Backer D, Vincent JL.  How can the
Intensive Care Med. 2004;30(9):1740–6. response to volume expansion in patients with sponta-
39. Mookadam F, Warsame TA, Yang HS, Emani UR, neous respiratory movements be predicted? Crit Care.
Appleton CP, Raslan SF. Effect of positional changes 2006;10:R102.
on inferior vena cava size. Eur J Echocardiogr. 48. De Backer D, Heenen S, Piagnerelli M, Koch M,
2011;12(4):322–5. Vincent JL. Pulse pressure variations to predict fluid
40. Goldhammer E, Mesnick N, Abinader EG, Sagiv responsiveness: influence of tidal volume. Intensive
M. Dilated inferior vena cava: a common echocardio- Care Med. 2005;31:517–23.
graphic finding in highly trained elite athletes. J Am 49. Bou Chebl R, Wuhantu J, Kiblawi S, Carnell
Soc Echocardiogr. 1999;12(11):988–93. J. Bedside echocardiography and passive leg raise as
41. Gadi SR, Ruth BK, Johnson A, Mazimba S, Kwon a measure of volume responsiveness in the emergency
Y.  Isolated marked inferior vena cava dilatation: department. J Ultrasound Med. 2019;38(5):1319–26.
unusual presentation or underrecognized common 50. Maizel J, Airapetian N, Lorne E, Tribouilloy C,
phenomenon? Case Rep Cardiol. 2018;2018:8396523. Massy Z, Slama M.  Diagnosis of central hypovole-
42. Pellicori P, Carubelli V, Zhang J, Castiello T, Sherwi mia by using passive leg raising. Intensive Care Med.
N, Clark AL, Cleland JG.  IVC diameter in patients 2007;33(7):1133–8.
with chronic heart failure: relationships and prog- 51. Zhang Y, Wang Y, Shi J, Hua Z, Xu J. Cardiac output
nostic significance. JACC Cardiovasc Imaging. measurements via echocardiography versus thermodi-
2013;6(1):16–28. lution: a systematic review and meta-analysis. PLoS
43. Vieillard-Baron A, Chergui K, Rabiller A, Peyrouset One. 2019;14(10):e0222105. https://doi.org/10.1371/
O, Page B, Beauchet A, Jardin F.  Superior vena journal.pone.0222105.
Vascular Ultrasound
6
Sudhakar Subramani, Satoshi Hanada,
and Arunangshu Chakraborty

6.1 Introduction diagnose venous thrombosis, while follow-up


controls help in the prediction of recurrent venous
Vascular ultrasound (VU) plays a significant role thrombosis. Ultrasound is a key tool to determine
in visual depiction of arteries and veins. The pro- the cause and severity of chronic venous insuffi-
cedure has, over the last two decades, undergone ciency and allows various therapeutic procedures
considerable upgrades in technology, approach, for the treatment of varicose veins to be visually
and utility to diagnose various vascular-related monitored. Like arterial cannulation, VU helps to
disorders in both major and minor vessels and cannulate major veins and the deeper veins in
determine management options for those condi- certain complex clinical settings. In this chapter,
tions. In the case of arteries, VU gives informa- the authors give a broader view of the role of
tion ranging from endothelial dysfunction over ultrasound in certain vascular structures.
measuring the increase of intima media thickness The supplementary videos provided along
to the detection of stenoses, occlusion, or aneu- with this chapter will help the beginners practice
rysm. VU helps to differentiate between primary their skills at—Video 6.1: Assessment of IJ and
vasculitis and arterial compression syndromes Carotid artery, Video 6.2: Assessment of
like entrapment syndrome of different arterial Subclavian vessels infraclavicular approach,
regions. It also plays an essential role in arterial Video 6.3: Assessment of IVC Trans abdominal
cannulation of various vessels such as femoral, coronal long axis, Video 6.4: Assessment of
brachial, radial, and dorsalis pedis in a wide abdominal aorta, Video 6.5: Assessment of femo-
range of clinical settings. In the case of veins, VU ral vessels.
is considered as the primary imaging modality to

Supplementary Information The online version con- 6.2 Carotid Artery


tains supplementary material available at [https://doi.
org/10.1007/978-­981-­16-­7687-­1_6].
6.2.1 Anatomy of Carotid Artery
S. Subramani (*) · S. Hanada
Department of Anesthesia, University of Iowa, The carotid arteries are the primary vessels that
Iowa City, IA, USA supply blood to the brain and the facial area. The
e-mail: [email protected] right common carotid artery (RCCA) originates
A. Chakraborty in the neck from the brachiocephalic artery while
Department of Anaesthesia, Critical Care and Pain, the left common carotid artery (LCCA) arises in
Tata Medical Center, Newtown, Kolkata,
the thoracic region from the arch of the aorta [1]
West Bengal, India

© The Author(s), under exclusive license to Springer Nature Singapore Pte Ltd. 2022 121
A. Chakraborty, B. Ashokka (eds.), A Practical Guide to Point of Care Ultrasound (POCUS),
https://doi.org/10.1007/978-981-16-7687-1_6
122 S. Subramani et al.

R Common carotid A

R Subclavian A

R Brachiocephalic A L Subclavian A

Arch of
aorta

aort ing

Descending aorta
end
a
Asc

Fig. 6.1  Origin of left and right carotid arteries

(Fig.  6.1). The common carotid artery typically


bifurcates at the upper border of the thyroid carti-
lage at the level of carotid sinus into the internal
carotid artery (ICA) predominantly supplies the
brain, and the external carotid artery (ECA) sup-
plies the neck and face (Fig. 6.2). However, sev-
eral studies have shown a variant in which the
common carotid artery bifurcates more distally
[2]. Generally, carotid bifurcation is the site of
clinically meaningful atherosclerosis, and a more
distal bifurcation may impact the ability to pro-
ceed with standard surgical or interventional
approaches. Evaluating carotid artery anatomy is
essential prior to cannulation or any interven-
tional procedures.

6.2.2 Ultrasound Assessment Fig. 6.2  Course of common carotid artery and IJV
of Carotid Artery
and Doppler US can be used for the evaluation of
Ultrasound of carotid artery (CA) remains a long-­ extracranial segments of the CA, with the com-
standing and reliable tool in the current arma- plexity of these two methods commonly being
mentarium of diagnostic modalities used to referred to as duplex ultrasound (DUS). The 2D
assess vascular morbidity, especially in an early US helps predominantly for the assessment of
stage. Both grayscale (two-dimensional—2D) morphology including size, variations in carotid
6  Vascular Ultrasound 123

bifurcation, anomaly, and of vascular wall ath- peri-operative and critical care aspects, knowing
erosclerotic changes (Figs.  6.3a, b and 6.4 nor- variations in CA is essential to minimize inadver-
mal carotid artery sax and lax). From the tent arterial cannulation while placing a central
venous cannula in the internal jugular vein
(Fig.  6.5 variations in carotid artery). Although
a Anterior the 2D US could differentiate between calcified
and noncalcified portions of the plaque, the
SCM advancement of technology using
­three-­dimensional (3D) methods makes such dif-
Medial Lateral
ferentiation much easier and more precise. The
IJV determination of noncalcified and hypoechoic
CA
portions of the plaques carries an independent
risk factor for stroke. 2D along with 3D ultra-
sound can determine the intima media thickness
Anterior (IMT) aiding in the detection of preclinical ath-
b
erosclerotic disease, and subsequent assessment
of risk stratification based on the grade of ather-
oma [3] (Fig. 6.6 atheroma).
IJV

6.2.3 Duplex Ultrasound of Carotid


Carotid Artery
artery
In addition to the morphological assessment of
carotid arteries, US permits measurement of flow
Fig. 6.3 (a) US image of carotid artery in short axis. (b) based on the reflection of ultrasound waves by
Color doppler of carotid artery with compression on IJV utilizing both pulse Doppler US and color flow

Fig. 6.4  US image of Anterior


carotid artery in long
axis showing bifurcation a

ICA
BULB CCA

ICA
Cranial Caudal
5mm

CCA
BULB

ICA

5mm

CCA-Common Carotid A, ECA-External Carotid


A, ICA-Internal Carotid A
124 S. Subramani et al.

a b

Fig. 6.5  Variants in the course of carotid artery and internal jugular vein

a b

Fig. 6.6  US image of carotid artery in short and long axis with atheromatous plaque

Carotid artery superimposed on 2D images can be done with


color Doppler US to determine grading of ste-
notic lesions (Fig.  6.8a, b spectral Doppler). In
Cranial
Caudal general, by using color flow Doppler, high-grade
stenotic lesions are more easily detected com-
pared with pulse wave Doppler although the lat-
ter allows direct estimation of flow velocity.
Although the peak systolic velocity (PSV) is
commonly used to quantify stenosis, end-­
Fig. 6.7  Color doppler of carotid artery in the longitudi- diastolic velocity (EDV), carotid index, and spec-
nal plane tral waveform analysis are also used for the
assessment of stenosis [4]. There are a number of
Doppler US.  Visualization of color-coded flow pitfalls when using velocity-based estimation of
provides information about the direction of blood CA stenosis such as elevated velocities when
flow (Fig.  6.7). The measurement of velocity there is contralateral carotid artery occlusion and
6  Vascular Ultrasound 125

a
b

Fig. 6.8 (a) Spectral doppler of carotid artery (normal) in the longitudinal plane. (b) Spectral doppler of carotid artery
with low systolic and diastolic velocity

higher velocities in females. Moreover, DUS is the vagus nerve posteriorly (Fig. 6.9). It generally
restricted to the cervical portion of the ICA with lies just lateral and anterior to the internal and
the option of obtaining added information from common carotid arteries; in rare situation it lies
transcranial Doppler studies recommended in entirely medial to the carotid artery. The IJV
certain challenging situations. Factors such as combines with the subclavian vein to form the
severe tortuosity of an artery, high carotid bifur- brachiocephalic or innominate vein at the junc-
cation, obesity, or extensive calcification of the tion of the neck and thorax. The left internal jug-
vasculature might reduce the accuracy of estima- ular vein is slightly smaller than the right internal
tion [5]. In spite of these limitations, Duplex jugular vein. Both veins contain valves located a
ultrasound has many advantages of safety, afford- few centimeters before the vessels drain into the
ability, and the ability to directly visualize mor- subclavian vein. Due to a relatively straighter
phology and measure flow compared to other course, right IJV is preferred over left IJV for
imaging modalities. It is also relatively inexpen- central venous cannulation. Familiarity with sur-
sive, readily performed at the bedside with no face anatomy of IJV is essential for both diagnos-
radiation or need for contrast. tic assessment of various disorders and for
cannulation.

6.3 Internal Jugular Vein


6.3.2 Ultrasound Assessment of IJV
6.3.1 Anatomy of Internal
Jugular Vein Both 2D and color Doppler ultrasound are com-
monly utilized to evaluate the morphology of IJV
The internal jugular vein (IJV) is a paired venous and its course (Fig. 6.10). Compression US is rou-
structure that receives blood from the brain, face, tinely used to differentiate arterial system from
and neck, and drains it to the right atrium. The venous structures, especially in significant venous
internal jugular vein is the continuation of the pulsation from adjacent arteries as well as from
sigmoid sinus. It arises in the posterior cranial high central venous pressure (Fig. 6.11). US of IJV
fossa and exits the cranium through the jugular helps to identify variants in anatomical course,
foramen. In the upper part of the neck just under- presence of valves in unusual location, thrombo-
neath the base of the skull, it drains the branches sis, chronic venous insufficiency, and even com-
of the facial, retromandibular, and lingual veins. plete absence of IJV on one or both sides (Video
The course of the internal jugular vein is directed 6.1) (Fig. 6.12). The role of US on IJV has been
caudally in the carotid sheath. It is accompanied expanded to measure the changes in the IJV cross-
by the common carotid artery anteromedially and sectional area CSA during the cardiac cycle simi-
126 S. Subramani et al.

Fig. 6.9  Relationship between IJV and carotid artery. (a) IJV anterior to CA, (b) IJV anterolateral to CA and (c) IJV
lateral to CA

Fig. 6.10  Color flow Anterior


doppler of IJV

IJV

Lateral

Carotid A

lar to caval index in inferior vena cava, and can abnormal extracranial venous flow in a variety of
estimate the jugular venous pulse and right atrial central nervous system disorders, such as multiple
pressure [6]. Despite potential inaccuracies, the sclerosis, migraine, Parkinson’s disorders, Meniere
DS observation of the IJV pulsatility remains an syndrome, and obstructive sleep apnea [9–12]. An
acceptable method of central venous pressure accurate and reproducible DS venous blood flow
monitoring in various clinical settings [7]. It also volume (BFV) method is essential to diagnose
helps to predict fluid responsiveness in mechani- those conditions.
cally ventilated patients after cardiac surgeries
similar to IVC variability [8]. In addition to regular 6.3.2.1 Anatomy of Subclavian Vessels
assessment of IJV for cannulation and fluid status, The subclavian vein is the continuation of the
Spectral DS analysis of the IJVs is an emerging axillary vein as it courses beneath the clavicle. At
new concept in examining their physiological the lateral border of the first rib, it travels superi-
hemodynamic ranges. Many studies have shown orly then under the clavicle travels medially until
6  Vascular Ultrasound 127

Fig. 6.11 Compression Anterior


USG of IJV

IJV
Medial CA

where the apex of the lung can extend just above


the first rib. Familiarity with anatomical variants
of subclavian vessels are essential prior to any
interventional procedures or cannulation of both
vein and artery.

6.3.2.2 Ultrasound Assessment


of Subclavian Vessels
Similar to IJV, 2D and color Doppler US are uti-
lized to assess the morphology, patency, anatomi-
cal variants, and certain pathological conditions
such as aneurysm, stenosis, or thrombosis of the
vessels (Video 6.2) (Fig.  6.13). Compared to
US-guided cannulation of IJV there is limited
data on the utilization of US for SV cannulation.
Meta-analysis of over 2000 patients from nine
Fig. 6.12  Thrombosis of IJV
studies showed that the use of US resulted in a
reduced rate of accidental arterial puncture (0.8%
it joins the internal jugular vein to form an vs 5.9%); and hematoma formation (1.2% vs
innominate vein. The subclavian artery runs pos- 6.6%). However, there was no statistically sig-
terior and superior to the subclavian vein. The nificant difference between the use of US and the
brachial plexus courses superiorly and deep to conventional landmark technique in the total
the medial portion of the subclavian artery. On complication rate, the overall success rate, the
the left side, the thoracic duct lies close to the number of attempts until success, the time to suc-
subclavian vessels. Most importantly, the lung cessful cannulation, and the success rate with the
and pleural cavity lie inferior and deeper to the first attempt [13, 14]. Currently, there is no strong
subclavian vein and are particularly vulnerable to evidence of using the US for SV cannulation
accidental puncture more often on the left chest from different societies.
128 S. Subramani et al.

a b

Fig. 6.13  US image of subclavian vessels (infraclavicular approach—short axis and long axis)

6.4 Anatomy of Inferior Table 6.1 Correlation between size of IVC and


Vena Cava estimated central venous pressure (CVP)
Maximum diameter of IVC(cm) CI (%) CVP (mmHg)
The inferior vena cava is formed by the confluence <1.5 100 0–5
of the two common iliac veins at the L5 vertebral 1–5–2.5 >50 6–10
level. The IVC has a retroperitoneal course within 1.5–2.5 <50 11–15
the abdominal cavity. It runs along the right side of >2.5 <50 16–20
the vertebral column with the aorta lying laterally >2.5 0 >20
on the left. Various other veins drain into the IVC
along its course before it passes through the dia- surrounding structures. Its size and dynamics
phragm at the caval hiatus at the T8 level. Hepatic vary with respiration and changes in intravascu-
veins are the last tributaries of IVC before it enters lar volume. The size of the IVC and its ­respiratory
into the thoracic cavity and drains into the right variability has been shown to correlate with right
atrium at the inferior cavo-atrial junction. atrial pressure (RAP) and intravascular volume.
(Table  6.1) These observations are valuable in
estimating RAP, detecting changes in intravascu-
6.4.1 Ultrasound Assessment of IVC lar volume, and monitoring a patient’s response
to volume resuscitation [15]. The development of
Ultrasound evaluation of the IVC provides a negative intrathoracic pressure during inspiration
rapid, noninvasive assessment of a patient’s increases the venous blood return from the extra-
hemodynamic status at the bedside. Structurally, thoracic veins into the right heart. This leads to
the IVC is a thin-walled, highly compliant vessel an increase in the blood flow through the IVC and
and its capacity to distend is not unlimited and is a subsequent decrease in its blood volume, result-
restricted by connective tissue in its walls and ing in a reduction in intraluminal pressure. These
6  Vascular Ultrasound 129

changes decrease the diameter of the IVC during 6.4.2 Assessment of IVC
inspiration relative to expiration. These observa- with Ultrasound
tions are reversed with positive pressure ventila-
tion in which IVC diameter increases during Various acoustic windows to visualize the IVC
inspiration. In extremely hypovolemic status, and it depends on the clinician’s ability to acquire
IVC nearly collapses completely with inspiration those views. Most commonly subcostal-­
during spontaneous breathing. Generally, respi- subxiphoid site is used to view the IVC, espe-
ratory phasic changes are easily appreciable in cially from intra-hepatic part to IVC—right atrial
relatively hypovolemic status compared to high junction. This view is the part of point-of-care
intravascular volume status. cardiac ultrasound (POCUS) described in detail
Traditionally, central venous pressure (CVP) in trans thoracic echocardiography chapter. Other
and volume status in the acute care setting have views to visualize IVC are from transabdominal
been measured by placing a central line. Central short axis and right lateral transabdominal coro-
lines are invasive, time-consuming to insert, and nal long axis commonly referred to as “rescue
may cause significant complications. US of IVC view.” Short-axis view was obtained by placing
is a relatively inexpensive bedside tool. Clinicians US in the mid-epigastric region with the orienta-
can perform serial IVC measurements on a criti- tion marker towards the patient’s right. By gently
cally ill patient to guide their decision in provid- fanning the probe up and down, one can visualize
ing more intravenous fluids or to administer more IVC and the aorta anterior to the central shadow-
aggressive medication therapy [16]. ing from the vertebral body (Fig.  6.14a, b). By

Cranial

Anterior
b

Aorta

Lateral

IVC

Fig. 6.14 (a) Trans abdominal SAX approach for IVC ( probe position pointer towards right). (b) Trans abdominal
SAX approach for IVC
130 S. Subramani et al.

fanning further cranially towards the chest, by using color flow or spectral Doppler. Pulsatility
hepatic veins joining the IVC come into view. is not a great way to differentiate between IVC
Longitudinal view of IVC is typically obtained and aorta because IVC frequently demonstrates
from subcostal view however in certain clinical transmitted pulsations. Moreover, in moderate to
situations due to surgical dressings, distended severe tricuspid regurgitation, IVC can be very
abdomen, etc., longitudinal view of the IVC is pulsatile mimicking the aorta. Irrespective of the
acquired through the right lateral transabdominal approach, a phased-array (frequency of 2.0–
coronal approach, also known as the “rescue 4.0 MHz) or curvilinear probe (frequency of 3.5–
view.” The probe is placed in the right anterior to 5.0 MHz) should be used to visualize IVC. These
midaxillary line similar to the placement for eval- relatively low-frequency probes provide better
uating pleural effusion with the orientation penetration and visualization of deep structures.
marker towards cranially (Video 6.3). By scan- Literature favors B-mode, subxiphoid Lax view
ning more anteriorly, the IVC can be visualized as the most reliable means of IVC acquisition.
running adjacent to the liver and crossing the dia- IVC measurement is less reliable in M-mode
phragm and joining the right atrium (Fig. 6.15a, when compared to B-mode. This discrepancy is
b). Aorta is often seen parallel to it and it is essen- augmented when calculating IVC collapsibility
tial to differentiate between these two structures index to determine fluid responsiveness [17, 18].

a Cranial

Pointer
towards head

Anterior
b

Cranial
IVC

Aorta

Fig. 6.15 (a) Right lateral trans-abdominal coronal approach (rescue view) probe position pointer towards head. (b)
Right lateral trans-abdominal coronal approach (rescue view) with color flow doppler
6  Vascular Ultrasound 131

6.4.3 Clinical Applications Table 6.2 Factors influencing fluid responsiveness


indices
and Limitations of IVC
as a Tool for Predicting Fluid Pathophysiological
factor affecting fluid
Responsiveness responsiveness Cause for inaccuracy
Mechanical Larger IVC size due to
An IVC collapsibility index (IVCCI), which cor- ventilation with systemic venous congestion and
relates with RAP and CVP, can be calculated with high PEEP and/or low respiratory variations
the following formula: (IVCmax − IVCmin)/ low tidal volumes
Assisted ventilation Spontaneous breathing makes
IVCmax. It is the common measurements use to
modalities, IVC variation unpredictable
estimate CVP as well as predict fluid responsive- noninvasive
ness in a critically ill patient. Systemic review from ventilation, CPAP
21 studies including over 1400 patients showed a Spontaneous Deep inspiratory effort, producing
medium to strong correlation between IVC diame- breathing with markedly negative intrathoracic
varying respiratory pressures that induce collapsibility
ter and IVCCI and estimated CVP in spontane- pattern even in the absence of FR. On the
ously breathing patients however weak correlation other hand shallow breathing
in mechanically ventilated patients, especially with creates minimal intrathoracic
PEEP [19]. Another meta-­analysis for caval index pressure changes, may induce
absence of collapsibility even in
with 20 studies showed a wide range of heteroge- presence of FR
neity with sensitivity and specificity of 0.71 (95% Acute exacerbation Lung hyperinflation and auto
CI: 0.62–0.80) and 0.75 (95% CI: 0.64–0.85), of COPD or asthma PEEP decrease venous return
respectively. Inconsistency results are not in favor falsely induces distension of IVC
of reliable methods in predicting fluid responsive- RV dysfunction RV dilatation and systemic
venous congestion causing large
ness [20]. From a single center study in mechani- IVC may be associated with FR
cally ventilated patients, Yao et al. proposed IVC Moderate to severe Chronic enlargement of IVC
ADI (area distensibility index) calculated as: (max- tricuspid and reduced collapsibility may
imum IVC area − minimum IVC area)/minimum regurgitation erroneously rule out FR
IVC area × 100% and IVC DR(diameter ratio) cal- Cardiac tamponade Hindrance of venous return
influences FR prediction
culated as: long axis IVC parameter/short axis IVC
Increased intra-­ IVC collapses or IVC gets
parameter have more value than IVC diameter dis- abdominal pressure distended depending on the
tensibility index for predicting fluid responsiveness types and mode of ventilation
in mechanically ventilated patients [21]. In spite of Local factors Mass, thrombosis, IVC filters
various available tools in IVC, one should evaluate or ECMO cannula might
influence size and collapsibility
ten common pathophysiologic conditions
Anatomical factors Significant displacement of IVC
(Table 6.2) that influence measurements [22]. might cause an error in diameter
as well as other indices

6.5 Abdominal Aorta


artery arises at the level of T12 and supplies the
6.5.1 Anatomy of Abdominal Aorta liver, stomach, abdominal esophagus, spleen, the
superior duodenum, and the superior pancreas.
The abdominal aorta (AA) is a continuation of Superior mesenteric artery arises at T1 just below
the thoracic aorta beginning at the level of the the celiac artery and supplies the distal duode-
T12 vertebrae. It is approximately 13  cm long num, jejuno-ileum, ascending colon, and part of
and bifurcates into the left and right common the transverse colon. Renal arteries are the paired
iliac arteries at the level of the L4 vertebra. There visceral arteries that arise laterally at the level
are many branches that arise from AA (Fig. 6.16) between L1 and L2 and supply the kidneys.
and for US assessment knowing the origin and Inferior mesenteric artery arises at the level of L3
course of celiac, superior mesenteric, renal, and and supply the large intestine from the splenic
inferior mesenteric arteries is essential. Celiac flexure to the upper part of the rectum.
132 S. Subramani et al.

a
Diaphragm
Coeliac trunk

Left renal
Superior artery
mesenteric
Infrarenal aorta
artery

Inferior
mesenteric
Right common artery Anterior
iliac artery b
Right external Left internal
iliac artery Abdominal
iliac artery
Aorta

Fig. 6.16  Course and branches of abdominal aorta


IVC

6.5.2 Ultrasound Assessment Vertebral


of Abdominal Aorta Shadow

Ultrasonography of the AA is primarily performed Posterior


to detect or exclude an abdominal aortic aneurysm
Fig. 6.17 (a) US probe position at the sub xiphoid
(AAA). By periodic screening, the occurrence of region. (b) US at the sub xiphoid region showing abdomi-
aneurysm rupture can generally be reduced in oth- nal aorta and IVC
erwise symptom-free patients. In addition, US of
AA is helpful to detect other aortic diseases/disor- patients to differentiate from adjacent great ves-
ders such as stenosis, embolism caused by throm- sels such as IVC.  Low frequency (2–5  MHz)
bosis in peripheral occlusion, dissection, and with higher penetration is preferred over high-
aortitis [23]. Compared with gold standard CT frequency probes. Place the US transducer in the
measurement errors and measured value discrep- subxiphoid region, apply gentle pressure and
ancies occur due to failure to use standardized adjust the depth to visualize the blood vessels
measurement methods. Multiple sources of error and vertebra. Displacing intra-abdominal air is
are due to differences in choosing between outer- essential to improve the image quality. Both IVC
to-outer (OTO) or inner-to-­inner (ITI) diameter, and aorta are viewed in short axis in front of the
systole or diastole, axial or orthogonal measure- vertebra (Video 6.4) (Fig. 6.17a, b). By moving
ment plane [24]. ECG gated measurements are the transducer gradually along the AA towards
generally advisable to minimize errors. caudally major branches such as celiac trunk,
superior mesenteric, renal arteries, and inferior
mesenteric will be visualized (Fig.  6.18).
6.5.3 Ultrasound Technique Appropriate angle of incidence is essential to
of Abdominal Aorta accurately measure the size of AA [25]. Being a
relatively less expensive bedside tool, assessing
2D ultrasound is generally used to assess the AA with ultrasound is the quickest way prior to
patency and course of AA. Color flow Doppler is extensive evaluation with other imaging modali-
required in certain technically challenged ties such as CT or 3D print.
6  Vascular Ultrasound 133

Anterior
Cranial

Celiac A

Superior
mesenteric A

Aorta

Fig. 6.18  US abdominal aorta shows origin of celiac trunk and superior mesenteric artery with color flow doppler

6.6 Role of Ultrasound certain pathological conditions including throm-


in the Deep Vein Thrombosis bosis (Fig. 6.20). 2D and color Doppler US are
(DVT) used for the assessment although spectral Doppler
might be needed in certain challenging condi-
6.6.1 Anatomy of Upper Limb Veins tions to differentiate between veins and arteries.
It is advisable to do a complete evaluation of
The upper extremity venous system comprises veins on both arms. In addition to the assessment
superficial and deep veins based on their relations of IJ and subclavian veins as described in the pre-
to the deep fascia. The major superficial veins of vious section, the patient’s arm should be par-
the upper limb are the cephalic and basilic veins. tially abducted to assess the axillary vein.
They are located within the subcutaneous tissue Axillary vein is generally visualized from infra-
of the upper limb. The basilic vein originates clavicular portion to axilla, where confluence of
from the dorsal venous network of the hand and cephalic vein into the axillary vein can be
ascends to the medial aspect of the upper limb. At observed (Fig.  6.13). On further moving the
the border of the teres major, the vein moves deep probe distally on the medial aspect basilic vein
into the arm. Here, it combines with the brachial along with brachial veins adjacent to brachial
veins from the deep venous system to form the artery can be viewed. Compression ultrasonogra-
axillary vein. The cephalic vein also arises from phy is recommended to differentiate blood ves-
the dorsal venous network of the hand. It ascends sels as often noticed vessels are close to each
the anterolateral aspect of the upper limb, passing other in this specific part of the arm (Fig. 6.21).
anteriorly at the elbow. At the shoulder, the On the lateral aspect of the arm, cephalic vein
cephalic vein travels between the deltoid and which is generally superficial compared to basilic
pectoralis major muscles and enters the axilla vein can be visualized. Assessment of veins at the
region via the clavipectoral triangle. Within the cubital fossa is essential as often noticed throm-
axilla, the cephalic vein empties into the axillary bophlebitis in this area due to prior cannulation.
vein (Fig. 6.19). By moving the probe caudally from the cubital
fossa, ulnar and radial veins can be viewed. High-­
frequency probes are used for UL assessment
6.6.2 Ultrasound Assessment although low frequency might require in certain
of Upper Limb Veins for DVT obese patients with excess fat tissue in the arm
and forearm.
US of upper limb can be performed from proxi- Color Doppler sonography is a rapid, accu-
mal to distal or from distal to proximal primarily rate, and noninvasive technique in the evaluation
to assess the patency, anatomical variants, and of venous disease in the upper extremity and is
134 S. Subramani et al.

Fig. 6.19  Upper limb Subclavian


venous system
Axillary

Cephalic

Subscapular

Brachial
Basilic

Median cubital

Cephalic

Radial

Median
antebrachial Basilic

Ulnar

KEY Palmar venous arches


Deep veins

Superficial veins Digital

6.6.3 Anatomy of Lower Limb


Vessels

Lower extremity venous system comprises deep


veins and superficial veins. The deep veins are
located beneath the muscular fascia and drain the
surrounding muscles, whereas the superficial
veins are above the deep fascia and drain the
cutaneous tissues. Both deep and superficial
Fig. 6.20  Basilic vein thrombosis veins have one-way valves inside the vessels that
contribute to unidirectional flow from distal to
the modality of choice in screening for UEDVT proximal portion of the vein. In order from the
[26]. However, contrast venography, CT, or MRI groin to the calf, deep veins consist of the com-
may be necessary in select cases in which sono- mon femoral vein (CFV), the deep femoral vein,
graphic findings are nondiagnostic or equivocal the femoral vein (FV), the popliteal vein (PV),
or when clinical suspicion for UEDVT remains and three tributaries of the PV (the anterior tibial
high despite normal Doppler findings. vein, posterior tibial vein, and peroneal vein)
6  Vascular Ultrasound 135

Anterior

Lateral

CV
Brachial
veins BA

Fig. 6.21  US at the mid arm medial aspect showing brachial vessels and basilic vein. BA brachial artery, CV Cephalic
vein

CFV at the saphenofemoral junction. Venous


duplication of the lower extremities is a common
anatomical variant. The prevalence of venous
duplication varies depending on the location of
the veins. The most common vessels are the FV
and the PV, and a retrospective study on the use
of ultrasonography for the detection of deep
venous thrombosis (DVT) shows a 10.1% preva-
lence of duplication within the femoral-popliteal
venous system [27].
Femoral artery (FA) is the continuation of the
external iliac artery, terminal branch of abdomi-
nal aorta. The external iliac artery becomes the
femoral artery when it crosses the inguinal liga-
ment which is an important anatomical structure
to determine cannulation or interventional site
for femoral artery (Fig. 6.23). Profunda femoris
artery (PFA) is the first branch that arises from
Fig. 6.22  Venous system of lower limb FA and there are variants in the origin of
PFA.  Clinicians should differentiate between
(Fig.  6.22). These deep veins are generally PFA from FA before placing catheters.
accompanied by an artery, and the veins are gen-
erally larger in diameter than the accompanying
artery. The FV was previously named as the 6.6.4 Ultrasound Evaluation
superficial femoral vein. However, the term of Femoral Vessels
“superficial” has been removed to avoid confu-
sion as the vein is actually a part of a deep venous The ultrasonographic assessment should be per-
system. There are numerous anatomical variants formed with the patient in a supine position. The
in the superficial venous system. Unlike deep head elevated up to 30 degrees with reverse tren-
veins, superficial veins do not have an accompa- delenburg position is preferable because it can
nying artery. The principal vein of the superficial pool the blood in the veins of the lower extremi-
veins is the great saphenous vein (GSV) that is ties and help the visualization. External rotation
the longest vein in the human body and joins the of the examining leg with slight knee flexion is
136 S. Subramani et al.

Femoral Artery

Medial
Circumflex
Femoral Artery
Profunda
Femoris
Artery

Lateral Fig. 6.24  External rotation of the examining leg with


Circumflex slight knee flexion
Femoral
Artery
Anterior

Fig. 6.23  Course of Femoral artery

recommended as it could further enlarge the FA Medial


Lateral
veins and decrease the depth of the target struc- FV

tures [28]. This position also allows the smooth


transition when a sonographer moves the probe
from the thigh to the popliteal fossa without repo-
sitioning the patient (Fig.  6.24). A linear ultra-
sound probe with a high-resolution 5–12 MHz is
usually chosen to assess the vessels of the lower
extremities; however, a lower frequency curved
probe may be needed in an obese patient to ensure
adequate penetration (Video 6.5).
The CFV is the most proximal vein and to be
scanned in the beginning during the sonographic Fig. 6.25  US of femoral vessels with color doppler at the
assessment of the lower extremities. By placing inguinal crease
the probe along the inguinal ligament between
the anterior superior iliac spine and the pubic (85.0% vs 48.7%). Similar to subclavian cannu-
symphysis, the transverse view of the CFV is lation there is no strong evidence of routine use
obtained. The CFV is medial to the common fem- of US for FV cannulation; however, authors sug-
oral artery and is usually larger in size (Fig. 6.25). gested to utilize US for certain challenging popu-
For CVC placement in the FV, the use of US lation such as morbid obesity with deeper
compared with the landmark technique increased structures. From the inguinal crease by rotating
the overall success rate (US-89.0% vs landmark the probe 90 degrees, the longitudinal view of
78.9%); and the success rate with the first attempt CFV is visualized. While keeping the CFV cen-
6  Vascular Ultrasound 137

tered on the transverse view’s image, the probe is view of the PV is visualized. In this view, the PV
moved distally 1–2 cm to reach the saphenofem- is posterior to the accompanying artery (the PV is
oral junction where the GSV joins the CFV closer to the skin, and the accompanying artery is
(Fig. 6.26). By further moving the probe 1–2 cm located deep in the vein) (Fig. 6.28). By further
distally, then the bifurcation of the CFV into the moving the probe distally to the level of the calf,
deep femoral vein and FV is observed (Fig. 6.27). the trifurcation is observed where the PV gives
The deep femoral vein dives deeper into the thigh rise to three tributaries (Fig. 6.29).
after the bifurcation, whereas the FV travels
along with the femoral artery through the adduc-
tor canal in the distal thigh and emerges as the PV 6.6.5 Role of Ultrasound
at the popliteal fossa. By placing the probe along in the Evaluation for Deep
the posterior crease of the knee, the transverse Vein Thrombosis

Deep vein thrombosis is generally diagnosed by


Anterior
symptoms of swelling and pain of the affected leg

Anterior

GSV
lateral CFA
CFV
PV
lateral

PA

Fig. 6.26  Common femoral vein at the saphenofemoral


vein junction

Anterior
Fig. 6.28  US of the popliteal vein in transverse view. PA
popliteal artery, PV popliteal vein

Anterior

CFA
lateral

FV Lateral

V V
V
DFV
PA

Fig. 6.27  US of common femoral vein (CFV) at its


bifurcation into deep femoral vein (DFV) and femoral Fig. 6.29  US of poplital vein inthe proximal part of the
vein (FV) calf
138 S. Subramani et al.

and tenderness by physical examination. lower extremities, the deep venous system should
However, individuals with DVT can also be be scanned routinely from the level just above the
asymptomatic. Nevertheless, unrecognized DVT inguinal ligament to at least the level of the pop-
could cause a life-threatening condition, such as liteal fossa. There has been a controversy on the
pulmonary embolism; therefore, early detection role of the routine evaluation of the three tributar-
and treatment are of utmost importance. ies of the PV at the level of the calf because iso-
Ultrasonography is the most appropriate initial lated thrombus in the tributaries often
diagnostic tool for detecting DVT.  Although it spontaneously resolves, and the risk of the anti-
requires knowledge of the vascular anatomy and thrombotic therapy may exceed the risk of pul-
experience with scanning technique, ultrasonog- monary embolism [29]. In addition, evaluating
raphy has numerous advantages compared with the calf veins increases the complexity and time
the other imaging modalities, such as contrast required to perform ultrasonography. Of note,
venography, CT, and MRI.  These advantages although the GSV is not a deep vein, investigat-
include portability, low cost, noninvasiveness, ing thrombus in the proximal GSV adjacent to
and avoidance of radiation or nephrotoxic con- the saphenofemoral junction is necessary because
trast exposure. When DVT is suspected at the of the high risk of thrombus dislodgement into
the deep venous system and may warrant antico-
Anterior
agulation therapy [30].
During an ultrasonographic examination,
direct visualization of thrombus is a promising
way to detect DVT.  It can be clearly observed
when the thrombus is echogenic (Fig.  6.30).
However, the presence of a thrombus is often
FA ambiguous under direct visualization. For exam-
ple, anechoic or hypoechoic thrombus may not
Lateral
FV be apparent. In contrast, slowly flowing blood or
blood stasis inside a vein mimics the image of a
thrombus. Therefore, direct visualization is not
sufficient enough to detect or rule out
DVT. Applying color flow Doppler ultrasonogra-
phy aids to identify thrombus as color flow inside
the vein is reduced or lost when the vein is par-
tially or completely occluded with thrombus
Fig. 6.30  Direct visualization of the DVT hyperechoic
material is seen in the common femoral vein (Fig.  6.31). Furthermore, the compressibility of

Complete occlusion of FV Partial occlusion of FV

Lateral

Fig. 6.31  Complete and partial occlusion of femoral vein due to thrombosis
6  Vascular Ultrasound 139

the venous segment should be routinely tested on the contrary, venous occlusion is likely if there is
the target vein as it provides high sensitivity and no increase in the venous flow. Of note, venous
specificity to diagnose DVT.  Inability to com- compression must be carefully performed as
press a vein while applying pressure by the probe there is a potential risk of dislodging thrombus
is the hallmark finding of thrombus in the venous and causing a catastrophic complication.
segment (Fig. 6.32). On the other hand, the com-
plete collapse of a vein rules out thrombus in that
region (Fig.  6.33). This compression technique 6.6.6 Ultrasound Evaluation
provides only regional information; thus, it may of Chronic Venous
need to apply every 2–3  cm on a targeted vein. Insufficiency
Lastly, the patency of a vein can be examined by
squeezing the distal part of the vein, often by per- Clinical symptoms of venous insufficiency are
forming calf compression. Augmented flow dur- similar to those of DVT. The symptoms include
ing the maneuver is observed by color and pulse leg pain, swelling, and skin manifestations such
Doppler ultrasonography in the normal vein. On as discoloration, induration, and ulceration.

Anterior

Lateral

FA FA

FV with
thrombus insitu

Fig. 6.32  DVT in the common femoral vein noncompressible thrombosed vein in short axis

Anterior

Medial

A
V
Compressed FV

Fig. 6.33  US compression test to exclude deep vein thrombus (DVT)


140 S. Subramani et al.

Although the immediate concern is ruling out


acute DVT and its potentially life-threatening
condition, venous insufficiency should not be
overlooked, especially in patients with persistent
or recurrent symptoms. Normally, venous blood
flow from a superficial venous system of the
lower extremities is directed towards the deep
venous system and ultimately moves to the
IVC.  Reversal of the venous blood flow is pre-
vented by numerous valves in the veins and mus-
cle contraction of the leg. However, when venous
valves become incompetent, venous blood flow
can be reversed. Venous blood pooling and
venous dilation caused by the flow reversal will
further accelerate valve failure at the distal por-
tion of the leg. The disease process can be trig-
gered and worsened by existing venous
obstruction caused by DVT or congestive heart
failure. Unlike a deep venous system, a superfi-
cial venous system is not surrounded by muscles;
thus, superficial veins are more vulnerable to
elevated venous pressure.
Ultrasonographic assessment of venous insuf- Fig. 6.34  US evaluation of chronic venous insufficiency
in standing position
ficiency is preferably performed with the patient
in the standing position because it induces hemo-
dynamics where venous reflux is significant that one should always keep in mind to avoid
(Fig.  6.34). Detecting flow reversal with color misdiagnosis when evaluating patients with sus-
and pulse Doppler ultrasonography is the most pected DVT.  A false-negative result may delay
useful method for localizing valvular incompe- or deprive the patient of appropriate anticoagula-
tence and evaluating the severity of the disease tion therapy and increase the risk of serious com-
(Fig. 6.35). When the venous valve is incompe- plications such as pulmonary embolism, whereas
tent, prolonged flow reversal is observed during a false-positive result may put the patient at
elevated intra-abdominal pressure by the Valsalva unnecessary risk of anticoagulation therapy.
maneuver. This is a reliable technique for the Firstly, gain and the other imaging parameters
proximal veins; however, venous insufficiency at must be properly adjusted prior to the examina-
the distal legs cannot be detected by this method tion because improper settings lead to a false
when the valve is competent at the proximal interpretation. For example, improperly high
veins. Thus, venous flow augmentation by gain produces artifactual echo images that can
squeezing the leg is applied when evaluating the be mistaken for thrombus. Color artifacts gener-
venous reflux at the distal legs. Additionally, ated by improperly high color scales also lead to
venous dilation and valvular structure assessed a false interpretation as it can obscure the image
by ultrasonography aid the diagnosis. of thrombus. Although testing the venous com-
pressibility is a useful diagnostic method, inad-
equate venous compression can lead to a
6.6.7 Pitfalls and Artifacts of Using false-positive result for DVT.  Pressure must be
Ultrasound for DVT firmly applied to the targeted venous segment
until the adjacent artery is slightly compressed.
Despite the high diagnostic accuracy of ultraso- Anatomical variants can also lead to misdiagno-
nography, there are a few pitfalls and limitations sis. Venous duplication is one of the commonest
6  Vascular Ultrasound 141

Fig. 6.35  Reflux of


blood flow with distal
compression in venous
insufficiency

Anterior anterior

A
A
1
FV 2 lateral
cranial V2
V1
V1
FV 1 2

Fig. 6.36 Duplicated femoral vein—Color Doppler Fig. 6.37  DVT in the duplicated femoral vein noncom-
ultrasonography in longitudinal view pressible thrombosed vein (V1) in transverse view

causes of a false-negative result for DVT diagno- can also be mistaken as DVT, especially in a
sis as an occluded duplicated vein can be over- patient with excess weight or enlarged superfi-
looked (Figs.  6.36 and 6.37) [31]. The cial veins. One of the key things that differenti-
femoral-popliteal venous system is a common ate DVT from the other abnormalities is
site of venous duplication, and it should be sus- identifying the adjacent artery as deep veins are
pected when a vein is smaller than normal or almost always accompanied by an artery. In
when the location is unusual. Experience and addition, unlike many other abnormal tissues,
careful attention to the anatomical variant could veins are often circular in the transverse view
prevent the false-­negative result. One should be and tubular in the longitudinal view. Lastly, the
aware that various other abnormalities at the differentiation between acute and chronic DVT
lower extremities can be easily mistaken for by ultrasonography can be challenging, although
DVT because clinical symptoms are similar and the differentiation is clinically relevant as the
many of the images resemble noncompressible treatment of the two conditions often differs. As
vessels. These abnormalities include Baker’s thrombus ages, it tends to be more echogenic;
cyst (a fluid-filled cyst behind the knee), pseu- however, the age of thrombus cannot be pre-
doaneurysms, hematomas, cellulitis, muscle cisely determined by the echogenicity alone as
tears, soft tissue tumors, enlarged lymph nodes, significant variation is observed. Therefore,
chronic venous insufficiency, and superficial attention needs to be paid to some other observa-
thrombophlebitis. Thrombus in a superficial vein tions that may help in the differentiation. For
142 S. Subramani et al.

Anterior needs to be applied on. Further evaluation with


alternative imaging modalities such as CT venog-
raphy and MR venography is necessary when the
acquisition of ultrasonographic images is techni-
cally challenging or when ultrasonographic
medial
results are nondiagnostic or equivocal. However,
these alternative modalities are expensive and
FA associated with complications of radiation and
FV
contrast exposure.

6.7 Outcome Studies Related


to DVT Assessment
with Ultrasonography

Over the past decades, many articles were pub-


Fig. 6.38  A free-floating thrombus in the common femo- lished establishing ultrasonography as the
ral vein in transverse view modality of choice for the DVT diagnosis when
investigating the patient upon initial presenta-
example, the size of the affected vein is useful tion. The sensitivity and specificity of ultraso-
information as acute DVT often dilates the vein, nography for symptomatic DVT are high,
whereas chronic DVT usually diminishes the reaching 90–99% in many studies [32]. However,
venous size. Development of the collateral ves- one should be aware of the unreliability when
sels is also an important finding as it is sugges- evaluating DVT in the pelvic veins and the calf
tive of chronic DVT. Additionally, a free-floating veins because, as described earlier, these veins
thrombus (Fig. 6.38) often suggests an acute eti- are less well visualized on ultrasonography.
ology, and this poses a high risk of pulmonary There has been a controversy on performing a
embolism. limited ultrasonographic examination. Examples
of a limited examination include 2-point ultraso-
nography where compression examination is
6.6.8 Limitations of Ultrasound only applied on the CFV and the PV (Fig. 6.39).
for the Diagnosis of DVT A large prospective study shows that 2-point
ultrasonography plus D-dimer is equivalent to a
Some of the limitations of ultrasonography for
DVT diagnosis are its dependency on the sonog-
rapher’s skill and its limited visualization of the Cranial
pelvic veins and calf veins (the tributaries of the
PV). The image acquisition is technically more Common
Femoral V
challenging in a patient with excess weight or
marked soft tissue edema. A more penetrating,
lower frequency probe (less than 5 MHz) may be Lateral

necessary for some obese patients to visualize the Popliteal Vein

veins in the deep region of the leg, although it


reduces the image quality. Another limitation is
that ultrasonography is not always feasible to
evaluate the entire course of the targeted vein
Fig. 6.39  Two point ultrasonography compression exam-
when a patient has an open wound or external ination is applied on the common femoral vein and the
orthopedic hardware in the area where the probe popliteal vein
6  Vascular Ultrasound 143

Cranial Questions
3 POINT DVT 1. The most commonly utilized duplex ultra-
CFV sound feature in the diagnosis of carotid
Medial artery stenosis is
Fem A. End diastolic velocity
B. Carotid index
C. Peak systolic velocity
Pop D. Continuous wave Doppler
2. The role of ultrasound in the assessment of
internal jugular vein includes all except
A. Assessing patency of IJV
Fig. 6.40  US of deep veins- three point scan B. To determine its relationship with carotid
artery
whole-leg/complete ultrasonography for diag- C. To identify vagus nerve
nosing suspected symptomatic DVT [33]. D. To estimate central venous pressure based
Several studies favor the use of 3-point limited on change in the cross-sectional area
ultrasonographic examination by adding the FV 3. The following is true based on the ultrasound
to the 2-point limited examination (Fig.  6.40), image
which may increase the sensitivity for DVT
diagnosis [34, 35]. The advantage of limited Anterior
ultrasonographic examination is a significant
reduction of the time required for the examina-
tion. In addition, it requires less technical exper-
tise and can be easily performed at the bedside
by front-line healthcare professionals with mini-
mum training. On the other hand, whole-leg/
complete ultrasonography requires more techni- Lateral
cal expertise as well as time to perform and is
often conducted by an experienced sonographer
(often by a vascular technologist) and interpreted
by a radiologist. It involves scanning the entire
lower extremities, including the calf veins with
color Doppler, pulse wave Doppler, and com-
pression examination at every 2  cm interval.
Although it is ideal, the complete examination
may not be readily performed within a clinically
acceptable time frame in many institutions, and
it could result in suboptimal patient outcomes as
a prompt DVT diagnosis at the bedside may
decrease morbidity and mortality. An under-
standing of lower extremity venous anatomy and A. It denotes ultrasound assessment of right
its common variations is essential for the perfor- IJV and carotid artery
mance of duplex ultrasound, interpretation of the B. The highlighted structure represents left
images, decision for surgery, and, ultimately, the common carotid artery
execution of radiofrequency ablation. In recent C. The highlighted structure represents right
years, the nomenclature of the lower extremity subclavian vein
venous system has been modified to eliminate D. It denotes supraclavicular approach of
confusion. subclavian vessels
144 S. Subramani et al.

4. The probe position indicates following A. It denotes ultrasound assessment of


assessment and view upper limb veins at the mid-forearm level
B. Highlighted structure in the yellow indi-
Cranial
cates brachial vein
C. Highlighted structure in the yellow indi-
cates cephalic vein
D. Highlighted structure in the yellow indi-
Pointer
towards head cates basilic vein
8. With inspiration in spontaneously breathing,
which of the following is true?
A. Abdominal pressure decreases and intra-
thoracic pressure increases
B. Abdominal pressure increases and intra-
thoracic pressure decreases
C. Abdominal pressure increases and intra-
A. Abdominal aorta in short axis thoracic pressure increases
B. IVC in short axis D. Abdominal pressure decreases and intra-
C. Superior mesenteric artery in long axis thoracic pressure decreases
D. IVC in long axis as rescue view 9. Which of the following would not increase
5. All are true in the utilization of IVC as tool to doppler velocities within a vessel?
predict fluid responsiveness except A. Enlarged segment of the vessel at the
A. Strong correlation between IVC diameter level of the carotid bulb
and collapsibility index in spontaneously B. Narrowing of a normal stented internal
breathing patients carotid artery
B. RV dysfunction influences the diameter C. Intraluminal 50–69% diameter stenosis
of IVC by causing large IVC within the internal carotid artery
C. Significant displacement of IVC due to D. Intraluminal 70–99% diameter stenosis
various abdominal factors will cause an within the internal carotid artery
error in the diameter of IVC 10. The ultrasound image represents following
D. Caval index shows consistent results view
from multiple studies
6. The following branches of aorta are visualized Anterior
relatively easy with ultrasonography except
A. Superior mesenteric artery
B. Gastric artery
C. Celiac artery
D. Inferior mesenteric artery
Lateral
7. The following is true in relation to the given
ultrasound image

Anterior

Lateral
A. Subxiphoid view for IVC assessment
B. Transabdominal longitudinal view for IVC
C. Transabdominal short-axis view of IVC
and aorta
D. Transabdominal longitudinal view of
hepatic vein
6  Vascular Ultrasound 145

11. The deep venous system of the lower extrem- 16. All the following potentially leads to false-­
ity consists of the following veins except positive or false-negative results for DVT
A. Common femoral vein except
B. Great saphenous vein A. Improperly high gain setting
C. Deep femoral vein B. Color artifact generated by improper
D. Femoral vein high color scale
E. Popliteal vein C. Inadequate venous compression while
12. The following veins are generally accompa- testing the venous compressibility
nied by an artery except D. None of the above
A. Common femoral vein E. All of the above
B. Great saphenous vein 17. Clinical symptoms and/or ultrasonographic
C. Deep femoral vein images of DVT can be similar to those of
D. Femoral vein A. Baker’s cyst
E. Popliteal vein B. Cellulitis
13. When evaluating the deep veins of the lower C. Enlarged lymph node
extremity, the better image quality of ultraso- D. Superficial thrombophlebitis
nography can be achieved by E. All of the above
A. The head elevated at 30 degrees in a 18. All of the following statements are true

reverse Trendelenburg position except
B. External rotation of the examining leg A. Most of the deep veins are accompanied
with slight knee flexion by an artery
C. A linear ultrasound probe with a high-­ B. The age of thrombus can be easily deter-
resolution 5–12 MHz mined by the echogenicity alone
D. All of the above C. A free-floating thrombus poses a high
14. The ultrasonographic findings of thrombus risk of pulmonary embolism
in the venous segment include D. Acute DVT often dilates the affected
A. Inability to compress the vein while vein
applying pressure by the probe E. Chronic DVT usually diminishes the
B. Complete collapse of the vein while venous size
applying pressure by the probe 19. The limitations of ultrasonography for DVT
C. Augmented venous flow by squeezing diagnosis include
the distal part of the vein A. Dependency on the sonographies’ skill
D. None of the above B. Limited visualization of the pelvic veins
E. All of the above and calf veins
15. All of the following statements are true
C. Technical difficulties in a patient with
expect excess weight or marked soft tissue
A. Clinical symptoms of venous insuffi- edema
ciency are similar to those of DVT D. None of the above
B. Venous insufficiency can be triggered E. All of the above
and worsened by existing venous obstruc- 20. Compared with whole-leg/complete ultraso-
tion caused by DVT or congestive heart nography, a limited ultrasonographic exami-
failure nation requires
C. Venous blood flow from a superficial A. More time
venous system is normally directed B. More technical expertise
towards the deep venous system C. Scanning the popliteal vein and its
D. The deep veins are more vulnerable to tributaries
the elevated venous pressure compared D. None of the above
with the superficial veins E. All of the above
146 S. Subramani et al.

References atic review and meta-analysis. Crit Care Med.


2015;43:1498–507.
15. Moreno FL, Hagan AD, Holmen JR, et al. Evaluation
1. Choi IS. Functional vascular anatomy of the head and
of size and dynamics of the inferior vena cava as an
neck. Interv Neuroradiol. 2003;9(Suppl 2):29–30.
index of right-sided cardiac function. Am J Cardiol.
2. Devadas D, Pillay M, Sukumaran TT.  A cadaveric
1984;53(4):579–85.
study on variations in branching pattern of external
16. Feissel M, Michard F, Faller JP, et al. The respiratory
carotid artery. Anat Cell Biol. 2018;51(4):225–31.
variation in inferior vena cava diameter as a guide to
3. Johri AM, Nambi V, Naqvi TZ, Feinstein SB, Kim
fluid therapy. Intensive Care Med. 2004;30(9):1834–7.
ESH, Park MM, et  al. Recommendations for the
17. Finnerty NM, Panchal AR, Boulger C, et al. Inferior
Assessment of Carotid Arterial Plaque by Ultrasound
vena cava measurement with ultrasound: what is
for the Characterization of Atherosclerosis and
the best view and best mode? West J Emerg Med.
Evaluation of Cardiovascular Risk: From the
2017;18(3):496–501.
American Society of Echocardiography. J Am Soc
18. Lang RM, Badano LP, Mor-Avi V, Afilalo J,
Echocardiogr. 2020;33(8):917–33.
Armstrong A, Ernande L, Flachskampf FA, Foster E,
4. Lee W. General principles of carotid Doppler ultraso-
Goldstein SA, Kuznetsova T, Lancellotti P, Muraru
nography. Ultrasonography. 2014;33(1):11–7.
D, Picard MH, Rietzschel ER, Rudski L, Spencer
5. Jahromi AS, Cinà CS, Liu Y, Clase CM.  Sensitivity
KT, Tsang W, Voigt JU.  Recommendations for car-
and specificity of color duplex ultrasound measure-
diac chamber quantification by echocardiography
ment in the estimation of internal carotid artery ste-
in adults: an update from the American Society of
nosis: a systematic review and meta-analysis. J Vasc
Echocardiography and the European Association of
Surg. 2005;41(6):962–72.
Cardiovascular Imaging. J Am Soc Echocardiogr.
6. Sisini F, Toro E, Gambaccini M, Zamboni P.  The
2015;28(1):1–39.
oscillating component of the internal jugular vein
19. Ciozda W, Kedan I, Kehl DW, Zimmer R, Khandwalla
flow: the overlooked element of cerebral circulation.
R, Kimchi A.  The efficacy of sonographic measure-
Behav Neurol. 2015;2015:170756.
ment of inferior vena cava diameter as an estimate
7. Constant J.  Using internal jugular pulsations as a
of central venous pressure. Cardiovasc Ultrasound.
manometer for right atrial pressure measurements.
2016;14(1):33.
Cardiology. 2000;93(1–2):26–30.
20. Orso D, Paoli I, Piani T, Cilenti FL, Cristiani L,
8. Ma GG, Hao GW, Yang XM, Zhu DM, Liu L, Liu H,
Guglielmo N. Accuracy of ultrasonographic measure-
Tu GW, Luo Z. Internal jugular vein variability pre-
ments of inferior vena cava to determine fluid respon-
dicts fluid responsiveness in cardiac surgical patients
siveness: a systematic review and meta-analysis. J
with mechanical ventilation. Ann Intensive Care.
Intensive Care Med. 2020;35(4):354–63.
2018;8(1):6.
21. Yao B, Liu J-y, Sun Y-b, Zhao Y-x, Li L.  The value
9. Chung CP, Chao AC, Hsu HY, Lin SJ, Hu
of the inferior vena cava area distensibility index
HH.  Decreased jugular venous distensibility in
and its diameter ratio for predicting fluid respon-
migraine. Ultrasound Med Biol. 2010;36(1):11–6.
siveness in mechanically ventilated patients. Shock.
10. Chi HY, Lin CS, Hsu MH, Chan PC, Hu HH. Chronic
2019;52:37–42.
influences of obstructive sleep apnea on cerebral
22. Via G, Tavazzi G, Ten Price S. situations where
venous flow. J Ultrasound Med. 2015;34(11):2043–8.
inferior vena cava ultrasound may fail to accurately
11. Liu M, Xu H, Wang Y, Zhong Y, Xia S, Utriainen
predict fluid responsiveness: a physiologically based
D, Wang T, Haacke EM. Patterns of chronic venous
point of view. Intensive Care Med. 2016;42:1164–7.
insufficiency in the dural sinuses and extracranial
23. Schäberle W. Ultrasonography in vascular diagnosis.
draining veins and their relationship with white matter
2nd ed. Heidelberg: Springer; 2011. p. 18, 400–403,
hyperintensities for patients with Parkinson’s disease.
441
J Vasc Surg. 2015;61(6):1511–20.
24. Chiu KW, Ling L, Tripathi V, Ahmed M, Shrivastava
12. Zivadinov R, Ramanathan M, Dolic K, Marr K,
V.  Ultrasound measurement for abdominal aor-
Karmon Y, Siddiqui AH, Benedict RH, Weinstock-­
tic aneurysm screening: a direct comparison of the
Guttman B.  Chronic cerebrospinal venous insuffi-
three leading methods. Eur J Vasc Endovasc Surg.
ciency in multiple sclerosis: diagnostic, pathogenetic,
2014;47(4):367–73.
clinical and treatment perspectives. Expert Rev
25. Schäberle W, Leyerer L, Schierling W, Pfister
Neurother. 2011;11(9):1277–94.
K.  Ultrasound diagnostics of the abdominal aorta:
13. Brass P, Hellmich M, Kolodziej L, Schick G, Smith
English version. Gefasschirurgie. 2015;20(Suppl
AF.  Ultrasound guidance versus anatomical land-
1):22–7.
marks for subclavian or femoral vein catheterization.
26. Joffe HV, Goldhaber SZ. Upper-extremity deep vein
Cochrane Database Syst Rev. 2015;1:Cd011447.
thrombosis. Circulation. 2002;106:1874–80.
14. Lalu MM, Fayad A, Ahmed O, Bryson GL, Fergusson
27. Simpson WL, Krakowsi DM.  Prevalence of lower
DA, Barron CC, Sullivan P, Thompson C. Ultrasound-­
extremity venous duplication. Indian J Radiol
guided subclavian vein catheterization: a system-
Imaging. 2010;20(3):230–4.
6  Vascular Ultrasound 147

28. Read H, Holdgate A, Watkins S.  Simple external 33. Bernardi E, Camporese G, Buller HR, Siragusa S,
rotation of the leg increases the size and accessi- Imberti D, Berchio A, et  al. Serial 2-point ultraso-
bility of the femoral vein. Emerg Med Australas. nography plus D-dimer vs whole-leg color-coded
2012;24(4):408–13. Doppler ultrasonography for diagnosing suspected
29. Robert-Ebadi H, Righini M. Should we diagnose and symptomatic deep vein thrombosis: a randomized
treat distal deep vein thrombosis? Hematology Am controlled trial. JAMA. 2008;300(14):1653–9.
Soc Hematol Educ Program. 2017;2017(1):231–6. 34. Zuker-Herman R, Ayalon Dangur I, Berant R, Sitt
30. Scott G, Mahdi AJ, Alikhan R.  Superficial vein EC, Baskin L, Shaya Y, et  al. Comparison between
thrombosis: a current approach to management. Br J two-point and three-point compression ultrasound
Haematol. 2015;168(5):639–45. for the diagnosis of deep vein thrombosis. J Thromb
31. Screaton NJ, Gillard JH, Berman LH, Kemp Thrombolysis. 2018;45(1):99–105.
PM.  Duplicated superficial femoral veins: a source 35. Pedraza Garcia J, Valle Alonso J, Ceballos
of error in the sonographic investigation of deep vein Garcia P, Rico Rodriguez F, Aguayo Lopez MA,
thrombosis. Radiology. 1998;206(2):397–401. Munoz-Villanueva MDC.  Comparison of the
32. Bhatt M, Braun C, Patel P, Patel P, Begum H, accuracy of emergency department-performed point-­
Wiercioch W, et  al. Diagnosis of deep vein throm- of-­
care-ultrasound (POCUS) in the diagnosis of
bosis of the lower extremity: a systematic review lower-extremity deep vein thrombosis. J Emerg Med.
and meta-analysis of test accuracy. Blood Adv. 2018;54(5):656–64.
2020;4(7):1250–64.
Focused Assessment
with Sonography in Trauma (FAST) 7
Exam

Vishakha Prasad Erasu and Priyadarshini Marathe

7.1 Definition sonography in trauma” or FAST scan as it has


come to be known today was developed based on
The Focused Assessment with Sonography in this and has four views of the torso. Studies that
Trauma (FAST) exam is a point of care Ultrasound looked at the impact of focused ultrasound in
exam to assess the presence of pathological fluid trauma patients showed a decrease in the delay
within the intraperitoneal, intrathoracic, and peri- times from admission to operating theatre [2, 3]
cardial spaces in patients presenting with blunt or and a benefit in performing the FAST exam in
penetrating trauma to the torso region. hypotensive patients with blunt truncal trauma
[4]. The newer protocol of e-FAST exam or
Extended Focused Assessment with Sonography
7.2 Introduction/Background in Trauma was proposed by Kirkpatrick et al. in a
paper in the Journal of Trauma in 2004 [5]. The
7.2.1 History of the FAST Exam extended FAST exam includes two additional
views of the anterior most areas of the chest to
The term FAST representing “focused assessment look for the presence of a pneumothorax. As it
by sonographic examination of the trauma gained popularity, the FAST exam has been com-
patient”  was first used towards the end of the pared to the diagnostic peritoneal lavage (DPL)
twentieth century to describe the use of ultrasound and the CT scan performed on the trauma
in abdominal trauma. In 1996 Rozycki described patient  (Table 7.1) [6, 7]. The FAST exam has
the FAST exam consisting of three views of the been recommended in the assessment of trauma
abdomen to rule out bleeding in abdominal trauma patients as part of the Advanced Trauma Life
[1]. The widely used “focused assessment with Support (ATLS) training since 1997 [8]. It has
become part of the management of trauma patients
V. P. Erasu (*) in most trauma centers in many parts of the world.
Oxford University Hospitals NHS Foundation Trust,
Oxford, UK
Emergency Medicine Research in Oxford, 7.2.2 Anatomy and Physiology of
Oxford, UK Fluid Collection in Body Cavities
e-mail: [email protected]
P. Marathe In the trauma patient with internal bleeding,
Oxford University Hospitals NHS Foundation Trust,
blood (as does fluid) tends to collect in the depen-
Oxford, UK
https://www.linkedin.com/in/ dent regions based on the position of the patient
dr-priyadarshini-marathe-56519677/

© The Author(s), under exclusive license to Springer Nature Singapore Pte Ltd. 2022 149
A. Chakraborty, B. Ashokka (eds.), A Practical Guide to Point of Care Ultrasound (POCUS),
https://doi.org/10.1007/978-981-16-7687-1_7
150 V. P. Erasu and P. Marathe

Table 7.1  Comparison of DPL vs. FAST exam vs. CT scan. This table compares the salient features of the FAST exam,
DPL, and CT scan in the trauma patient
Feature DPL FAST exam CT scan
Bedside test Yes Yes No
Needs transport No No Yes
Ease of performing test in an Can be done Can be done Not without risk of
unstable patient transfer and time
Performed simultaneously with Yes Yes No
clinical assessment
Invasive procedure Yes No No
Radiation exposure No No Yes
Ease of a repeat test Cumbersome Easily done Increases radiation
risk
Operator dependent Yes Yes No
Affected by previous surgery Yes (if altered anatomy like Yes (if altered No
adhesions, etc.) anatomy)
Time to perform test Time to instill lavage fluid and Few minutes 10–15 min excluding
aspirate—Average of 10–15 min transfer times
Cost of test Low Low Higher

and the site of bleeding. As these patients are usu- male and female patient. In the female patient,
ally positioned supine, the dependent regions the bladder lies anterior to the uterus, with the
become the hepatorenal recess, splenorenal rectum lying most posteriorly. The vescicovagi-
recess, and pelvis within the peritoneal cavity. In nal or Douglas pouch is the dependent region
the pleural cavity, the fluid settles posteriorly and formed between the fascia overlying the bladder
inferiorly within each pleural space. If the patient and uterus. In the male patient, the rectovescical
assumes a sitting posture, free fluid would collect pouch is the space between the bladder and rec-
in the pelvis as this would become the most tum. The vescicovaginal and rectovescical
dependent area of the peritoneal cavity. The pouches, form the most dependent pockets for
FAST exam takes advantage of this phenomenon fluid collection in the female and male pelvis,
to rapidly detect fluid (i.e., blood, urine, or faecal respectively.
matter) in these dependent spaces by only focus- The site of injury and hence the location of the
ing on these regions. The FAST exam can detect bleeding point influences the collection of blood
a minimum of 200–250 mls of fluid in the perito- in the dependent regions of the peritoneal cavity.
neal cavity. Injury in the right upper quadrant will cause
The hepatorenal recess is the potential space blood to collect in Morrison’s pouch and then
in the right upper quadrant of the peritoneal cav- flow into the subdiaphragmatic and pelvic spaces.
ity. It lies between the liver and the right kidney The left upper quadrant differs from the right
and in physiological conditions appears as a upper quadrant as blood tends to collect in the
hyperechoic line on ultrasound where the fascial subdiaphragmatic space more often than in the
layers adjoin each other. The splenorenal recess splenorenal space. Injury to the pelvic region will
is the potential space in the left upper quadrant of cause blood to collect in the pelvis but large pel-
the peritoneal cavity and lies between the spleen vic bleeds can cause blood to flow into the
and the left kidney. In physiological situations Morrison’s pouch as well.
when there is no fluid or blood in this space, it In the pleural cavity, fluid or blood in the
appears as a hyperechoic line on ultrasound rep- supine patient tends to collect in the dependent
resenting the adjoining fascia of the spleen and regions. In each hemithorax fluid will collect in
left kidney. The anatomy of the pelvis varies in the posterior and inferior parts with the lung
7  Focused Assessment with Sonography in Trauma (FAST) Exam 151

floating in the free fluid. Air does the opposite by 7.3 Focused Question in FAST
rising to occupy the nondependent regions and Exam
will collect above the respective lung. In the
upright patient, this will be the upper part of the The objective of the FAST exam is to answer the
hemithorax while in the supine patient this will question—“is there fluid?” within the pericar-
correspond to the anterior most part of the dial, pleural, and peritoneal spaces by obtaining
hemithorax. the right upper quadrant, left upper quadrant,
The pericardial space lies within the two lay- pelvic and subcostal views using a low-fre-
ers of the pericardium, creating a potential space quency probe.
for fluid to collect around the heart. This potential
FAST scan pearls
space has a much smaller capacity than the pleu-
1. ONE Question—Is there free fluid?
ral and peritoneal cavities and fluid here tends to
2. TWO Possible answers—Yes or no (if unclear view
surround the heart forming a thin film and then then cannot determine)
expanding as more fluid collects. 3. THREE Body cavities—Pericardium, peritoneum
and pleural
4. FOUR Views—RUQ, LUQ, pelvic, and subcostal
7.2.3 Diagnostic Peritoneal Lavage 5. FIVE MHz or less- frequency of probe

Diagnostic Peritoneal Lavage (DPL) in trauma


patients was first described by Root in 1965 [9] 7.4 Indications of FAST Exam
but over the years has been replaced by the FAST
exam and CT scan. This procedure consists of an The indications for the FAST exam in the trauma
initial diagnostic tap which consists of aspirating patient are:
for fluid from the abdominal cavity to confirm the
presence of blood. The lavage can be performed • Blunt or penetrating trauma to the chest.
as the second step and is very sensitive in detect- • Blunt or penetrating trauma to the abdomen/
ing blood in the peritoneal cavity. The diagnostic pelvis.
peritoneal lavage test is considered positive when • Any unexplained shock—to look for a possi-
blood is aspirated from the peritoneal cavity on ble cause.
direct tap or lavage. Detailed explanation of DPL
is out of scope of this chapter. The FAST exam can be used outside the con-
text of trauma and in unstable patients without a
clear history of trauma to assess for the presence
7.2.4 CT Scan in the Trauma Patient of fluid in the body cavities. In this case, the scan
is called a FAFF (focussed assessment for free
The diagnostic imaging of choice in the trauma fluid) scan.
patient depends on the findings of the ATLS pri-
mary survey in addition to the mechanism of
injury and patient characteristics. When assess- 7.5 Contraindications
ing the patient with blunt or penetrating trauma to
the torso region, a CT scan is the imaging of A FAST exam should never delay or hinder the
choice which may be done as part of a pan scan resuscitation of the trauma patient. This is the
(consisting of CT scans of the head, cervical only situation-based contraindication for a FAST
spine, and torso) or regional scan of the chest, exam and there are no other absolute contraindi-
abdomen, and pelvis independently [10, 11]. cations to performing a FAST exam.
152 V. P. Erasu and P. Marathe

7.6 Preparation on the right, the images received will be reversed


to those shown in this chapter.
The FAST exam is performed as an adjunct of the
primary survey in trauma patients. The general
recommendation is to position the ultrasound 7.9 Scanning Technique
machine to the right side of the patient if the
operator is right-handed and to the left side if the The order of scanning to obtain the four views of
operator is left-handed. This allows the dominant the FAST exam (Fig.  7.1) is determined by the
hand to be used for scanning and the nondomi- region injured and on the clinical suspicion of
nant hand to manage the controls on the machine. finding blood. For example, in case of a penetrat-
It is recommended to use necessary personal pro- ing injury to the left torso, the FAST exam can
tective equipment and probe or device covers for begin with the pericardial view followed by the
infection control as per local policy. Ideally, ster- splenorenal recess and then the other abdominal
ile gel sachets should be used for each scan to views. Similarly, in a patient with an injury to the
reduce the risk of contamination. pelvic region, the FAST exam can begin with the
pelvic views. When the site of injury is unclear it
is reasonable to begin by scanning the hepatore-
7.7 Position of Patient nal recess moving on to the splenorenal recess,
followed by the pelvic and cardiac views.
The trauma patient is usually positioned supine Irrespective of the order of scanning the operator
and this position facilitates the accumulation of must ensure that all areas have been assessed to
fluid in the potential body spaces as discussed complete the FAST exam.
above. Occasionally, if the injury permits, the
trauma patient may be in the sitting position, and
in such instances, fluid will gravitate to the depen-
dent portions of the body cavities, i.e., the lower
parts of the pleural cavities or the pelvis. If the
patient is sitting up then in a patient with a pneu-
mothorax air will collect superiorly and underlie
the upper anterior ribs, in contrast to the lower
anterior chest when the patient is examined in the
supine position. This should be considered while
performing an e-FAST (Extended FAST) exam.

7.8 Probe and Machine Preset

A low-frequency probe of 2–5 Hz is preferred for


the FAST exam to visualize structures in depth. If
a wider footprint is preferred the curvilinear
probe is used whereas the phased array probe can
be used to enhance visualization between the ribs
given its narrower footprint.
Newer Ultrasound machines have the option
to select a preset for the probe. The Abdominal
Exam or FAST Exam preset is the recommended
choice with the marker on the screen placed on
the left. If the marker is placed unconventionally Fig. 7.1  Standard FAST views—Probe positions
7  Focused Assessment with Sonography in Trauma (FAST) Exam 153

7.10 Views Lung


Diaphragm
7.10.1 Right Upper Quadrant View Liver (including tip of liver)
Morison’s pouch
Right kidney (both upper and lower poles)
The probe is placed between the anterior and
Right psoas major muscle
midaxillary lines in the 10th–11th intercostal
space, orienting the marker or probe indicator Characteristic features of this view (Fig. 7.3):
towards the patient’s head (Fig. 7.2). The probe is
aimed towards the patient’s vertebral column • The lung contains air which is a poor conduc-
starting with a depth selection of more than tor of sound while the liver provides a good
20  cm, and then adjusting to get the optimum sonographic window. The diaphragm acts as a
image. Identifying the liver with its hepatic sinu- specular reflector at this tissue-air interface
soids is the first step before proceeding to iden- and hence a normal lung appears as a mirror
tify other structures. Then sliding the probe up image of the liver.
and down a rib space or more, while fanning the • If there is fluid in the pleural cavity this
probe, continue to look for an anechoic stripe of appears as an anechoic black area (Fig. 7.4).
fluid in the hemithorax above the diaphragm Fluid is a good conductor of sound and the
(suggestive of hemothorax) or in the peritoneal thoracic vertebral bodies that are not normally
cavity below the diaphragm (suggestive of hemo- visible above the diaphragm due to air in the
peritoneum). The structures that are visualized lungs can now be visualized. This is known as
craniocaudally in this view are: the thoracic spine sign or spine sign.
• When a significant amount of fluid collects in
the pleural cavity, the lung becomes atelec-
tatic and collapses. This collapsed lung dances
with respiration in the anechoic fluid, giving
the appearance of a whale tail or a jellyfish
sign.
• The diaphragm appears as a curved bright
white line due to specular reflection.
• The liver is identified by its homogenous
appearance and sinusoids. It is a good conduc-
tor of sound providing an acoustic window for
visualizing the other structures in the region.
• Morison’s pouch or hepatorenal recess is nor-
mally collapsed and appears as a hyperechoic
white line between the liver and right kidney.
When fluid collects in this recess, it appears as
a black anechoic stripe between the liver and
the right kidney.
• The right kidney is identified by its character-
istic shape and the presence of an outer cortex
and inner medulla.
• It is important to fan the probe and slide up
and down the ribs to visualize the liver and
kidney in entirety. The tip of the liver and both
poles of the kidney should be looked for and it
Fig. 7.2  The probe placement for the right upper quad- should be documented if unable to obtain a
rant view clear view of these structures.
154 V. P. Erasu and P. Marathe

Fig. 7.3  The right


upper quadrant view Lung
showing a section of the
liver and the right Diaphragm
kidney. There is no
pathological fluid seen
in this image Liver (including tip of liver)

Morison’s pouch

Right kidney (both upper and lower poles)

Right psoas major muscle

Fig. 7.4  Shows free


fluid in the space
between the liver and
right kidney as a black
anechoic stripe
7  Focused Assessment with Sonography in Trauma (FAST) Exam 155

• The psoas muscle is located posterior to the


kidney and fluid can collect anterior to it in
some instances.
• The patient can be requested to take a deep
breath, which helps move the above structures
caudally for better visualization. During deep
inspiration the side of the chest opens up,
placing the ribs in a more horizontal position
and increasing the space between the ribs thus
improving the image by reducing rib shadows.
The patient can be asked to move their arm out
of the way by placing their hand near their
head which again makes the ribs more hori-
zontal while opening up the space between the
ribs.
• Rib shadows can cause hindrance in visualiz-
ing structures, leaving the operator with an
unclear picture. In such instances, it helps to
remember the anatomy of the rib cage and
rotate the Ultrasound probe to position it par-
allel to and in between the ribs. Posteriorly the
ribs leave the transverse processes and travel
downwards and laterally and, in the front, they
travel upwards and medially to join the
Fig. 7.5  The probe placement for the left upper quadrant
sternum. view

7.10.2 Left Upper Quadrant View Characteristic features of this view (Fig. 7.6):

The probe is placed in the 7–9th intercostal • The lung is difficult to visualize in this view. It
space in the posterior axillary line with the may be seen as a curtain that comes across the
operator’s hand almost resting on the surface screen as the patient breathes.
that the patient is lying on (Fig. 7.5). The probe • The diaphragm is visualized as a hyperechoic
marker is towards the patient’s head and the semilunar structure and the space between the
probe tilted to look slightly anteriorly towards diaphragm and spleen is a common site for
the vertebral column with a depth selection of blood to collect in case of splenic lacera-
more than 20 cm, adjusting later to get the opti- tions (Fig. 7.7).
mum image. The spleen has a similar echotex- • The splenorenal recess and left kidney should be
ture to the liver but is located more posteriorly, visualized in entirety, fanning the probe to look
hidden under the ribs. The structures visualized for fluid. Fluid will appear as an anechoic black
in this view are: stripe between the spleen and the left kidney.
• The psoas muscle is located posterior to the
Lung kidney and fluid can collect anterior to it in
Diaphragm some instances.
Spleen • Rib shadows can cause hindrance in visualiz-
Splenorenal recess ing structures and the tips given above (right
Left kidney (both upper and lower poles) upper quadrant view) can be used to mitigate
Left psoas major muscle
these.
156 V. P. Erasu and P. Marathe

Lung

Diaphragm

Spleen

Splenorenal recess

Left Kidney (both upper and lower poles)

Left psoas major muscle

Fig. 7.6  The left upper quadrant view showing the diaphragm, a section of the spleen, and the left kidney. There is no
pathological fluid seen in this image

7.10.3 Pelvic Cavity: Sagittal female patient. Rotate the probe 90° clockwise so
and Transverse Views that the probe marker is now pointing towards the
patient’s head, maintaining the tilt to look into
The pelvic cavity is assessed in both transverse the pelvic cavity (Fig. 7.8). Now fan or slide the
and longitudinal sections. Begin with the probe probe from side to side to look for fluid in the
in the suprapubic region with the probe marker to pelvis. The structures visualized in this view are:
the right of the patient. Slide the probe caudally
Female patient:
to almost touch the pubic symphysis and then
Urinary bladder
angle the probe away from the patient’s feet, to
Vesicouterine pouch/pouch of Douglas
look into the pelvic cavity with a depth selection Uterus
of 15 cm. Fan the probe, cranio-caudally to look Rectouterine space
for fluid that appears as an anechoic black line Rectum
behind the bladder or behind the uterus in a
7  Focused Assessment with Sonography in Trauma (FAST) Exam 157

Fig. 7.7  Shows free


fluid as a black anechoic
stripe between the
diaphragm and spleen in
the left upper quadrant
view

Characteristic features of this view (Figs. 7.9


and 7.10):

• When empty, the urinary bladder lies col-


lapsed within the pelvis and when full, it can
rise all the way up to the umbilicus. The uri-
nary bladder is better visualized when con-
taining urine and provides an acoustic window
to assess the space behind the bladder where
fluid can potentially collect.
• If the bladder is full, then an enhancement of
the structures posterior to the bladder may
cause them to appear hyperechoic. The TGC
(time-gain-compensation) can be adjusted to
get around this artifact.
• The vesicouterine and rectouterine spaces in
the female patient and the rectovesical space
in the male patient are normally collapsed.
When fluid collects in them, they appear as
anechoic black areas on ultrasound (Figs. 7.11
and 7.12).
• Small pockets of fluid may be easily missed;
hence, it is important to fan the probe to look
for fluid in all areas of the pelvis.
Fig. 7.8  The probe placement for the 2 pelvic views • Physiological fluid in the pelvis of the young
female patient is common and can mimic a
positive FAST scan in trauma. Clinical
Male patient: presentation, patient’s hemodynamic status
­
Urinary bladder and surgical opinion along with further imag-
Rectovesical space ing with CT should be considered to confirm
Rectum
the cause of the fluid on a case-to-case basis.
158 V. P. Erasu and P. Marathe

Female patient:

Urinary bladder

Vesicouterine pouch / pouch of Douglas

Uterus

Rectouterine space

Rectum

Male patient:

Urinary bladder

Rectovesical space

Rectum

Fig. 7.9  The transverse view of the pelvis showing a well-filled urinary bladder. There is no pathological fluid in this
image

• Occasionally, fluid in the rectum may be 7.10.4 Pericardium


mistaken as fluid in the pelvis. In such cases,
tilting the patient while maintaining neces- The subcostal view is usually used to access the
sary spinal precautions, to see if the fluid heart, but if a clear view is not obtained due to
remains in the same place or moves into the abdominal viscera obstructing the view, the para-
paracolic gutters helps to differentiate sternal long-axis view can be used instead. The
between the two. subcostal view can be accessed using the curvi-
7  Focused Assessment with Sonography in Trauma (FAST) Exam 159

Fig. 7.10  The sagittal


view of the pelvis
showing a well-filled
urinary bladder. There is
no pathological fluid in
this image

Fig. 7.11  Shows free


fluid in the pelvis in
transverse view

linear probe or the phased array probe. The when using this probe as the images will be mir-
marker on the screen for the phased array probe rored accordingly. Alternatively, newer machines
is conventionally placed to the right of the screen will have an abdominal preset option with the
in the cardiac preset. If the probe marker is to the cardiac probe which can be selected for the sub-
right of the patient in cardiac preset, the struc- costal views of the FAST exam.
tures on the patient’s right will appear on the left Before placing the probe on the patient, the
of the screen and vice versa. The probe marker depth should be increased to 20 cm as the struc-
may be placed to the left of the patient if the oper- tures to be visualized are situated at a considerable
ator would like to obtain images with the right distance from the probe. Place the probe in the epi-
side of the patient corresponding to the right side gastrium of the patient and slide up to almost touch
of the screen while using the phased array probe the xiphisternum. Now tilt the probe to aim the
in cardiac preset. This should be kept in mind probe towards the patient’s left shoulder, attempt-
160 V. P. Erasu and P. Marathe

Fig. 7.12  Shows free


fluid in the pelvis in
sagittal view

ing to look just beneath the ribs to locate the heart


through the acoustic window of the liver. If this
view is unclear then proceed to attempt the para-
sternal long-axis view. This view is obtained by
placing the probe on the left sternal border in the
third or fourth intercostal space with the probe
marker towards the right shoulder. Alternatively,
the probe marker can be towards the left hip to
obtain images with the patient’s right correspond-
ing to the right side of the screen  (Fig. 7.13).
Structures visualized in this view:

Subcostal view:
Liver
Heart
Parasternal long-axis view:
Heart
Descending aorta

Characteristic features of the subcostal


view (Fig 7.14):

• The liver provides an acoustic window for


visualizing the heart in the subcostal view and
is the structure seen closest to the probe. The
closest chamber to the liver is the right ven- Fig. 7.13  The probe placement for the subcostal and
parasternal views
tricle. and all four chambers of the heart can
be visualized in this view.
• Pericardial fluid (Fig. 7.15) in small amounts for any anechoic shadow behind the heart to
tends to collect posteriorly and as the quantity avoid missing a small pericardial collection.
increases it surrounds the heart and is detected This is done by fanning through the heart in an
anteriorly. Therefore, it is important to look anteroposterior direction.
7  Focused Assessment with Sonography in Trauma (FAST) Exam 161

Fig. 7.14  The subcostal


view showing the liver Subcostal view:
(structure closer to the
probe) and the heart Liver

Heart

Parasternal long axis view:

Heart

Descending aorta

Fig. 7.15  Free fluid is


seen in the pericardial
cavity on subcostal view
162 V. P. Erasu and P. Marathe

• While attempting to look for the heart beneath • In the parasternal long-axis view the descend-
the ribs the probe is placed tangentially on the ing aorta is posterior to the heart. This is used
skin. Care must be exercised to keep the probe as a landmark to differentiate whether the
in contact with the patient’s skin at all times to anechoic black shadow seen posterior to the
avoid compromising on the image quality. A heart is a pericardial or pleural collection. A
generous amount of ultrasound gel will help pericardial collection (Fig. 7.17) will be situ-
maintain good probe-to-skin contact. ated anterior to the aorta whereas pleural fluid
• Occasionally the subcostal view may not pro- is located posterior to the descending aorta. If
vide a clear image due to a barrel chest or any fluid is detected in the subcostal view it is
abdominal viscera obstructing or a tender recommended to confirm it on a parasternal
abdomen. In such instances, the parasternal long-axis view because the subcostal view
long-axis view (Fig. 7.16) can be obtained and may mistake pleural fluid for pericardial fluid.
the depth setting should be decreased to • Other views that can be used to supplement
10–15 cm before proceeding with this view. imaging of the heart for fluid in the pericardial

Fig. 7.16 The
parasternal long-axis
(PLAX) view showing
the heart

Fig. 7.17  Free fluid is


seen around the heart
both anteriorly and
posteriorly in the
parasternal long-axis
view
7  Focused Assessment with Sonography in Trauma (FAST) Exam 163

cavity are the parasternal short-axis view and pointing towards the patient’s head. The struc-
the apical four-chamber view (please refer to tures towards the head end of the patient will
chapter on ECHO for this). appear on the left side of the screen. Once a cer-
• When performing a FAST scan, the operator tain rib space is examined in both 2D mode and
must be aware of the features of cardiac M-mode, the probe is moved caudally to cover
­tamponade to differentiate it from a pericar- most of the rib spaces, repeating the assessment
dial effusion sans tamponade. Identification of for features of a pneumothorax bilaterally. The
the features of cardiac tamponade in the structures visualized in this view are:
hemodynamically unstable trauma patient
Skin
necessitates immediate emergency manage-
Subcutaneous tissues
ment. The features on ultrasound that indicate
Adjoining ribs with intercostal muscles
cardiac tamponade are pericardial effusion Parietal pleura and lung sliding
plus: Superficial lung parenchyma
• Right ventricular (RV) collapse in diastole.
• Right atrial (RA) collapse in systole. Characteristic features of this view:
• Plethoric IVC with minimal respiratory
variation. • The focus of the image obtained is the bright
pleural line in the middle of the screen with
The collapse of the RA/RV due to cardiac the two adjoining ribs and their shadows on
tamponade appears as a wavy seesaw on ultra- either side.
sound and it has been referred to as the “little • Lung sliding is the gliding of the parietal
man jumping on the trampoline” sign. pleura on the visceral pleura during breathing
and is appreciated as “ants marching” on the
white pleural line. This movement has to be
7.11 Extended FAST or e-FAST appreciated in real time and is absent when
Scan there is air between the two pleural surfaces as
in a pneumothorax.
The extended FAST exam was first proposed in • Occasionally a lung point is appreciated at the
2004 and included bilateral chest ultrasound junction of the pneumothorax and normal
exams to detect the presence of a pneumothorax lung where lung sliding abruptly ends. The
in addition to the four views of the FAST exam. normal lung parenchyma may show a few B
Ideally, the high-frequency linear probe is used lines or comet tails which may also end
for assessing the chest for features of a pneumo- abruptly at the lung point.
thorax as the pleura is a superficial structure. If • The M-mode is then used to complement the
this probe is not available the scan can be per- findings of the 2D mode. The line of the
formed with the low-frequency probes. In the M-mode is placed over the pleural line and the
supine trauma, patient air collects in the anterior-­ obtained image is analyzed. If the upper half
most part of the thorax, whereas in an upright of the image shows horizontal lines with a
trauma patient the air will collect in the apical grainy bottom half, the appearance is called a
regions of the thorax. The anterior-most rib space sandy beach sign and indicates a normal lung.
is different for each patient and is usually between If the entire image shows lines throughout,
the fourth to sixth intercostal spaces. The opera- without the sandy granular appearance, it is
tor may face difficulty in obtaining a good view if known as the barcode or stratosphere sign and
the patient’s breast tissue interferes with the is characteristic of a pneumothorax.
imaging. • It is important to look in multiple intercostal
The exam is performed by placing the probe in spaces for a pneumothorax.
the second intercostal space in the mid-clavicular • The presence of subcutaneous emphysema
line in a linear fashion with the probe marker hinders the use of ultrasound to assess for a
164 V. P. Erasu and P. Marathe

pneumothorax (air is a poor conductor of void in the service that can be delivered in
sound) and in such instances other imaging such cases.
modalities should be used. 7. Training—As with the availability of equip-
(Also see chapter on lung for more ment the access to training may limit the use
information) of the FAST scan in some parts of the world.
The availability of virtual learning platforms
can help overcome this problem.
7.12 Limitations of FAST Scan

1. Amount of fluid—There has to be a signifi- 7.13 FAST Scan Flowchart


cant amount of fluid in the potential spaces for (Fig. 7.18)
detection via ultrasound. This amount is esti-
mated at 200–250 mL in the peritoneal cavity.
In the pericardial space, effusions as small as
50 mL can cause pericardial tamponade [12]. 7.14 Reporting a FAST Scan
2. Rule-in (vs.) Rule-out—The FAST exam
should always be used as a Rule-in test and The FAST exam is always reported for the par-
should not be used to rule out pathology. A ticular time at which it was performed and can
patient who has a negative FAST scan can still change as the clinical status of the patient pro-
have an injury [13] but has not yet collected gresses. Therefore, FAST exam reports should
enough blood in the peritoneal cavity for it to mention the time at which the exam was per-
be detected on ultrasound. formed. Images of the FAST exam should be
3. Retroperitoneal fluid—A FAST scan cannot saved on the device and serve as a baseline scan
detect fluid in the retroperitoneal space and for clinical correlation at a later time.
this should always be considered as an area of A report on a FAST exam should indicate
potential bleed in the unstable trauma patient. whether all views were clearly visualized and
4. Subcutaneous emphysema—Air is a poor whether any fluid was present in any of those
conductor of sound hence the presence of sub- views. If certain regions of a view were not
cutaneous emphysema will hinder visualiza- clearly visualized this must be commented on. A
tion of any underlying pathology. FAST report should also recommend the next
5. Operator dependent—The speed and accu- clinical management, e.g., no further imaging,
racy of the FAST exam is dependent on the repeat FAST scan in certain time, proceed to CT
training and experience of the operator. or urgent management in theatre. Below is a sam-
6. Equipment—In many areas of the world, an ple template for the FAST exam used at the
ultrasound machine is not available for the Oxford University Hospitals, UK.
management of the trauma patient leaving a
7  Focused Assessment with Sonography in Trauma (FAST) Exam 165

POCUS FAST
(Focused Assessment by Sonography in Trauma)
OPERATOR SUPERVISOR OBSERVER
DATE: TIME: LOCATION:

Before scan:
What is the indication for this scan?
Patient consent obtained?
Procedure explained?

Comment on clarity
of views obtained
Free fluid seen?
Please use Any other
FAST scan location Please say
C=clear comments?
YES or NO
P=Partial
N=not seen
RIGHT UPPER QUADRANT VIEWS
Right Hemithorax
Right side of diaphragm
Liver
Morrisons space
Tip of the liver
Upper pole of right
kidney
Lower pole of right
kidney
Right psoas muscle
LEFT UPPER QUADRANT VIEWS
Left Hemithorax
Left side of the
diaphragm
Spleen
Splenorenal space
Upper pole of left
kidney
Lower pole of left
kidney
Left psoas muscle
PELVIC VIEWS
Transverse
Longitudinal
PERICARDIAL VIEWS
View used?

Any other scans done?


Extended – FAST? e.g. Lungs, MSK, eye etc – please write your findings below.

After scan:
Images saved?
Patient informed of scan findings?
POCUS machine cleaned and plugged in safely at designated area?

Summary of FAST scan


Clinical decision taken post scan?
166 V. P. Erasu and P. Marathe

Patient with potential Trauma to the Abdomen

Assess Haemodynamic Stability

STABLE UNSTABLE

Remains
unstable
YES Resuscitate as per ATLS;
If history or other injury
(pelvic/spinal) indicates perform FAST exam
need for CT

NO

No Free fluid
FAST exam

Stabilises

CT scan

Free Fluid

No Free fluid Free Fluid

Theatre;
Based on clinical
Consider repeat FAST CT scan to define situation and resources –
exam at regular intervals; can have CT scan on
No further investigations route to theatre
if stable and FAST
remains negative.
If becomes unstable
move to Unstable* arm
of flowchart

Fig. 7.18  This flowchart shows possible clinical pathways for the trauma patient with an abdominal injury based on
the patient’s hemodynamic stability and FAST exam results
7  Focused Assessment with Sonography in Trauma (FAST) Exam 167

7.15 FAST Scan vs. US Abdomen simultaneously during the clinical examination
as a bedside test and can be repeated as needed. A
A FAST exam is clearly different from an ultra- FAST scan should never delay or interrupt resus-
sound abdomen exam with regards to the purpose citation measures.
and procedure of the scan. A FAST exam is per-
formed in trauma and focuses on the question of
whether there is fluid in the peritoneal cavity  elf-Assessment Questions: Chose
S
(along with pleural and pericardial spaces). It the Single Best Answer
does not comment on any of the internal organs
while an ultrasound abdomen assesses all abdom- 1. The FAST scan includes all of the following
inal structures looking for pathology that can views EXCEPT:
explain the presenting symptoms. A FAST scan A. Right upper quadrant
is easier to learn and recommended as part of B. Left upper quadrant
training for emergency physicians, critical care C. Pelvic
physicians, and prehospital medical personnel. D. Longitudinal view of the aorta
2. You are going to begin your FAST scan with
the abdominal exam. The probe that can be
7.16 Caution used for this exam is:
A. The curvilinear probe with abdominal/
Do NOT delay resuscitation to perform a FAST exam. FAST preset
Always report the FAST scan for a particular time in
the patient’s journey.
B. The cardiac probe with abdominal preset
Always use the FAST exam as a rule-in test and not to C. Both of the above
rule out pathology. D. Only the linear probe
Do NOT clear a region as negative for fluid unless all 3. You are going to perform the extended FAST
areas are clearly visualized. scan. Which probe is best suited for visual-
izing the pleura and why?
A. Linear probe because of its high
frequency
7.17 Summary B. Linear probe because of its low
frequency
The FAST exam is a useful tool in unstable C. Phased array probe because of its low
trauma patient and should be used as part of the frequency
primary survey when available. A clear under- D. None of the above
standing of the physiology of pathological fluid 4. The Morison’s pouch is the most dependent
collection within potential body spaces and the area in the abdomen of the supine patient.
effect of gravity on the location of this fluid is This lies between the:
necessary before attempting the FAST scan. The A. Spleen and left kidney
FAST exam is performed with a low-frequency B. Liver and right kidney
probe and attempts to answer the question of C. Urinary bladder and rectum
whether there is any fluid within the three body D. Urinary bladder and uterus
spaces. These are the peritoneal, pleural, and 5. All of the following are expected to be visu-
pericardial spaces and are examined in four alized on the left upper quadrant view
views, namely the right upper quadrant, left EXCEPT:
upper quadrant, pelvic, and cardiac. It is abso- A. Heart
lutely vital to visualize the four views clearly B. Diaphragm
before reporting a negative scan. A useful advan- C. Spleen
tage of the FAST scan is that it can be performed D. Left kidney
168 V. P. Erasu and P. Marathe

6. The FAST exam on a hemodynamically D. All of the above


unstable young gentleman involved in a sig- 11. Physiological fluid is commonly found in the
nificant motor vehicle collision is positive. following space in the FAST scan:
What would be the next step in the manage- A. Hepatorenal recess
ment of this patient (ATLS management B. Splenorenal recess
ongoing) C. Pelvis of the female patient
A. DPL to check if the fluid in the abdomen D. Pleural spaces
is blood 12. The next step when unable to obtain a sub-
B. Observe and repeat FAST scan when costal view of the pericardium is to:
patient becomes stable A. Press harder while pointing to the right
C. CT scan if stabilizes by resuscitation, shoulder
else emergency exploratory laparotomy B. Obtain the parasternal long-axis view
in theatre C. Proceed with other views of the FAST
D. Repeat FAST scan every 5  min till the scan
patient is stable D. Perform a transthoracic echocardiography
7. The FAST exam on a hemodynamically sta- 13. The direction of the probe when obtaining
ble young lady who fell down a flight of the subcostal view is pointing towards the:
stairs after a few drinks is negative. She is A. Left shoulder
tender in the abdomen and confused. What B. Right shoulder
would be the most appropriate next step in C. Suprasternal notch
the management of this patient? D. Umbilicus
A. Repeat a FAST scan if clinical picture 14. The depth setting for obtaining the subcostal
changes view is:
B. Pan CT to exclude injuries from head to A. 2 cm
pelvis B. 5 cm
C. Discharge home to sober up C. 10 cm
D. Get a radiologist to perform a bedside D. 20 cm
abdominal ultrasound 15. While performing the extended FAST exam,
8. The following non-trauma patients benefit the sign that signifies the presence of a pneu-
from having a FAST/FAFF scan: mothorax in the M-mode is called:
A. Suspected ruptured ectopic pregnancy A. Seashore/Sandy beach sign
B. Medical patient with unexplained B. Barcode/Stratosphere sign
hypotension C. Trampoline sign
C. Both D. Fishtail sign
D. None 16. The FAST exam should be performed:
9. The following are normal findings while per- A. To look for pathological fluid in the
forming an extended FAST scan EXCEPT: unstable trauma patient
A. A-lines B. To rule out any abdominal pathology in
B. Occasional B-lines the trauma patient
C. Lung Sliding C. In every trauma patient irrespective of
D. Lung point hemodynamic status and injury
10. The full bladder can enhance structures situ- D. By the senior-most clinician available
ated posterior to it and make them appear 17. Signs of Cardiac Tamponade are:
hyperechoic. The adjustment that can help to A. Large pericardial effusion
make the image better is: B. Right ventricular (RV) collapse in
A. Increase depth diastole
B. Time-gain-compensation C. Right atrial (RA) collapse in systole
C. Increase gain D. All of the above
7  Focused Assessment with Sonography in Trauma (FAST) Exam 169

18. The presence of fluid in the thorax, e.g.,


2. Rozycki GS, Feliciano DV, Schmidt JA, Cushman
JG, Sisley AC, Ingram W, et al. The role of surgeon-
Hemothorax on a FAST scan is seen as: performed ultrasound in patients with possible cardiac
A. Anechoic shadow above the diaphragm wounds. Ann Surg. 1996;223(6):737–44; discussion
B. Vertebral bodies become visible above 44–6
the diaphragm (Spine sign) 3. Melniker LA, Leibner E, McKenney MG, Lopez P,
Briggs WM, Mancuso CA.  Randomized controlled
C. Collapsed lung dances in the fluid (Whale clinical trial of point-of-care, limited ultrasonogra-
tail or Jellyfish sign) phy for trauma in the emergency department: the first
D. All of the above sonography outcomes assessment program trial. Ann
19. A posterior pericardial effusion in the PLAX Emerg Med. 2006;48(3):227–35.
4. Rozycki GS, Ballard RB, Feliciano DV, Schmidt JA,
view is located: Pennington SD. Surgeon-performed ultrasound for the
A. Anterior to the descending aorta assessment of truncal injuries: lessons learned from
B. Posterior to the descending aorta 1540 patients. Ann Surg. 1998;228(4):557–67.
C. Anterior to the IVC 5. Kirkpatrick AW, Sirois M, Laupland KB, Liu D,
Rowan K, Ball CG, et al. Hand-held thoracic sonog-
D. Posterior to the IVC raphy for detecting post-traumatic pneumothoraces:
20. Your colleague performed a FAST scan in a the extended focused assessment with sonography for
stable trauma patient that was negative, after trauma (EFAST). J Trauma. 2004;57(2):288–95.
which the patient went on to have a CT scan 6. Quinn AC, Sinert R. What is the utility of the focused
assessment with sonography in trauma (FAST) exam
that showed small lacerations of the right in penetrating torso trauma? Injury. 2011;42(5):482–7.
kidney and right lobe of the liver with mini- 7. Griffin XL, Pullinger R.  Are diagnostic peritoneal
mal intra-abdominal fluid. Your colleague lavage or focused abdominal sonography for trauma
feels upset that they recorded a negative safe screening investigations for hemodynamically
stable patients after blunt abdominal trauma? A review
FAST exam. What advice will you give your of the literature. J Trauma. 2007;62(3):779–84.
colleague? 8. ATLS.  Advanced Trauma Life Support. Advanced
A. Advise them that they need more practice Trauma Life Support® Student Course Manual. 10 ed:
in FAST scans as their FAST report was Library of Congress Control Number: 2017907997;
2018.
incorrect 9. Root HD, Hauser CW, McKinley CR, Lafave JW,
B. Advise them that the FAST exam has low Mendiola RP Jr. Diagnostic peritoneal lavage. Surgery.
sensitivity in detecting small amounts of 1965;57:633–7.
fluid and the fluid may have started col- 10. Gupta M, Schriger DL, Hiatt JR, Cryer HG, Tillou
A, Hoffman JR, et  al. Selective use of computed
lecting after the FAST was done, so the tomography compared with routine whole body imag-
FAST they did was valid for that time ing in patients with blunt trauma. Ann Emerg Med.
C. Advise them to get a second opinion on 2011;58(5):407–16.e15.
the CT report as it cannot be correct 11. Salim A, Sangthong B, Martin M, Brown C, Plurad
D, Demetriades D.  Whole body imaging in blunt
D. Advise them to arrange for an explor- multisystem trauma patients without obvious signs
atory laparotomy of injury: results of a prospective study. Arch Surg.
2006;141(5):468–75.
12. Goodman A, Perera P, Mailhot T, Mandavia D.  The
role of bedside ultrasound in the diagnosis of pericar-
References dial effusion and cardiac tamponade. J Emerg Trauma
Shock. 2012;5(1):72–5.
1. Rozycki GS, Ochsner MG, Schmidt JA, Frankel HL, 13. Nishijima DK, Simel DL, Wisner DH, Holmes
Davis TP, Wang D, et al. A prospective study of sur- JF. Does this adult patient have a blunt intra-abdom-
geon-performed ultrasound as the primary adjuvant inal injury? JAMA. 2012;307(14):1517–27.
modality for injured patient assessment. J Trauma.
1995;39(3):492–8; discussion 8–500
Miscellaneous POCUS: Gastric
Ultrasound, Urinary Bladder 8
Ultrasound, Ocular Ultrasound,
Obstetric POCUS

Chetan Mehra and Amit Dikshit

8.1 Introduction 8.2 Gastric Ultrasound

The presence of the ultrasound machine in the 8.2.1 Introduction


operation theater has led to chance usage of the
modality and incidental development of the fields • Preoperative fasting guidelines help limit the
such as gastric, urinary bladder, and ophthalmic aspiration risk in patients undergoing elective
ultrasound, which is probably performed only by surgical procedures. However, fasting inter-
the anesthesiologists. These techniques have vals are not applicable or reliable in urgent or
unique indications and they provide valuable emergency surgeries or for patients with
clinical insight at the point of care in a timely delayed gastric emptying [1].
manner. • Gastric ultrasound is a point-of-care tool for
In this chapter, we will discuss gastric ultra- the assessment of antral gastric volume,
sound, ultrasonic assessment of the urinary blad- which is an indirect correlate for risk of aspi-
der, ocular ultrasound, and point-of-care ration of gastric contents. It can help to
ultrasound of the gravid uterus. For the benefit of determine the nature and volume of gastric
the readers, there is a Video 8.1 linked with this contents (empty, clear fluid, thick fluid/
chapter that will guide the readers through the solid) [2].
new and interesting modalities of miscellaneous
POCUS.
8.2.2 Indications [3]

Potential advantage of gastric ultrasound can be


seen in the following patient population:

Supplementary Information The online version con- • Suspected delay in gastric emptying:
tains supplementary material available at [https://doi. –– Pregnancy.
org/10.1007/978-­981-­16-­7687-­1_8].
• Suspected gastro-paresis.
–– Diabetes Mellitus.
C. Mehra (*) –– Chronic hepatic/renal dysfunction.
Department of Anaesthesiology, Critical Care and –– Neuromuscular disorders.
Pain, Indraprastha Apollo Hospital, New Delhi, India
• Lack of adherence to fasting instructions:
A. Dikshit –– Emergency/urgent procedure.
Ruby Hall Clinic and Hospital, Pune, India

© The Author(s), under exclusive license to Springer Nature Singapore Pte Ltd. 2022 171
A. Chakraborty, B. Ashokka (eds.), A Practical Guide to Point of Care Ultrasound (POCUS),
https://doi.org/10.1007/978-981-16-7687-1_8
172 C. Mehra and A. Dikshit

–– Unreliable fasting history, because of. 8.2.5 Applied Anatomy


Language barrier.
Altered sensorium. 8.2.5.1 Gastric Antrum
Cognitive dysfunction. –– Gastric antrum, as a part of stomach, is easily
amenable to sonographic examination. It accu-
rately reflects the content of the entire stomach.
8.2.3 Limitations –– It appears as a hollow viscus with a prominent
multilayered wall.
Gastric ultrasound findings may be inaccurate in –– Antrum is sandwiched between liver (anteri-
subjects with abnormal underlying gastric anat- orly) and pancreas (posteriorly).
omy (e.g., previous gastric resection or bypass, –– Landmarks for identification of gastric antrum
gastric band in situ, previous fundoplication, are [1] left lobe of liver, [2] Pancreas, [3]
large hiatus hernia) [4]. Aorta, [4] Inferior vena cava, and [5] Superior
mesenteric artery and vein.
–– It is usually located superficially at a depth of
8.2.4 Scanning Technique 3–4 cm.

• Patient position: 8.2.5.2 Gastric Wall


–– Start scanning with the patient in supine –– It appears as a characteristic five-layered wall and
position, followed by examination with sets the organ apart from other hollow viscus.
patient in right lateral decubitus position.
Final assessment of gastric antral volume/
contents should be based on examination in 8.2.6 Ultrasound Image Correlation
both positions [2]. (Table 8.1)
• Transducer selection:
–– Curved array/low-frequency (2–5 MHz). Ultrasound appearance of antrum can be graded
–– Select abdominal settings for scan. based on antral shape, antral wall appearance,
• Probe position: and antral content, as follows [2]:
–– Start scanning epigastrium in mid-sagittal
plane, just caudal to xiphisternum. 8.2.6.1 Empty Stomach
• Sweep the transducer both ways, from left to • Antrum has no appreciable content in both
right subcostal margins (Fig. 8.1). supine and RLD position (Grade 0 antrum).
• Identification of relevant structures is based • Bull’s eye or Target Pattern: It appears flat and
on characteristic ultrasound images as dis- collapsed or with a round-to-ovoid shape.
cussed ahead (Fig. 8.2).

Fig. 8.1  Probe placement in supine and right lateral decubitus position
8  Miscellaneous POCUS: Gastric Ultrasound, Urinary Bladder Ultrasound, Ocular Ultrasound, Obstetric… 173

Fig. 8.2  Gastric Antrum ultrasound image with sche- mesenteric branches, orange—liver, yellow—pancreatic
matic representation of anatomic structures (Black—ver- body, green—flat empty antrum). *left side of image is
tebral column, dark red—aorta with celiac and superior cranial and right side of image is caudal end

Table 8.1  Appearance of Antrum and antral wall with contents


Grade Antral shape Antral wall Content
Empty Flat and collapsed or Thick with prominent None (grade 0) OR small amount of
round (Bull’s eye) muscularis propriae hypoechoic content (grade 1)
Solid Round/distended Thin Heterogenous, particulate, mixed with air
Clear fluid Round/distended Thin Hypoechoic
Milk/ Round/distended Thin Hyperechoic
suspensions

8.2.6.2 Solid Early Stage • Antrum will appear larger in RLD position
• Antrum appears distended with thin walls. compared to supine position (Fig. 8.4).
• The content is of high or mixed echogenicity.
• Specific pattern: “Frosted-glass” pattern (usually 8.2.6.5 Fluid with Air Bubbles
shortly after a solid meal). It is due to a mix of air • Starry night pattern (multiple air bubbles on a
and solid along the anterior wall, blurring the hypoechoic background) usually seen shortly
posterior wall and deeper structures (Fig. 8.3). after ingestion of clear fluids or effervescent
drinks.
8.2.6.3 Solid Late Stage
• Heterogeneous, particulate content (usually
after 1–2 h following a solid meal). 8.2.7 Gastric Volume Assessment
• Homogeneous hyperechoic content: charac- (Clear Fluids)
teristic of dairy products or particulate fluids.
• Volume assessment can differentiate a low
8.2.6.4 Clear Fluid (normal) quantity of baseline gastric secre-
• Antrum appears round and distended with thin tions from a higher (non-fasting) volume.
walls. • Cross-sectional area of the antrum (CSA) has
• The content appears anechoic or hypoechoic. a linear correlation with the gastric volume
• The size of antrum is proportional to gastric (Figs.  8.3 and 8.4: marked by yellow dotted
volume. lines).
174 C. Mehra and A. Dikshit

Fig. 8.3  Gastric antrum image acquired 150 min after meal. (a) Supine and (b) Right lateral decubitus position

a b

Fig. 8.4  Gastric antrum image acquired 15  min after Image (c) Image acquired 2  h after water intake
drinking clear water. Image (a) Patient in supine position; (Measurements: A—area, C—circumference, D1 and
Image (b) Patient in right lateral decubitus position; D2—diameters in two perpendicular planes)
8  Miscellaneous POCUS: Gastric Ultrasound, Urinary Bladder Ultrasound, Ocular Ultrasound, Obstetric… 175

Fig. 8.4 (continued)

Table 8.2  Antral grading system (Grades 0–2)


Grade Antral presentation Volume implications Aspiration risk
0 Empty in both supine and Minimal Low risk
RLD position
1 Empty in supine, clear fluid ≤1.5 ml/kg, compatible with Low risk
visible in RLD position baseline gastric secretions
2 Clear fluid visible in both >1.5 ml/kg, likely in excess of High risk
positions baseline gastric secretions

• Measurement of CSA: [4] 8.2.8 Qualitative Versus


–– Identify antrum at the level of aorta in RLD Quantitative Assessment
position.
–– Obtain a still image of antrum at rest • Qualitative assessment determines the appear-
(between peristaltic contractions). ance of the antrum, wall and its contents.
–– Use the free-tracing tool of ultrasound • Quantitative assessment determines aspiration
machine to measure the CSA including the risk derived from calculated gastric volume
full thickness of gastric wall (serosa to using a mathematical model.
serosa).
–– Volume (ml)  =  27.0  +  14.6  ×  CSA (RLD Clinical data suggest that a volume of up to
position) – 1.28 × age 1.5 mL/kg (100 mL for the average adult) is nor-
mal in fasted patients [1].
8.2.7.1 Antral Grading System
(Grades 0–2) [5] (Table 8.2)
176 C. Mehra and A. Dikshit

Fig. 8.5  Urinary bladder with foley’s catheter in situ (Longitudinal and transverse probe orientation) left image depicts
longitudinall scan and right image depicts transverse scan

8.3 Point-of-Care Ultrasound


Examination of Urinary
Bladder

8.3.1 Introduction

Ultrasound examination of the urinary bladder is


an easy, quick, and accurate way to directly visu-
alize and assess the bladder at the bedside.

8.3.2 Indications

1. Assess bladder volume (pre and post-void


residual) [6, 7].
2. Confirmation of proper urinary foley catheter
placement/troubleshooting malfunctioning
foley catheter (Fig. 8.5) [8].
Fig. 8.6  Bladder Ultrasound showing Ureteric jet
3. Assess bladder pathologies such as bladder
stones, ureteral jets (Fig.  8.6), and bladder
masses and other causes of hematuria.
4. It is helpful for quick assessment in patients
with concern for postrenal acute kidney injury,
dilated collecting system, and patients with
frequent urinary tract infections.
8  Miscellaneous POCUS: Gastric Ultrasound, Urinary Bladder Ultrasound, Ocular Ultrasound, Obstetric… 177

8.3.3 Following Topics Will 2. The probe marker should point cranially in
Be Covered in this Description order to obtain a sagittal view of the
bladder.
Use Point-of-Care Bladder ultrasound to: 3. Sweep the probe on either side to examine the
lateral borders of the bladder.
1. Perform Bladder Ultrasound in a Step-by-­ 4 . As the bladder is directly posterior to the
Step Fashion. pubic symphysis, the ultrasound beam should
2. Measure Bladder Volume. be directed towards the pelvis, so as to iden-
3. Recognize Common Bladder Ultrasound tify the bladder underneath the bone.
Pathology. 5. Transverse view: Rotate the probe 90° to
obtain a transverse view and sweep it superior
to inferior, so as to image the bladder com-
8.3.4 Patient Position pletely (Fig. 8.7).
6. The shape of the bladder varies depending on
Patient placed in supine position with suprapubic anatomical differences and filled-up status of
area exposed. Knees can be flexed to soften the bladder. Its shape can be thought of as spheri-
anterior abdominal wall. cal, a triangular prism, cylinder (ellipsoid), or
cuboid.
7. Surrounding structures including the pubic
8.3.5 Ultrasound Probe Selection
bone, abdominal cavity, rectum (or bowel
gas), uterus (in females), and prostate (in
Curvilinear Ultrasound Probe with abdominal or
males) are important ultrasound landmarks
renal preset: With its wider footprint, it allows
for the identification and assessment of blad-
full visualization of the bladder.
der volume.

8.3.6 Technique [7]


8.3.7 Calculation of Bladder Volume
1. Longitudinal view: Place the probe longitudi-
nally in mid-line above the pubic symphysis Bladder volume can be estimated using the for-
(Fig. 8.7). mula: Height × Width × Length × 0.7

a b

Fig. 8.7  Bladder ultrasound. (a) Transverse probe orientation and (b) Transverse probe orientation
178 C. Mehra and A. Dikshit

a b

Fig. 8.8  Bladder ultrasound. (a) Transverse and (b) Longitudinal Views with transverse, longitudinal, and vertical
lengths measurements

1. The factor (0.7) in the formula refers to the 8.4 Point-of-Care Ultrasound
correction coefficient, that varies with the pro- Examination of Eye
posed shape of bladder (unknown–0.72, trian-
gular prism–0.66, ellipsoid–0.81, 8.4.1 Optic Nerve Sheath Diameter
cuboid–0.89, spherical–0.52). (ONSD)
2. Width and Depth of the bladder are measured
in the transverse view (Fig. 8.8). 8.4.1.1 Introduction
3. Height (vertical dimension) is measured in the 1. An elevation of intracranial pressure (ICP)
longitudinal view. secondary to cerebral edema is a major con-
4. Both pre- and post-void measurements should tributor to morbidity and mortality in trauma,
be taken to diagnose urinary retention. neurosurgical patients, and acute liver failure
Point-of-care ultrasound can detect foley’s (ALF) [9].
catheter, bladder mass, bladder calcu- 2. Geriatric surgical populations with multi-
lus, enlarged prostate, or hematoma. ple comorbidities undergo robotic surgeries
Kinking of foley’s catheter can cause with steep Trendelenburg position.
falsely low urine output which can be Impaired cerebral autoregulation may be
easily ruled out using bedside bladder compounded by alterations in cerebral
ultrasound. homeostasis in this population, due to pro-
Use of color Doppler interrogation at the longed duration of surgery in steep head
base of the urinary bladder can show low positioning.
ureteric jets of urine. In the author’s 3. ONSD measurement can be a valuable surro-
opinion, this technique can also be uti- gate for invasive monitoring for assessment of
lized in the differential diagnosis of an ICP in such patients who present with coagu-
oliguric patient. lopathy and high ICP [9, 10].
This technique is beneficial in the postop- 4. Perioperative ICP monitoring may be benefi-
erative assessment of renal transplant cial for guiding the administration of targeted
recipient patients. therapy (i.e., mannitol, hypertonic saline,
8  Miscellaneous POCUS: Gastric Ultrasound, Urinary Bladder Ultrasound, Ocular Ultrasound, Obstetric… 179

hyperventilation, and reverse Trendelenburg 8.4.1.4 Limitations


positioning) to prevent brain herniation. 1. Acute changes in blood pressures (>10% of

5. Ultrasonographic ONSD assessment has initial values) at the time of assessment can
proven to be useful for monitoring high ICP in theoretically lead to blood pressure-induced
patients with brain injury, idiopathic intracra- changes in ONSD.
nial hypertension, and spontaneous intracra- 2. Cannot be performed in ophthalmic injuries
nial hemorrhage [9, 10]. such as ruptured globe.
3. Arguable predictive value in patients with
It has proven to be more than 80% sensitive glaucoma.
and more than 80% specific in detecting raised
ICP by many authors. 8.4.1.5 Procedure for Ocular
Papilledema can be detected as an optic disc Ultrasound
bulge into the retina (Fig. 8.11b). 1. Probe selection.
(a) A high resolution 7.5–10 MHz or higher
8.4.1.2 Anatomic Correlation linear array ultrasound transducer, held
Optic nerve sheath is contiguous with dura mater with a pencil grip.
and its contents are contiguous with the sub- 2. Probe—eye interface preparation:
arachnoid space. Thus, raised ICP leads to an (a) Cover eyelids with a sterile transparent
increase in the optic nerve sheath diameter and dressing (Tegaderm) to prevent contami-
hence can be used as an indirect correlate for ICP nation or trauma during the scan proce-
assessment. dure (Fig. 8.9a, b).
It is measured at a point 3.0  mm behind the (b) Apply adequate amount of water-based
globe which is believed to be an area of greatest gel (ultrasound gel) onto the interface of
ultrasound contrast. A measurement of up to tegaderm and ultrasound probe.
5.0 mm is considered a normal ONSD. Elevated (c) Apply very gentle pressure over the eye,
ICP should be suspected with an ONSD of more while keeping your hand on the patient’s
than 5.0 mm. face so as to avoid discomfort/trauma to
In patients with suspected raised ICP present- the eye.
ing with symptoms like agitation, sedation, con- 3. Ultrasound image:
fusion, blurring of vision, conjunctival or (a) Eye is scanned to assess normal anatomy
periorbital edema, or delayed awakening from and landmarks.
anesthesia, ONSD measurement can guide fur- (b) Following structures are visualized from
ther management [10]. anterior to posterior direction to gain a
For perioperative use, ONSD should be mea- proper orientation, before proceeding
sured before patient positioning for surgery, as a with any measurements (Fig.  8.10)
baseline measurement. (Table 8.3) [11].
4. Target depth:
8.4.1.3 Advantages (a) A point 3 mm posterior to the optic disc is
1. Noninvasive. considered the target point. Optic nerve is
2. Can be used in patients with deranged coagu- considered to be most distensible and
lation profile. hence representative of rise in ICP at this
3. Easy to master, quick, and reproducible tech- particular point (Fig. 8.11a, b).
nique and can be done in any patient (b) Optic nerve is seen as anechoic (black)
position. linear structure bounded by hyperechoic
4. It does not involve extra cost as most of the (bright) lines, i.e., optic nerve sheath at
operating rooms are equipped with point-of-­ this level (behind the optic disc).
care ultrasound machines. 5. Scan protocol: Ultrasound examination is car-
ried out at two axes perpendicular to each other.
180 C. Mehra and A. Dikshit

Fig. 8.9 (a) Preparation of eye (tegaderm and jelly application) and probe placement for Ocular Ultrasound. (b)
Tegaderm application over eye and position of probe

Fig. 8.10 Ultrasound
appearance of Ocular
structures
CORNEA
IRIS

VITREOUS CHAMBER

OPTIC DISC

OPTIC NERVE
OPTIC NERVE SHEATH
8  Miscellaneous POCUS: Gastric Ultrasound, Urinary Bladder Ultrasound, Ocular Ultrasound, Obstetric… 181

Table 8.3  Ultrasound correlation of Ocular structures


Structure Echogenicity Relative USG anatomy (normal eye)
Cornea Hypoechoic Thin hypoechoic layer parallel to the
eyelid
Anterior chamber Anechoic Anechoic area between cornea, iris, and
anterior reflection of lens capsule
Lens capsule Hyperechoic Curvilinear anterior and posterior lines
enclosing anechoic lens
Iris and ciliary body Hyperechoic Linear structures extending from the
peripheral globe towards lens
Posterior chamber Anechoic Hollow spherical structure behind lens
capsule
Retina/choroid/sclera Hyperechoic Spherical covering outlining the eyeball,
(covering layers of cannot be differentiated from each other
eyeball)
Retro-orbital region Heterogeneously hyperechoic Posterior to globe
(EOM/orbit)
Optic nerve with its Anechoic linear structure enclosed in Linear structure radiating away from the
covering sheath hyperechoic sheath (optic nerve sheath) globe

a b

Papilledema

Fig. 8.11 (a) Measurement of Optic Nerve Sheath Diameter (0.27 cm in the above figure). (b) Papilledema bulging
into retina with grossly dilated optic nerve sheath diameter

(c) Horizontal Scan: Probe is placed in a line 8.4.1.6 Conclusion


joining both canthi, i.e., horizontal axis. 1. Technique of ONSD measurement by ultra-
(d) Vertical Scan: Probe is placed in a plane sound of the eye is an established screening
perpendicular to first plane, i.e., vertical tool for predicting raised ICP in trauma
axis. patients.
(e) Measurements are taken in both axes and 2. The cut-off values of ONSD for predicting a
a mean of ONSD in both transverse and raised ICP vary from 0.48 to 0.59 cm in differ-
vertical axes is taken as final reading, rep- ent studies.
resentative of ONSD.
182 C. Mehra and A. Dikshit

3. A serial change in ONSD is more important 2. Coronal view:


than an absolute single value, though this has (a) Patient is asked to look with his gaze
to be validated. down. Probe is placed over the lower part
of the eyeball with ultrasound beam point-
ing cranially. Iris with pupil can be identi-
8.4.2 Consensual Light Reflex [12] fied in coronal orientation.

8.4.2.1 Introduction 8.4.2.3 Assessment of Consensual


Ultrasound examination of the eye can be used to Light Reflex
assess consensual light reflex in patients who (a) Light is directed to the opposite eye and con-
cannot open an ipsilateral eye for pupillary exam- sensual response in the form of pupillary
ination because of reasons like trauma, swollen constriction is observed with ultrasound in
eyelid, perifacial edema, etc. the eye to be examined. In both views (trans-
Probe selection and preparation remain the verse and coronal), pupils will be seen to
same as for ONSD measurement. contract. Pupillary activity along with maxi-
Patients can be in a sitting or upright position mum and minimum pupillary diameters can
for this procedure (Fig. 8.12). be assessed (Fig. 8.13).

8.4.2.2 Ultrasound Views 8.4.2.4 Central Retinal Artery


Iris can be visualized in either of the two ways: Occlusion (CRAO) [13]
Ultrasound examination of the eye can help to
1. Transverse/sagittal view: diagnose Central Retinal Artery Occlusion in
(a) Probe is placed transverse or para-sagittal cases of sudden blurring or loss of vision, e.g.,
over the eyeball, with ultrasound beam after major spine surgery, long-duration prone
directed towards the back of the eye. position ventilation in ARDS patients or intu-
Intraocular anatomy can be identified, as bated patients of COVID-19, etc.
mentioned in the table. Iris can be seen as It is observed as a hyperechoic spot (ditzel) in
a linear structure extending from the the center of the Optic nerve (Retro-bulbar Spot
peripheral globe towards the lens. It can sign). This hyperechogenic spot represents
be observed as a smiling face, with eyes thrombosis of the Central Retinal Artery. Retinal
representing iris and smiling lips repre- blood flow occlusion can be confirmed with the
senting lenses. use of color Doppler, accuracy of which itself
depends upon the angle of insonation of ultra-
sound beam.
A false positive assessment can be made in
case of Optic Nerve Drusen, which is a calcific
material deposit in the head of Optic nerve/retina
and also presents clinically as blurring or loss of
vision.
Ocular ultrasound provides high specificity
but low sensitivity to confirm diagnosis of CRAO.

8.4.2.5 Vitreous Versus Retinal


Detachment [14]
Ultrasound examination of the eye can help to
diagnose and differentiate between retinal and
Fig. 8.12  Method to assess consensual light reflex (probe vitreous detachment, in cases of emergency
placed over the eye to be examined and light shown over scenarios.
the opposite eye)
8  Miscellaneous POCUS: Gastric Ultrasound, Urinary Bladder Ultrasound, Ocular Ultrasound, Obstetric… 183

Fig. 8.13  Transverse and Coronal views of the iris (pupil)

Retinal detachment: POCUS may also aid in the diagnosis of


underlying obstetric pathologies, such as fetal
Appears as a bright, continuous, smooth and demise, premature rupture of membranes
somewhat folded membrane within the vitre- (PROM), and antenatal hemorrhage from abnor-
ous, and freely moving on real-time imaging mal placentation, such as placenta previa or
and is seen coming out of the optic nerve. accreta. Additionally, POCUS can be used in the
assessment of various other conditions that
Vitreous detachment: impact pregnant women, including abdominal
pain and maternal trauma.
Posterior vitreous detachment is seen as a freely It is especially useful in health care centers
mobile hyperechoic membrane that swirls away located in remote locations where radiologists
from the optic disc with movement of eye. are not easily available. Complete obstetric ultra-
The mobile membrane is seen to cross the mid- sound is the domain of radiologists.
line, with the optic disc representing the mid-
line, i.e., the line is seen crossing over the
optic nerve and not attached to it. 8.5.2 Anatomical Correlation

Use of ocular ultrasound can also detect globe The enlarging uterus will displace the normal
rupture and intraocular foreign body which is position of surrounding structures and this can
beyond the scope of this book. affect the ultrasound appearance of intra-­
abdominal pathology (Fig. 8.14).

8.5 Point-of-Care Obstetric


Ultrasound 8.5.3 Scope of a POCUS in Second-
or Third-Trimester Ultrasound
8.5.1 Introduction Examination

Point-of-care ultrasound (POCUS) is used to pro- Obstetric POCUS can aid in assessing the follow-
vide an accurate assessment of gestational age, ing conditions:
fetal number, presence or absence of cardiac
activity, and placental location. 1. Fetal lie and presentation.
184 C. Mehra and A. Dikshit

36 wk Xiphisternum
40 wk
32 wk
28 wk
24 wk
Umbilicus
20 wk

16 wk
Public symphysis
(Uterus becomes an
abdominal organ
after 12 weeks)

Fig. 8.14 Gravid uterus abdominal assessment on


palpation Fig. 8.15  Sagittal scan for obstetric POCUS

2. Fetal cardiac activity.


3. Fetal number (singleton, twin, triplet, or
higher order multiple gestation).
4. Amniotic fluid volume.
5. Placental localization and assessment.
6. Fetal biometry.

8.5.4 Image Acquisition

Two techniques are generally used for


examination
Fig. 8.16  Transverse scan for obstetric POCUS
• Transabdominal Ultrasound examination.
• Transvaginal Ultrasound examination. left lateral tilt and relieve aortocaval
compression.
• Probe selection:
8.5.5 Scanning Procedure A low-frequency, curvilinear ultrasound
for Transabdominal probe (2–5 Hz) is preferred with a wide field
Examination Technique of view and adequate penetration for pelvic
(Figs. 8.15 and 8.16) imaging. An obstetric exam preset should be
selected to perform some key measurements
• Patient positioning for ultrasound and calculations.
examination. • Scan protocol:
The patient should be supine with the lower A transverse scan is performed in midline
abdomen exposed. A full bladder is preferred examining in a caudocranial direction. This is
because it provides an acoustic window to followed by a sagittal scan extending from either
visualize the uterus and adnexa. Patient’s side and also in the craniocaudal direction.
upper body and head are slightly inclined and An optimal pressure should be maintained
supported by a soft pillow or cushion. In the while performing this transabdominal ultra-
latter part of pregnancy, a pillow may be sound scan. An excessive probe pressure over
placed under the patient’s right side to provide a gravid uterus is not desirable.
8  Miscellaneous POCUS: Gastric Ultrasound, Urinary Bladder Ultrasound, Ocular Ultrasound, Obstetric… 185

For a transverse view of the uterus, the


transducer is placed immediately above the
pubic symphysis with the marker pointing to
the patient’s right side. A sagittal view is
obtained in the same position by rotating the
transducer 90 degrees clockwise with the
transducer marker pointing cephalad.
• Ultrasound correlation of surrounding
anatomy:
In a transverse orientation, the bladder is
seen in the near field with the uterus immedi-
ately posterior to it. Within the uterus, the
myometrium surrounds the endometrium and Fig. 8.18  Transverse scan suggestive of placenta adher-
is less echogenic by comparison. ent to the anterior uterine wall in the lower segment

8.5.6 Applications

• Fetal cardiac activity:


Quick bedside obstetric ultrasound can
detect fetal cardiac activity, fetal heart rate,
and thereby fetal distress. This technique is
also useful for decision-making in suspected
patients of intrauterine fetal death (IUFD).
Doppler ultrasound usage is desirable for Fig. 8.19  Transverse scan with fetal spine perpendicular
to maternal spine suggestive of transverse lie
picking up fetal heart rate.
• Placental localization, fetal lie, and fetal
presentation:
Point-of-care approach can help in antici-
pating difficult labor as with obstetric ultra-
sound we can also detect placental localization
(placenta previa, placenta accreta, etc.)
(Figs. 8.17 and 8.18) fetal lie (Fig. 8.19), fetal

Fig. 8.20  Footling presentation

number, and fetal presentation with its present-


ing part (Fig. 8.20), e.g., transverse lie occurs
when the fetal spine is positioned perpendicu-
lar to the maternal spine (Fig. 8.21). Antenatal
detection of placenta previa decreases the fetal
and maternal mortality [15].
• Multiple cords around fetus’s neck:
Color Doppler interrogation can detect
number of cords around fetus’s neck which
Fig. 8.17  Placenta Previa with anticipated difficult nor- can be very useful information to the attend-
mal labor ing pediatrician (Fig. 8.22).
186 C. Mehra and A. Dikshit

8.5.7 Limitations

Amniotic fluid volume assessment and fetal


biometry including head circumference and
femur length is beyond the scope of this chapter.

Questions

1. Which type of resolution is primarily deter-


mined by the width of the ultrasound beam?
A. Axial
B. Lateral
Fig. 8.21  Transverse lie

Fig. 8.22  Multiple cords around fetus’s neck using color Doppler
8  Miscellaneous POCUS: Gastric Ultrasound, Urinary Bladder Ultrasound, Ocular Ultrasound, Obstetric… 187

C. Elevational 5. When point-of-care ultrasound detects peri-


D. Temporal toneal free fluid in the pelvis, it is always a
2. Which transducer type is most commonly pathologic finding in both men and women.
used for ocular ultrasound? A. True
A. Curvilinear B. False
B. Intracavitary 6. Which of the following artifacts is usually
C. Linear seen posterior or deep to a fluid-filled
D. Phased array bladder?
3. How will you adjust the ultrasound settings A. Acoustic shadowing
to improve the quality of transverse scan of B. Edge artifact
urinary bladder, as shown in Fig. 8.23? C. Mirror artifact
A. Decrease depth D. Acoustic enhancement
B. Increase depth 7. Which ultrasound imaging mode is used to
C. Increase gain best visualize ureteral jets?
D. Decrease gain A. A. Two-dimensional grayscale
4. Which of the following patient characteris- B. M-mode
tics facilitates ultrasound imaging of the C. Spectral Doppler
abdominopelvic cavity? D. Power Doppler
A. Asplenia 8. Which of the following formulas is most
B. Subcutaneous emphysema often used to calculate bladder volume?
C. Urine-filled bladder A. 0.25 × width × length × height
D. Gas-filled loops of bowel B. 0.35 × width × length × height
C. 0.50 × width × length × height
D. 0.75 × width × length × height
9. Which dimension best describes prostatic
hypertrophy by ultrasound?
A. Transverse diameter > 5 cm
B. Longitudinal diameter > 5 cm
C. Transverse diameter > 2.5 cm
D. Longitudinal diameter > 2.5 cm
10. A 19-year-old man presents with acute uni-
lateral eye pain and visual impairment after
sustaining direct trauma to the eye. Which of
the following complications of traumatic eye
injury would NOT be detected by ocular
ultrasound?
A. Globe rupture
B. Retinal detachment
C. Corneal abrasion
D. Hyphema
E. Extraocular motor dysfunction
Fig. 8.23  Transverse scan of urinary bladder
188 C. Mehra and A. Dikshit

11. Which of the following is the correct loca- 2. Perlas A, Chan VWS, Lupu CM, Mitsakakis N,
tion to measure the optic nerve sheath Hanbidge A. Ultrasound assessment of gastric content
and volume. Anesthesiology. 2009;111:82–9.
diameter? 3. El-Boghdadly K, Wojcikiewicz T, Perlas
A. At the optic disc where the optic nerve A. Perioperative point-of-care gastric ultrasound. BJA
sheath engages the retina Education. 2019;19(7):219e226.
B. 1 mm posterior to where the optic nerve 4. Sharma G, Jacob R, Mahankali S, Ravindra
MN. Preoperative assessment of gastric contents and
sheath engages the retina volume using bedside ultrasound in adult patients: a
C. 3 mm posterior to where the optic nerve prospective, observational, correlation study. Indian J
sheath engages the retina Anaesth. 2018;62:753–8.
D. 5 mm posterior to where the optic nerve 5. Perlas A, Van de Putte P, Van Houwe P, Chan
VW. I-AIM framework for point-of-care gastric ultra-
sheath engages the retina sound. Br J Anaesth. 2016 Jan;116(1):7–11.
12. Which of the following statements regarding 6. Matsumoto M, Tsutaoka T, Yabunaka K, Handa M,
cleaning and disinfection of ultrasound trans- Yoshida M, Nakagami G, et  al. Development and
ducers is TRUE? evaluation of automated ultrasonographic detection of
bladder diameter for estimation of bladder urine vol-
A. If a disposable sterile probe cover is used ume. PLoS One. 2019;14(9):e0219916.
over an endocavitary transducer during 7. Dicuio M, Pomara G, Fabris FM, Ales V, Dahlstrand C,
contact with mucous membranes, high-­ Morelli G. Measurements of urinary bladder volume:
level disinfection is not necessary after comparison of five ultrasound calculation methods in
volunteers. Arch Ital Urol Androl. 2005;77(1):60–2.
disposal of the probe cover. 8. Garagliano J, Madhok J.  POCUS for visulaliza-
B. Alcohol-based disinfecting wipes are tion and facilitation of urinary catheter placement.
recommended after scanning patients POCUS J. 2020;5(2):35.
with Clostridium difficile infection. 9. Dubourg J, Javouhey E, Geeraerts T, Messerer M,
Kassai B.  Ultrasonography of optic nerve sheath
C. Ultrasound transducers should be auto- diameter for detection of raised intracranial pressure:
claved after use on patients with a systematic review and metaanalysis. Intensive Care
multidrug-­resistant bacteria, especially Med. 2011;37:1059–68.
Pseudomonas aeruginosa. 10. Lin J-J, Chen AE, Lin EE, Hsia S-H, Chiang M-C,
Lin K-L.  Point-of-care ultrasound of optic nerve
D. Transducers that do not make contact sheath diameter to detect intracranial pressure in neu-
with mucous membranes can be cleaned rocritically ill children - a narrative review. Biomed J.
with nonabrasive soap, or low- or 2020;43(3):231–9.
intermediate-­level disinfectant wipes 11. Lorente-Ramos RM, Armán JA, Muñoz-Hernández
A, Gómez JMG, de la Torre SB.  US of the eye
after each use. made easy: a comprehensive how-to review with
13. Which vessels determine the correct location ophthalmoscopic correlation. Radiographics.
in visualizing the antrum of stomach when 2012;32:E175–200.
performing a gastric ultrasound 12. Sargsyan AE, Hamilton DR, Melton SL, Amponsah
D, Marshall NE, Dulchavsky SA.  Ultrasonic evalu-
examination? ation of pupillary light reflex. Crit Ultrasound J.
A. Superior mesenteric vessels 2009;1:53–7.
B. Abdominal aorta 13. Stoner-Duncan B, Morris SC.  Early identifica-
C. Inferior vena cava tion of central retinal artery occlusion using point-­
of-­
care ultrasound. Clin Pract Cases Emerg Med.
D. All of the above 2019;3(1):13–5.
14. Gottlieb M, Holladay D, Peksa GD.  Point-of-care
ocular ultrasound for the diagnosis of retinal detach-
ment: a systematic review and meta-analysis. Acad
Emerg Med. 2019;26(8):931–9.
References 15. American College of Obstetricians and Gynecologists:
ACOG committee opinion no. 560: medically indi-
1. Van de Putte P, Perlas A.  Ultrasound assess- cated late-preterm and early-term deliveries. Obstet
ment of gastric content and volume. Br J Anaesth. Gynecol. 2013;121(4):908–10.
2014;113(1):12–22.
POCUS: What does the Future
Hold? 9
Rachel Hui Xuan Chia and Balakrishnan Ashokka

9.1 Introduction 9.2 Artificial Intelligence

The invention of the stethoscope revolutionized Imagine a world where the expertise of a doctor
the practice of medicine in the 1800s. Fast for- could be supported by the power of computer-­
ward two centuries later, the era of “Point-of-­Care assisted decision-making. The combined efforts
Ultrasound” (POCUS) arrived, heralding a great of multiple experts in the field of computer sci-
milestone in the practice of medicine. POCUS is ence, medicine, robotics, and more have made
well-established in the field of emergency medi- this a reality. There is a plethora of reasons why
cine and trauma management. It is emerging as a traditional technologies in healthcare should
valuable tool in a wide range of applications embrace artificial intelligence (AI). In the field of
including procedural guidance for general and imaging technology, AI has an extraordinary
regional anesthesia, vascular access, decision- ability to not only make sense of images but also
making in intensive care, perioperative care, goal- reveal to the clinician, potentially hidden patterns
directed management, and therapeutics. Now, that would evade even the most highly trained
real-time ultrasound-guided assessment of inpa- human eye. In other words, AI is training the
tients in the wards is within reach for many clini- computer to see what the human eye cannot.
cians. Medical schools are also including
ultrasound interpretation skills in their curricu-
lum. In the next decade, it would be unsurprising 9.2.1 Deep Learning
to see POCUS become the new standard of care.
As advances in imaging technology develop fur- The demand for greater accuracy and more objec-
ther in parallel with the continued upgrading of tive imaging assessments have led to the incorpo-
ultrasound skills of clinicians, the future of ration of deep learning technology into the field
POCUS could bring about enhancements in visu- of ultrasonography. Deep learning is a machine
alization and accuracy, while minimizing the cog- learning technique where artificial neural net-
nitive load on the operator. The future of POCUS works, inspired by the human brain, are able to
can be simply summarized in a few words: “What make sense of data themselves. As one of the top
is advanced today, may be basic tomorrow.” ten breakthrough technologies of 2013 [1], deep
learning has solidified its position as the leading
R. H. X. Chia (*) · B. Ashokka machine learning tool in imaging analysis [2]. In
Department of Anaesthesia, National University 2020, the US Food and Drug Administration
Health System, Singapore, Singapore
approved the first cardiac ultrasound automated
e-mail: [email protected]

© The Author(s), under exclusive license to Springer Nature Singapore Pte Ltd. 2022 189
A. Chakraborty, B. Ashokka (eds.), A Practical Guide to Point of Care Ultrasound (POCUS),
https://doi.org/10.1007/978-981-16-7687-1_9
190 R. H. X. Chia and B. Ashokka

software [3], which uses deep learning technol- its capabilities in detecting lesions is warranted to
ogy to guide the operator to acquire images of prevent overdiagnosis and wastage of resources
diagnostic quality. It accomplishes this via real-­ and man-hours.
time prescriptive guidance of the user’s hand
motions and transducer positions to obtain the
desired image [4]. Once this image is captured, 9.3 Handheld Ultrasound (HHU)
the ejection fraction can be automatically calcu-
lated. With this novel technology, even novice In the field of POCUS-enabled healthcare
operators with no formal training in ultrasound advancements, the demand for “pocket sized”
will be able to obtain high-quality cardiac images, devices is increasing [11]. The hallmark of HHU
allowing rapid assessment and decision-making. is its accessibility, making it readily available to
Automated software has also been deployed in virtually every patient in the hospital that is lim-
other ultrasound applications such as to identify ited by the need for large bulky equipment and
and track endocardial structures to calculate indi- perpetually busy qualified ultrasound technolo-
ces of myocardial function in commercially gists. This complements and enhances any point-­
available platforms (HeartModelA.I., Philips of-­care ultrasound assessment. A variety of
Healthcare) [5, 6]. In the field of regional anes- handheld devices with different characteristics
thesia, identification of nerves can be considered are currently available in the market. Some prod-
one of the more challenging tasks for the novice ucts have single or multiple probes, while some
provider and image recognition technology could have the ability to stream images onto a mobile
potentially be incorporated into ultrasound-­ device. A few devices have incorporated AI sys-
guided regional nerve blocks. A machine learn- tems, aiding the novice operator to obtain high-­
ing model has been employed in ultrasound quality images and estimate certain measurements
images of the femoral nerve and brachial plexus of interest, such as ejection fraction in echocar-
[7, 8]. Besides nerve identification, AI-powered diography [12].
systems in ultrasound have also been deployed to Traditionally, the piezoelectric effect is the
increase the accuracy of spinal landmark identifi- cornerstone of ultrasound, which is common-
cation for complex anatomical presentations in place in the transducers of heavier cart-based
neuraxial anesthesia (uSINE™) [9]. Indeed, AI ultrasound machines. Few disadvantages of
allows clinicians to look forward to exciting piezoelectric technology include high cost of
future applications in clinical use, as well as in production and need for separate probes for the
regional anesthesia training. analysis of high and low frequencies [13]. The
Despite its unique capabilities in refining inherent limitations of piezoelectric technology
diagnostic accuracy and sensitivity in medical have inspired the development of a novel
imaging, users of AI-powered systems need to be ultrasound-­on-chip technology. Known as
cognizant of certain potential perils. A study on Capacitive Micromachined Ultrasound
screening mammograms showed that deep learn- Transducers (CMUT), piezoelectric crystals are
ing is no more sensitive than radiologists in replaced with a single silicon chip, which con-
detecting cancer, but was consistently less spe- verts electrical currents into ultrasound waves
cific, i.e., higher false-positive rates [10]. The [12]. The advantage of using an ultrasound chip
powerful ability of AI to detect minor changes in is its lower cost of production and ability to ana-
images may potentially be a double-edged sword, lyze a wider range of frequencies without the use
as increased sensitivity leads to overdiagnosis of of multiple probes. In addition, images can be
subclinical diseases with minimal impact on displayed on a mobile device, further enhancing
morbidity and mortality. Undoubtedly, portability (Butterfly iQ) [13].
AI-powered systems have promising clinical Image quality in HHU has been an area of
applications with the potential to improve public concern as some historical evidence suggests that
health. However, more research into finetuning of they are slightly inferior to traditional standalone
9  POCUS: What does the Future Hold? 191

devices [12, 14–16]. HHUs classically utilize 2D The authors found that the HHU accurately
and color Doppler imaging and lack the capabil- estimated the epidural depth, with a mean dif-
ity to visualize all abnormalities. Indeed, image ference of −0.61 cm [95% CI, −0.79 to −0.44]
quality in HHU may be inadequate in performing compared to needle depth, and furthermore
a comprehensive ultrasound examination; how- showed a high first pass rate of 87% with HHU-
ever, users need to understand that the use of guided identification of the lumbar interspace.
POCUS is to function as an adjunct to physical In fact, the Accuro HHU has also shown that it
examination in order to answer a focused clinical is comparable to a console ultrasound machine
question. In a systematic review comparing hand- in estimating epidural depth [25].
held ultrasound with high-end ultrasound sys-
tems, the authors showed that there was a good
overall correlation when the use of HHU is lim- 9.4 Contrast-Enhanced
ited to a distinct focused clinical question [16]. Ultrasound (CEUS)
This consensus is supported by several studies
that have also found HHU to be a valuable tool in The development of Contrast-Enhanced
the various settings, displaying images of suffi- Ultrasound (CEUS) could be a major paradigm
cient quality to detect ascites, hydronephrosis shift in the field of imaging. CEUS was first
and even screening for abdominal aorta aneu- introduced in the 1960s, where the injection of
rysms and measuring aortic diameters [17–22]. agitated saline bubbles produced a detectable sig-
In the field of obstetric anesthesia, the nal change on ultrasound [25]. Decades of
placement of epidural catheters for labor anal- research have culminated in the discovery of
gesia traditionally relies on the palpation of novel ultrasound contrast agents (UCA), which
body surface landmarks. First pass success are microbubbles of perfluorocarbon, nitrogen
rates with palpation alone have been reported gas, or sulfur hexafluoride stabilized in a phos-
to be as low as 20% [23]. In the first study of pholipid membrane [26]. The backscatter of
its kind, Seligman et  al. studied the accuracy ultrasound waves is enhanced when UCA micro-
of a HHU device (Accuro, Rivanna Medical, bubbles are injected into the vasculature, which
Charlottesville, VA) (Fig.  9.1) in estimating provides information on microvascular blood
depth to epidural space compared to using the flow and tissue perfusion. The analysis of micro-
conventional method of Tuohy epidural needle vasculature therefore confers a benefit when
depth (depth to loss of resistance to saline) [24]. compared to Doppler ultrasound [27, 28].

Fig. 9.1  The Accuro


spinal navigation
system. (Reproduced
with permission,
Accuro, Rivanna
Medical, Charlottesville,
VA)
192 R. H. X. Chia and B. Ashokka

With contrast agents no longer restricted to valves, and patients with acute respiratory dis-
computed tomography (CT), magnetic resonance tress syndrome [32].
imaging (MRI), and angiography, the application CEUS has been widely used in abdominal
of CEUS in various clinical settings could benefit and cardiac imaging for decades throughout Asia
patients significantly. Major advantages of CEUS and Europe. Yet, it was only in recent years, its
include its low cost, radiation sparing effects, use was approved by the US Food and Drug
bedside accessibility, lack of need for sedation, Administration for diagnostic liver imaging in
and lack of nephrotoxicity. In addition, its lack of adults and children [33]. The characterization of
iodine content, which is present in other contrast liver lesions has been one of the most well-­
media avoids the risk of thyroid function impair- established clinical applications of CEUS, with
ment [29]. Contrast agents are foreign materials several published reports claiming its success
and although it has a good safety profile, hyper- [34–36] (Fig.  9.2). Various studies have shown
sensitivity reactions have been reported in about that CEUS has an excellent accuracy in detecting
0.002% of abdominal studies [30, 31]. The inci- liver lesions, with a reported 87–91% sensitivity
dence of hypersensitivity in CEUS is lower than [37]. Contrast-enhanced imaging of liver lesions
the use of iodine contrast in CT and comparable were found to be related to phagocytosis of the
to the use of Gadolinium contrast in MRI [31]. UCA microbubbles [38]. CEUS is highly accu-
The main contraindications for the use of CEUS rate in differentiating benign and malignant liver
include patients with known coronary artery dis- lesions as well, with a sensitivity of 96–97.2%
ease, severe arrhythmias, right-to-left shunts, and a negative predictive value (NPV) of 94.1–
severe pulmonary hypertension, prosthetic 98.5% [39]. With such remarkable results in

Fig. 9.2  “File:Ultrasonography of a dysplastic nodule.jpg” by R. Badea and Simona Ioanitescu, CC BY 3.0
9  POCUS: What does the Future Hold? 193

liver imaging, CEUS has found its way into a world. Tele-medicine offers healthcare providers
wide variety of non-hepatic imaging applica- a unique opportunity to provide remote consults,
tions, examples of which are the detection and and at the same time increase the opportunity for
differentiation of kidney lesions, clearer delinea- patients globally to receive proper assessments
tion of pancreatic cancer and the adjacent ves- and diagnoses. As data transmission technology
sels, evaluation of aortic or other vascular in healthcare grows in parallel with the miniatur-
abnormalities, and the evaluation solid organ ization of ultrasound devices, this has spurred the
injuries in blunt abdominal trauma [29]. A novel development of the tele-ultrasound practice.
territory within the field of POCUS, CEUS has Tele-ultrasound could potentially eliminate a
gained popularity in the trauma setting, where it major limitation of traditional ultrasound, as with
was able to detect avascular laceration planes as the aid of a remote expert interpreter, the lack of
well as pseudoaneurysms and active bleeding image interpretation skills may no longer pose a
[40, 41]. The expansion of the use of CEUS in problem [43].
emergency medicine has recently garnered the The bulk of the tele-ultrasound application
interest of intensivists caring for critically ill can be found in the field of obstetric and fetal
COVID-19 patients. A case report by Yusuf et al. medicine, majority of which relates to the confir-
proved CEUS to be a useful tool in point-of-care mation of pregnancy, assessing for pregnancy-­
lung ultrasound. With its well-known ability to related complications, and monitoring fetal
distinguish perfused from avascular tissues, growth [44]. In recent years, the scope of tele-­
CEUS was able to identify microvascular lung ultrasound has reached various clinical applica-
infarcts classically seen in COVID-19 patients tions involving echocardiography to imaging of
with immune-mediated thrombotic events. The trauma patients as well as guided procedures [45,
authors also showed that the degree of lung 46]. The assessment of trauma patients relies
infarction correlated with clinical status, provid- heavily on physical examination supplemented
ing relevant prognostic and diagnostic informa- by the use of ultrasound for rapid diagnosis and
tion to guide management [42]. decision-making. The Focused Assessment with
CEUS is emerging as a promising imaging Sonography for Trauma (FAST) scan is tradition-
technique for POCUS applications and more ally performed by emergency medicine physi-
research into its diagnostic accuracy, healthcare cians to rapidly diagnose life-threatening injuries
costs, and patient safety profile is warranted to requiring immediate management. Recently, sys-
prove its value in various clinical settings. These tems have been developed to enable ultrasound-­
techniques when developed into critical care pro- novice paramedics to perform prehospital
tocols can help in early diagnosis, timely initia- ultrasound scans in trauma victims and obtain
tion of definitive management, and more FAST images under the remote direction of emer-
importantly minimize transfer of critical ill venti- gency medicine physicians, allowing the medical
lated patients to radiology suites. Where dissemi- team to obtain an early diagnosis and prepare for
nation of infective pathogens is a concern, the use any emergency procedures [47].
of these techniques help to prevent the occur- In the most extreme example of a remote loca-
rences of super spreader events from patient tran- tion, astronauts aboard the International Space
sit within the hospital. Station (ISS) with limited ultrasound training
were guided by remote experts in the Mission
Control Center (MCC) to aid the diagnosis of
9.5 Tele-Ultrasound several medical ailments commonly occurring in
crew members [48]. Examples of which are mus-
Tele-medicine has been on the rise in the past few culoskeletal pain secondary to joint contusions or
decades, but the recent COVID-19 pandemic has effusions, trauma injuries when crew members
given it a major boost in popularity, as healthcare engage in heavy-duty tasks, and even ocular inju-
systems become overwhelmed throughout the ries. Eye foreign bodies are a common problem
194 R. H. X. Chia and B. Ashokka

as small particles float freely in space. It was Self-Test MCQ (20 Questions)
reported that an untrained astronaut had used
ultrasound to perform an eye examination with 1. Fluid appears _____ on the ultrasound screen
remote guidance from the MCC, and was capable A. White
of visualizing the anatomical structures of the B. Black
globe, iris, and pupil [49]. C. Grey
As telecommunication systems and data trans- D. Not seen
mission technologies improve, tele-ultrasound 2. The linear probe has a range of frequency of
could potentially be ubiquitous around the globe. A. 2–5 MHz
The remote interpretation of ultrasound images B. 4–7 MHz
requires software, hardware, and video-­ C. 5–10 MHz
conferencing platforms which are costly and D. 10–15 MHz
could be a major barrier in implementing tele-­ 3. If the image on the monitor screen is too dark,
ultrasound in developing countries [50]. While you should
video streaming might require higher Internet A. Increase depth
streaming capacities, short video clips and popu- B. Use more ultrasound gel
lar 1 min social media and networking platforms C. Decrease gain
could help in rapid transmission of video content D. Increase gain
and timely interpretation by the experts. In low 4. Which of the following is a piezoelectric
resource settings and remote hospitals, there has material?
been some evidence of its clinical feasibility in A. Nickel
emergency medicine however more rigorous B. Tourmaline
research is needed to evaluate its diagnostic accu- C. Iron
racy and effectiveness before tele-ultrasound can D. None of the above
be fully utilized with confidence [51]. 5. Speed of ultrasound is the fastest when travel-
ing through
A. Bone
9.6 Conclusion B. Air
C. Water
The introduction of POCUS has revolutionized D. Muscle
the process of bringing diagnostic imaging possi- 6. Properties of sound waves include
bilities and clinical interpretation is possible in a A. Amplitude
more time-sensitive manner. The advances in B. Frequency
technology and data processing have helped in the C. Wavelength
development of pattern recognition software that D. All of the above
enhance the accuracy of decision-making through 7. The main factor affecting attenuation is
large data points from machine learning. Applying A. Scattering
these could enhance the safety of healthcare, min- B. Divergence
imizing errors and potentially reducing healthcare C. Absorption
costs. Bulkier imaging equipment and long wait D. Reflection
in obtaining specialist and expert opinions can 8. Which of the following are not reported ben-
now be supplemented with portable, hand help efits of using artificial intelligence (AI) in
and modern technology. With the advent of ultrasonography?
enhanced connectivity help is now available rap- A. Improved survival rates
idly. These advances in ultrasound technology B. Enhanced diagnostic accuracy
and the application of POCUS are a welcome C. Wide clinical applications
promise of the near future with scope for reduc- D. Image capture assistance
tion in patient morbidity and mortality.
9  POCUS: What does the Future Hold? 195

9. Which of the following statements regarding 15. Which of the following statements regarding
artificial intelligence (AI) is FALSE? contrast-enhanced ultrasound (CEUS) is
A. Deep learning technology is one of the FALSE?
machine learning techniques used in AI A. The US was the first country to approve
B. AI-powered systems may lower false-­ the use of CEUS
positive rates in cancer detection B. CEUS may be used in pediatric patients
C. Untrained operators may obtain diagnos- C. POCUS lung ultrasound may benefit
tic quality imaging from CEUS in detecting lung infarcts
D. AI allows automated calculation of car- D. CEUS is highly accurate in differentiat-
diac ejection fraction ing benign and malignant lesions
10. Which of the following statements regarding 16. The gold standard for diagnosis of hepato-
handheld ultrasound is FALSE? cellular carcinoma (HCC) is
A. Piezoelectric crystals have a lower cost A. Computed tomography (CT) perfusion
of production compared to Capacitive B. Liver biopsy
Micromachined Ultrasound Transducers C. Contrast-enhanced ultrasound (CEUS)
(CMUT) D. Magnetic resonance imaging (MRI)
B. Devices may use single or multiple 17. In contrast-enhanced ultrasound of the liver,
probes microbubbles are taken up by
C. 2D imaging is classically used A. Sinusoid cells
D. Epidural depth is one of the parameters B. Kupffer cells
that can be measured C. Stellate cells
11. Which of the following is not a benefit of D. Hepatocytes
handheld ultrasound compared to traditional 18. Which of the following regarding POCUS
cart-based ultrasound? lung ultrasound is FALSE?
A. Low cost A. B-lines are artifacts caused by acoustic
B. Improved image quality impedance due to the underlying lung
C. Portable B. Loss of lung sliding is sensitive to
D. Reduced examination time pneumothorax
12. Current ultrasound contrast agents in the
C. In M-mode, the “barcode” sign suggests
market consist of a pneumothorax
A. Hydrofluorocarbons D. “Lung point” is the most specific sign of
B. Agarose gel pneumothorax
C. Galactose and air 19. Which of the following regarding the FAST
D. Sulfur hexafluoride exam is true?
13. The incidence of hypersensitivity reactions A. It leads to few diagnostic peritoneal lavages
with the use of contrast-enhanced ultrasound B. FAST can be repeated for serial
(CEUS) is reported to be examinations
A. 2% C. It is safe for use in pregnant and pediatric
B. 0.2% patients
C. 0.002% D. All of the above
D. 0.0002% 20. Regarding the FAST exam
14. Which of the following is not a contraindica- A. FAST is more sensitive in obese patients
tion of contrast-enhanced ultrasound (CEUS) B. Solid organ injuries are easily identified
A. Coronary artery disease C. Peritoneal free fluid will not be detected
B. Acute respiratory distress syndrome until more than 500 ml is present
C. Acute kidney injury D. Diagnostic accuracy differs significantly
D. Arrhythmias between radiologists and non-
radiologists
196 R. H. X. Chia and B. Ashokka

References ultrasound probes: the new generation of POCUS.  J


Cardiothorac Vasc Anesth. 2020;34(11):3139–45.
https://doi.org/10.1053/j.jvca.2020.07.004.
1. Wang G.  A perspective on deep imaging. IEEE
13. Malik AN, Rowland J, Haber BD, et  al. The use of
Access. 2016;4:8914–24.
handheld ultrasound devices in emergency medicine.
2. Liu S, Wang Y, Yang X, Lei B, Liu L, Li SX, Ni D,
Curr Emerg Hosp Med Rep. 2021;9(3):73–81. https://
Wang T. Deep learning in medical ultrasound analy-
doi.org/10.1007/s40138-­021-­00229-­6.
sis: a review. Engineering. 2019;5(2):261–75. https://
14. Galusko V, Bodger O, Ionescu A. A systematic review
doi.org/10.1016/j.eng.2018.11.020. ISSN 2095-8099
of pocket-sized imaging devices: small and mighty?
3. U.S. Food and Drug Administration. FDA authorizes
Echo Res Pract. 2018;5(4):113–38.
marketing of first cardiac ultrasound software that
15. Zardi EM, Franceschetti E, Giorgi C, Palumbo A,
uses artificial intelligence to guide user. 2020. https://
Franceschi F.  Accuracy and performance of a new
www.fda.gov/news-­events/press-­a nnouncements/
handheld ultrasound machine with wireless system.
fda-­authorizes-­marketing-­first-­cardiac-­ultrasound-­
Sci Rep. 2019;9(1):14599. https://doi.org/10.1038/
software-­uses-­artificial-­intelligence-­guide-­user
s41598-­019-­51160-­6.
4. Cheema BS, Walter J, Narang A, Thomas JD. Artificial
16. European Society of Radiology (ESR). ESR state-
intelligence-enabled POCUS in the COVID-19
ment on portable ultrasound devices. Insights
ICU: a new spin on cardiac ultrasound. JACC Case
Imaging. 2019;10(1):89. https://doi.org/10.1186/
Rep. 2021;3(2):258–63. https://doi.org/10.1016/j.
s13244-­019-­0775-­x.
jaccas.2020.12.013.
17. Barreiros AP, Cui XW, Ignee A, De Molo C, Pirri
5. Volpato V, Mor-Avi V, Narang A, et  al. Automated,
C, Dietrich CF.  EchoScopy in scanning abdominal
machine learning-based, 3D echocardiographic quan-
diseases: initial clinical experience. Z Gastroenterol.
tification of left ventricular mass. Echocardiography.
2014;52:269–75.
2019;36(2):312–9. https://doi.org/10.1111/
18. Stock KF, Klein B, Steubl D, Lersch C, Heemann U,
echo.14234.
Wagenpfeil S, Eyer F, Clevert D-A. Comparison of a
6. Medvedofsky D, Mor-Avi V, Byku I, et  al. Three-­
pocket-size ultrasound device with a premium ultra-
dimensional echocardiographic automated quan-
sound machine: diagnostic value and time required
tification of left heart chamber volumes using an
in bedside ultrasound examination. Abdom Imaging.
adaptive analytics algorithm: feasibility and impact
2015;40:2861–6.
of image quality in nonselected patients. J Am Soc
19. Andrea S, Giovanna L, Pietro C, Luca F.  Teaching
Echocardiogr. 2017;30(9):879–85. https://doi.
echoscopy for the early diagnosis of ascites in cir-
org/10.1016/j.echo.2017.05.018.
rhosis: assessment of an objective structured clinical
7. Huang C, Zhou Y, Tan W, et al. Applying deep learn-
examination (OSCE). J Ultrasound. 2017;20:123–6.
ing in recognizing the femoral nerve block region on
20. Esposito R, Ilardi F, Schiano Lomoriello V, Sorrentino
ultrasound images. Ann Transl Med. 2019;7(18):453.
R, Sellitto V, Giugliano G, Esposito G, Trimarco B,
https://doi.org/10.21037/atm.2019.08.61.
Galderisi M. Identification of the main determinants
8. Smistad E, Johansen KF, Iversen DH, Reinertsen
of abdominal aorta size: a screening by pocket size
I.  Highlighting nerves and blood vessels for
imaging device. Cardiovasc Ultrasound. 2017;15:2.
ultrasound-­ guided axillary nerve block procedures
21. Bonnafy T, Lacroix P, Desormais I, Labrunie A,
using neural networks. J Med Imaging (Bellingham).
Marin B, Leclerc A, Oueslati A, Rollé F, Vignon P,
2018;5(4):044004. https://doi.org/10.1117/1.
Aboyans V.  Reliability of the measurement of the
JMI.5.4.044004.
abdominal aortic diameter by novice operators using
9. KKH collaborates with NUS to develop world-first
a pocket-sized ultrasound system. Arch Cardiovasc
AI-powered system to enhance accuracy of spinal
Dis. 2013;106:644–50.
anaesthesia. https://www.kkh.com.sg/news/research/
22. Dijos M, Pucheux Y, Lafitte M, Réant P, Prevot
kkh-­collaborates-­with-­nus-­to-­develop-­world-­first-­
A, Mignot A, Barandon L, Roques X, Roudaut R,
ai-­powered-­system-­to-­enhance-­accuracy-­of-­spinal-­
Pilois X, et  al. Fast track echo of abdominal aortic
anaesthesia
aneurysm using a real pocket-ultrasound device at
10. Becker AS, Marcon M, Ghafoor S, Wurnig MC,
bedside. Echocardiography (Mount Kisco, NY).
Frauenfelder T, Boss A. Deep learning in mammog-
2012;29:285–90.
raphy: diagnostic accuracy of a multipurpose image
23. Grau T, Leipold RW, Conradi R, Martin E, Motsch
analysis software in the detection of breast cancer.
J.  Ultrasound imaging facilitates localization of the
Investig Radiol. 2017;52:434–40.
epidural space during combined spinal and epidural
11. Baran JM, Webster JG.  Design of low-cost por-
anesthesia. Reg Anesth Pain Med. 2001;26:64–7.
table ultrasound systems. In Annual International
24. Seligman KM, Weiniger CF, Carvalho B.  The accu-
Conference of the IEEE Engineering in Medicine and
racy of a handheld ultrasound device for neuraxial
Biology Society, IEEE; 2009. pp. 792–795.
depth and landmark assessment: a prospective cohort
12. Baribeau Y, Sharkey A, Chaudhary O, Krumm S,
trial. Anesth Analg. 2018;126(6):1995–8. https://doi.
Fatima H, Mahmood F, et al. Handheld point-of-care
org/10.1213/ANE.0000000000002407.
9  POCUS: What does the Future Hold? 197

25. Carvalho B, Seligman KM, Weiniger CF. The compar- 38. Yanagisawa K, Moriyasu F, Miyahara T, Yuki M,
ative accuracy of a handheld and console ultrasound Iijima H.  Phagocytosis of ultrasound contrast agent
device for neuraxial depth and landmark assessment. microbubbles by Kupffer cells. Ultrasound Med
Int J Obstet Anesth. 2019;39:68–73. https://doi. Biol. 2007;33(2):318–25. https://doi.org/10.1016/j.
org/10.1016/j.ijoa.2019.01.004. Epub 2019 Jan 11 ultrasmedbio.2006.08.008.
26. Wilson SR, Greenbaum LD, Goldberg BB. Contrast-­ 39. Sawatzki M, Meyenberger C, Brand S, Semela
enhanced ultrasound: what is the evidence and D. Contrast-enhanced ultrasound (CEUS) has excel-
what are the obstacles? AJR Am J Roentgenol. lent diagnostic accuracy in differentiating focal liver
2009;193(1):55–60. https://doi.org/10.2214/ lesions: results from a Swiss tertiary gastroentero-
AJR.09.2553. logical centre. Swiss Med Wkly. 2019;149:w20087.
27. Greis C.  Technology overview: SonoVue (Bracco, https://doi.org/10.4414/smw.2019.20087.
Milan). Eur Radiol. 2004;14(Suppl 8):P11–5. 40. Sidhu PS, Cantisani V, Dietrich CF, et  al. The
28. Sontum PC.  Physicochemical characteristics of EFSUMB guidelines and recommendations for the
Sonazoid, a new contrast agent for ultrasound imag- clinical practice of contrast-enhanced ultrasound
ing. Ultrasound Med Biol. 2008 May;34(5):824–33. (CEUS) in non-hepatic applications: update 2017
29. Chung YE, Kim KW. Contrast-enhanced ultrasonog- (long version). Ultraschall Medi. 2018;39(02):
raphy: advance and current status in abdominal imag- e2–e44.
ing. Ultrasonography. 2015;34(1):3–18. https://doi. 41. Catalano O, Aiani L, Barozzi L, Bokor D, et al. CEUS
org/10.14366/usg.14034. in abdominal trauma: multi-center study. Abdom
30. Piscaglia F, Bolondi L. Italian Society for ultrasound Imaging. 2009;34(2):225–34.
in medicine and biology (SIUMB) study group on 42. Yusuf GT, Wong A, Rao D, et al. The use of contrast-­
ultrasound contrast agents the safety of Sonovue enhanced ultrasound in COVID-19 lung imag-
in abdominal applications: retrospective analy- ing. J Ultrasound. 2020;25(2):319–23. https://doi.
sis of 23188 investigations. Ultrasound Med Biol. org/10.1007/s40477-­020-­00517-­z.
2006;32:1369–75. 43. Whitson MR, Mayo PH.  Ultrasonography in the
31. Sidhu PS, Choi BI, Nielsen MB.  The EFSUMB emergency department. Crit Care. 2016;20:227.
guidelines on the nonhepatic clinical applications of Pmid:27523885
contrast enhanced ultrasound (CEUS): a new dawn 44. Ferreira AC, Mahony E, Oliani AH, Junior AE,
for the escalating use of this ubiquitous technique. da Silva CF.  Teleultrasound: historical perspec-
Ultraschall Med. 2012;33:5–7. tive and clinical application. Int J Telemed Appl.
32. The European Agency for the Evaluation of Medicinal 2015;2015:306259. pmid:25810717
Products. Public statement on SONOVUE (Sulphur 45. Sheehan FH, Ricci MA, Murtagh C, Clark H,
hexafluoride) new contraindication in patients with Bolson EL.  Expert visual guidance of ultrasound
heart disease: restriction of use to non-cardiac imag- for telemedicine. J Telemed Telecare. 2010;16(2):
ing. London: The European Agency for the Evaluation 77–82.
of Medicinal Products; 2014. 46. Law J, Macbeth PB. Ultrasound: from earth to space.
33. Seitz K, Strobel D.  A milestone: approval of CEUS McGill J Medi. 2011;13(2):59–65.
for diagnostic liver imaging in adults and children 47. Pian L, Gillman LM, McBeth PB, et  al. Potential
in the USA.  Ultraschall Med. 2016;37(3):229–32. use of remote telesonography as a transformational
English. https://doi.org/10.1055/s-­0042-­107411. technology in underresourced and/or remote settings.
34. Wilson SR, Jang HJ, Kim TK, Burns PN. Diagnosis Emerg Med Int. 2013;2013:986160.
of focal liver masses on ultrasonography: compari- 48. Law J, Macbeth PB. Ultrasound: from earth to space.
son of unenhanced and contrast-enhanced scans. J Mcgill J Med. 2011;13(2):59.
Ultrasound Med. 2007;26(6):775–87; quiz 788-90 49. Chiao L, Sharipov S, Sargsyan AE, Melton S,
35. Leen E, Ceccotti P, Kalogeropoulou C, Angerson WJ, Hamilton DR, McFarlin K, Dulchavsky SA.  Ocular
Moug SJ, Horgan PG.  Prospective multicenter trial examination for trauma; clinical ultrasound
evaluating a novel method of characterizing focal aboard the International Space Station. J Trauma.
liver lesions using contrast-enhanced sonography. 2005;58(5):885–9.
AJR Am J Roentgenol. 2006;186(6):1551–9. 50. Crawford I, McBeth PB, Mitchelson M, Ferguson
36. Ding H, Wang WP, Huang BJ.  Imaging of focal J, Tiruta C, Kirkpatrick AW.  How to set up a low
liver lesions low-mechanical-index real-time ultra- cost tele-ultrasound capable videoconferencing
sonography with SonoVue. J Ultrasound Med. system with wide applicability. Crit Ultrasound J.
2005;24(3):285–97. 2012;4(1):13.
37. Cabassa P, Bipat S, Longaretti L, Morone M, Maroldi 51. Marsh-Feiley G, Eadie L, Wilson P. Telesonography
R.  Liver metastases: Sulphur hexafluoride-enhanced in emergency medicine: a systematic review. PLoS
ultrasonography for lesion detection: a systematic One. 2018;13(5):e0194840. https://doi.org/10.1371/
review. Ultrasound Med Biol. 2010;36(10):1561–7. journal.pone.0194840.
Answer Keys

Chapter 1 Chapter 7

Answers: 1C, 2D, 3A, 4C, 5D, 6B, 7C, 8B, 9D, Answers: 1D, 2C, 3A, 4B, 5A, 6C, 7B, 8C, 9D,
10B 10B, 11C, 12B, 13A, 14D, 15B, 16A, 17D, 18D,
19A, 20B

Chapter 2
Chapter 8
Answers: 1E, 2B, 3E, 4E, 5D, 6E, 7E, 8E, 9A,
10D, 11E, 12A, 13B, 14A, 15E, 16B, 17C, 18D, Answers: 1B.  Lateral resolution is determined
19C, 20D primarily by the width of the ultrasound beam.
The focal zone, or narrowest portion of the ultra-
sound beam, is where the highest lateral resolu-
Chapter 3 tion is obtained. Ideally, the target structure(s)
should be positioned within the focal zone to
Answers: 1A, 2D, 3B, 4C, 5C, 6A, 7D, 8C, 9D, maximize resolution.
10D, 11A, 12C, 13B, 14B, 15A, 16C, 17A, 18A, 2C.  Linear transducers produce the highest
19C, 20D resolution images of superficial structures
(<6  cm) as they transmit high-frequency ultra-
sound waves in a parallel orientation. Linear
Chapter 4 transducers are used to image any superficial
structures, including blood vessels, skin/soft tis-
Answers: 1C, 2A, 3B, 4C, 5C, 6C, 7A, 8C, 9C, sues, joints, eyes, testicles, thyroid, lymph nodes,
10D, 11C, 12D, 13B, 14A, 15D, 16B, 17A, 18F, and nerves.
19D, 20A 3B. Only the anterior portion of the bladder is
visualized. Increasing the depth will allow visu-
alization of the entire bladder, including the pos-
Chapter 6 terior wall.
4C.  Fluid-filled structures serve as acoustic
Answers: 1C, 2C, 3C, 4D, 5D, 6B, 7D, 8B, 9B, windows for sound waves to travel into the
10C, 11B, 12B, 13D, 14A, 15D, 16E, 17E, 18B, abdomino-pelvic cavity to generate ultrasound
19E, 20D images. The blood-filled liver and spleen serve as
acoustic windows in the right and left upper
quadrants. Similarly, a urine-filled bladder is an

© The Editor(s) (if applicable) and The Author(s), under exclusive license to Springer Nature
Singapore Pte Ltd. 2022
A. Chakraborty, B. Ashokka (eds.), A Practical Guide to Point of Care Ultrasound (POCUS), 199
https://doi.org/10.1007/978-981-16-7687-1
200 Answer Keys

ideal acoustic window for the visualization of 10C. The differential diagnosis for traumatic
pelvic structures. Gas-filled loops of the bowel vision loss includes corneal abrasion, hyphema,
scatter sound waves and prevent visualization of traumatic iritis, lens dislocation, foreign body,
deep structures in the abdomen and pelvis. retinal detachment, vitreous hemorrhage, trau-
Subcutaneous emphysema and asplenia also limit matic optic neuropathy, acute maculopathy, and
ultrasound imaging of the abdomino-pelvic globe rupture. Ocular ultrasound serves as a use-
cavity. ful tool in the setting of trauma to assess pupil-
5B. A small amount of physiologic free fluid lary function, extraocular muscle function, lens
may be detected with point-of-care ultrasound in position, and globe abnormalities, and to evaluate
the pelvic cavity of women. However, in women for evidence of increased intracranial pressure.
with a positive pregnancy test and abdominal All of the conditions listed above can be diag-
pain, the presence of abdomino-pelvic free fluid nosed by ocular ultrasound, except corneal
is an ectopic pregnancy until proven otherwise. abrasion.
6D.  Acoustic enhancement, or posterior 11C.  The subarachnoid space in the optic
acoustic enhancement, is an artifact that is seen nerve sheath does not dilate uniformly. The most
deep in fluid-filled structures. Due to the low pronounced response to increased fluid in the
acoustic impedance of a fluid-filled body, such as subarachnoid space due to increased intracranial
the bladder, a disproportionately high number of pressure occurs 3  mm posterior to the optic
echoes return to the transducer from tissues deep nerve—retina junction. The optic nerve sheath
into the fluid-filled body, resulting in these tis- diameter should be measured in transverse and
sues appearing hyperechoic, or brighter, than sagittal planes and compared to measurements
usual. obtained on the contralateral side. An optic nerve
7D. Power Doppler is particularly well suited sheath diameter greater than 5 mm is considered
for low-flow states, such as ureteral jets. The ure- abnormal in patients with clinical concern for
teral jets appear as bright yellow-orange emis- elevated intracranial pressure.
sions streaming from the base of the bladder 12D. It is true that transducers that have only
towards its center. Power Doppler does not dis- been in contact with intact skin can be cleaned
play directional information, like color flow with a nonabrasive soap or with low- or interme-
Doppler. None of the other imaging modalities diate-level disinfectant wipes. Statements A
listed is used for the detection of ureteral jets. through C are false. Use of disposable sterile
8D.  Past research has demonstrated a close transducer covers followed by cleaning with a
correlation between the estimated bladder vol- nonabrasive soap and high-level disinfection is
ume using the formula above (bladder vol- recommended for endocavitary transducers that
ume = 0.75 × width × length × height) compared have been in contact with mucous membranes.
to the actual catheterized volume (correlation Transducers exposed to patients with Clostridium
factor  =  0.983). It is noteworthy that bladder difficile infection should be cleaned with a hypo-
shape has a considerable impact on the accuracy chlorite-based or hydrogen peroxide-based solu-
of US estimation of bladder volume. Though tion to kill the bacterial spores. Ultrasound
applying the corresponding correction coefficient transducers should never be autoclaved or sub-
to volume calculations will improve the accuracy jected to high heat, electricity, or pressure
of the estimation, the correction factor 0.75 yields because the piezoelectric elements can be
the closest approximation regardless of bladder damaged.
shape. 13D. The transducer is placed in the parasagit-
9A. The prostate sits deep in the bladder and tal plane of the epigastric region to obtain general
normally encircles the bladder neck. Normally, qualitative observations of the gastric antrum and
the prostate measures less than 5 cm in transverse body, the stomach cavity, and the stomach con-
diameter. tents. It is scanned curving from right to left. The
Answer Keys 201

antrum is generally seen in the parasagittal plane Chapter 9


immediately to the right of the midline. Reference
points to be considered are the left anterior lobe Answers: 1B, 2C, 3D, 4B, 5A, 6D, 7C, 8A, 9B,
or caudal lobe of the liver, the pancreas head, and 10A, 11B, 12D, 13C, 14C, 15A, 16B, 17B, 18A,
the superior mesenteric vessels with the vena 19D, 20C
cava inferior or abdominal aorta.

You might also like